Eyes, Ears, and Nose Exam
Quiz-summary
0 of 56 questions completed
Questions:
- 1
- 2
- 3
- 4
- 5
- 6
- 7
- 8
- 9
- 10
- 11
- 12
- 13
- 14
- 15
- 16
- 17
- 18
- 19
- 20
- 21
- 22
- 23
- 24
- 25
- 26
- 27
- 28
- 29
- 30
- 31
- 32
- 33
- 34
- 35
- 36
- 37
- 38
- 39
- 40
- 41
- 42
- 43
- 44
- 45
- 46
- 47
- 48
- 49
- 50
- 51
- 52
- 53
- 54
- 55
- 56
Information
NHSPA Eyes, Ears, and Nose Self-Assessment Exam: Approved for 5 category I AAPA Credits
After completion of the NHSPA Eyes, Ears, and Nose self-assessment exam PAs should be more familiar with the different types of conjunctivitis, be able to identify eye pathology associated with certain medical conditions and aging, be able to identify causes of acute vision loss, be able to manage eye infections and injuries, be able to interpret fluorescein exams, be able to identify and manage a pterygium, chalazion, orbital cellulitis, and hyphema be able to treat ceruman impaction and perform foreign body removal, know how to treat acute otitis external and media, subchondral hematoma of the ear, identify causes of sensorineural hearing loss, barotrauma, eustachian tube dysfunction, and vertigo. Be able to dentify abnormal growths in the inner ear and external auditory canal, identify ear drum pathology, know when to initiate audiometric testing, know causes and treatment of epistaxis, identifying and treat nasal growths and nasal trauma.
You have already completed the quiz before. Hence you can not start it again.
Quiz is loading...
You must sign in or sign up to start the quiz.
You have to finish following quiz, to start this quiz:
Results
0 of 56 questions answered correctly
Your time:
Time has elapsed
You have reached 0 of 0 points, (0)
Average score |
|
Your score |
|
Categories
- Not categorized 0%
-
You are required to complete the post-exam evaluation to receive CME credits. Please complete the post-exam evaluation here. Your CME certificate will be emailed to you shortly. If you don’t receive your certificate today, and the certificate email from CME4PAC is not located in your spam folder, please email, Dagan Cloutier, at dcloutier@cme4pac.com.
- 1
- 2
- 3
- 4
- 5
- 6
- 7
- 8
- 9
- 10
- 11
- 12
- 13
- 14
- 15
- 16
- 17
- 18
- 19
- 20
- 21
- 22
- 23
- 24
- 25
- 26
- 27
- 28
- 29
- 30
- 31
- 32
- 33
- 34
- 35
- 36
- 37
- 38
- 39
- 40
- 41
- 42
- 43
- 44
- 45
- 46
- 47
- 48
- 49
- 50
- 51
- 52
- 53
- 54
- 55
- 56
- Answered
- Review
-
Question 1 of 56
1. Question
A patient presents complaining of left eye redness, discharge, and minimal discomfort for 24 hours. There is no history of contact lens use or any known trauma. Physical exam reveals the accompanying image with normal visual acuity. What is the most appropriate treatment option?Correct
The correct answer is C. The history and exam are most consistent with a bacterial conjunctivitis which can be self-limiting within 1-2 weeks but generally resolves quicker with antibiotic drops or ointment. Artificial tears may be helpful in patients with allergic conjunctivitis or dry eye disease. Cool or warm compresses and naphazoline-pheniramine drops can be beneficial for both viral and allergic etiologies. Steroid drops may be indicated for a corneal ulcer but should not be initiated without consultation with an ophthalmologist.
Bacterial conjunctivitis typically manifests as redness and discharge in one eye more commonly than bilateral. The purulent yellow, white, or green discharge occurs throughout the day and quickly reaccumulates after wiping it away manually. The color, consistency, and frequency of discharge is typically what separates it from other types of conjunctivitis. Staph aureus is the most common pathogen in adults with others like Strep pneumoniae and H. flu more common in the pediatric patient. Chlamydia and gonorrhea should be considered initially or in cases not responding to treatment. Bacterial etiologies are highly contagious, and every effort should be made to wash hands frequently and avoid sharing things such as glasses, towels, cosmetics, etc. with others in order to avoid spread.
In addition to the previously mentioned erythromycin drops, trimethoprim-polymyxin B (Bactrim) drops are an option as well as bacitracin or bacitracin-polymyxin B (Polycin) ointment. If the patient is a contact lens user, a fluoroquinolone such as ciprofloxacin or a 4th generation such as moxifloxacin (Vigamox) should be first-line treatment. Patients should be advised to stay out of work or school for a minimum of 24 hours after starting the topical antibiotics to minimize spread.
References:
A. UpToDate
B. https://www.ncbi.nlm.nih.gov/pmc/articles/PMC4049531/Incorrect
The correct answer is C. The history and exam are most consistent with a bacterial conjunctivitis which can be self-limiting within 1-2 weeks but generally resolves quicker with antibiotic drops or ointment. Artificial tears may be helpful in patients with allergic conjunctivitis or dry eye disease. Cool or warm compresses and naphazoline-pheniramine drops can be beneficial for both viral and allergic etiologies. Steroid drops may be indicated for a corneal ulcer but should not be initiated without consultation with an ophthalmologist.
Bacterial conjunctivitis typically manifests as redness and discharge in one eye more commonly than bilateral. The purulent yellow, white, or green discharge occurs throughout the day and quickly reaccumulates after wiping it away manually. The color, consistency, and frequency of discharge is typically what separates it from other types of conjunctivitis. Staph aureus is the most common pathogen in adults with others like Strep pneumoniae and H. flu more common in the pediatric patient. Chlamydia and gonorrhea should be considered initially or in cases not responding to treatment. Bacterial etiologies are highly contagious, and every effort should be made to wash hands frequently and avoid sharing things such as glasses, towels, cosmetics, etc. with others in order to avoid spread.
In addition to the previously mentioned erythromycin drops, trimethoprim-polymyxin B (Bactrim) drops are an option as well as bacitracin or bacitracin-polymyxin B (Polycin) ointment. If the patient is a contact lens user, a fluoroquinolone such as ciprofloxacin or a 4th generation such as moxifloxacin (Vigamox) should be first-line treatment. Patients should be advised to stay out of work or school for a minimum of 24 hours after starting the topical antibiotics to minimize spread.
References:
A. UpToDate
B. https://www.ncbi.nlm.nih.gov/pmc/articles/PMC4049531/ -
Question 2 of 56
2. Question
A 26 year old male patient presents to the office with bilateral eye redness and associated itching sensation but no photophobia. No contact lens use is reported. Exam reveals bilateral conjunctival injection and chemosis along with a thin, watery discharge. Which of the following is the most appropriate diagnosis?
Correct
The correct answer is A. Bilateral conjunctival injections with a predominant complaint of associated pruritis (Corneal ulcers are more commonly unilateral, painful, and associated with contact lens use. Ultraviolet keratitis is extremely painful and may result in tearing but generally not something that would be considered “discharge”. Viral conjunctivitis is most similar to allergic with the primary difference being a gritty sensation rather than itching. The discharge may also appear to more stringy rather than watery. Associated symptoms may also help differentiate allergic from viral. Primarily sneezing or other symptoms of seasonal allergies as opposed to fever, thickened rhinorrhea, pharyngitis, or adenopathy associated with typical upper respiratory tract viral infections.
Patients with allergic conjunctivitis may also have a seasonal history of similar events, atopic dermatitis, or report a recent exposure to pets, dust, etc. They may report that rubbing the eyes worsens the symptoms and have associated swelling and darkening of the lower lids and surrounding areas known as “allergic shiners”.
Therapy for allergic conjunctivitis consists of a combination of either an ocular decongestant or mast cell stabilizer plus antihistamine. Vasoconstrictor/antihistamine combinations are available as prescription or OTC and include naphazoline-pheniramine (Naphcon-A, Opcon-A, Visine-A). They should be used for no more than 2 weeks to prevent rebound effects. Mast cell stabilizer/antihistamine options include olopatadine (Patanol), alcaftadine (Lastacaft), bepotastine (Bepreve), and azelastine hydrochloride (Optivar), amongst others. These drugs may take up to 2 weeks to demonstrate maximum effect. Oral antihistamines should also be considered. Supportive care instructions can also include the use of cool compresses to the eyes or refrigerated artificial tears as needed and avoiding contact lenses until symptoms resolve.References:
A. UpToDate
B. https://www.ncbi.nlm.nih.gov/pmc/articles/PMC4049531/
C. https://www.aafp.org/afp/2016/0601/p915.htmlIncorrect
The correct answer is A. Bilateral conjunctival injections with a predominant complaint of associated pruritis (Corneal ulcers are more commonly unilateral, painful, and associated with contact lens use. Ultraviolet keratitis is extremely painful and may result in tearing but generally not something that would be considered “discharge”. Viral conjunctivitis is most similar to allergic with the primary difference being a gritty sensation rather than itching. The discharge may also appear to more stringy rather than watery. Associated symptoms may also help differentiate allergic from viral. Primarily sneezing or other symptoms of seasonal allergies as opposed to fever, thickened rhinorrhea, pharyngitis, or adenopathy associated with typical upper respiratory tract viral infections.
Patients with allergic conjunctivitis may also have a seasonal history of similar events, atopic dermatitis, or report a recent exposure to pets, dust, etc. They may report that rubbing the eyes worsens the symptoms and have associated swelling and darkening of the lower lids and surrounding areas known as “allergic shiners”.
Therapy for allergic conjunctivitis consists of a combination of either an ocular decongestant or mast cell stabilizer plus antihistamine. Vasoconstrictor/antihistamine combinations are available as prescription or OTC and include naphazoline-pheniramine (Naphcon-A, Opcon-A, Visine-A). They should be used for no more than 2 weeks to prevent rebound effects. Mast cell stabilizer/antihistamine options include olopatadine (Patanol), alcaftadine (Lastacaft), bepotastine (Bepreve), and azelastine hydrochloride (Optivar), amongst others. These drugs may take up to 2 weeks to demonstrate maximum effect. Oral antihistamines should also be considered. Supportive care instructions can also include the use of cool compresses to the eyes or refrigerated artificial tears as needed and avoiding contact lenses until symptoms resolve.References:
A. UpToDate
B. https://www.ncbi.nlm.nih.gov/pmc/articles/PMC4049531/
C. https://www.aafp.org/afp/2016/0601/p915.html -
Question 3 of 56
3. Question
What is the most common risk factor for the findings in the attached image?Correct
The correct answer is A. The image demonstrates the formation of a cataract as noted by the white opacity in the center of the eye. All of the listed answers are known causes of cataract formation with age being the most common. Cataracts may start to develop between 40-50 years of age. Other risk factors include smoking, malnutrition, corticosteroid use, trauma, lead ingestion, and HIV/AIDS.
The cells that comprise the optic lens are highly specialized and cannot be shed if damaged. Instead, they are compressed toward the center of the lens eventually lose their transparency. Patients are more likely to have bilateral involvement. Before the visible cloudiness is apparent, a patient may initially develop nearsightedness. Once the lens becomes cloudy, a patient may experience difficult driving at night due to oncoming headlights, a “halo” effect from lights, or a declining ability to read fine print. Physical exam will usually reveal some amount of opacification or darkening/absence of the red reflex with ophthalmoscope exam. Cataracts by themselves are painless, however, a small percentage of patients may report pain or present with ocular injection which should raise the concern of a secondary glaucoma.
Cataracts almost always continue to progress in size and opacity however, some that are primarily related to smoking or other substance ingestion may slow or somewhat reverse if the substance is discontinued. The treatment for cataracts is surgical and the decision to pursue surgery is usually dictated by the visual impact on the patient’s activities of daily living. During the same procedure, a synthetic lens is usually implanted. Mild post-operative pain is expected for 1-2 days. Post-operative complications can include infection, corneal edema, lens dislocation, anterior chamber inflammation (toxic anterior segment syndrome), and delayed retinal detachment, macular degeneration, or posterior capsule opacification.
References:
A. UpToDate
B. https://www.hopkinsmedicine.org/health/conditions-and-diseases/cataractsIncorrect
The correct answer is A. The image demonstrates the formation of a cataract as noted by the white opacity in the center of the eye. All of the listed answers are known causes of cataract formation with age being the most common. Cataracts may start to develop between 40-50 years of age. Other risk factors include smoking, malnutrition, corticosteroid use, trauma, lead ingestion, and HIV/AIDS.
The cells that comprise the optic lens are highly specialized and cannot be shed if damaged. Instead, they are compressed toward the center of the lens eventually lose their transparency. Patients are more likely to have bilateral involvement. Before the visible cloudiness is apparent, a patient may initially develop nearsightedness. Once the lens becomes cloudy, a patient may experience difficult driving at night due to oncoming headlights, a “halo” effect from lights, or a declining ability to read fine print. Physical exam will usually reveal some amount of opacification or darkening/absence of the red reflex with ophthalmoscope exam. Cataracts by themselves are painless, however, a small percentage of patients may report pain or present with ocular injection which should raise the concern of a secondary glaucoma.
Cataracts almost always continue to progress in size and opacity however, some that are primarily related to smoking or other substance ingestion may slow or somewhat reverse if the substance is discontinued. The treatment for cataracts is surgical and the decision to pursue surgery is usually dictated by the visual impact on the patient’s activities of daily living. During the same procedure, a synthetic lens is usually implanted. Mild post-operative pain is expected for 1-2 days. Post-operative complications can include infection, corneal edema, lens dislocation, anterior chamber inflammation (toxic anterior segment syndrome), and delayed retinal detachment, macular degeneration, or posterior capsule opacification.
References:
A. UpToDate
B. https://www.hopkinsmedicine.org/health/conditions-and-diseases/cataracts -
Question 4 of 56
4. Question
A patient presents with eye pain for 24 hours and denies any trauma prior to onset. Exam reveals obvious photophobia, mild injection, round pupil, and clear anterior and posterior chambers. Fluorescein exam under slit lamp is shown in the attached image. What is the most appropriate treatment?Correct
The correct answer is B. This image is consistent with a corneal ulcer which should be aggressively managed using a topical antibiotic which covers Pseudomonas. Erythromycin does not adequately cover Pseudomonas. Naphazoline-pheniramine is used for symptoms associated with allergic conjunctivitis, which is typically bilateral, itchy, and does not cause photophobia or staining on exam. Proparacaine is a topical anesthetic generally used in the evaluation of eye pain to help facilitate the exam but should not be used indefinitely in order to avoid secondary corneal epithelial defects or even perforation. The slit lamp exam does not demonstrate the presence of a rust ring.
Contact lens use is the most common etiology of a corneal ulcer and should be asked about when taking a history in a patient with eye pain. A contact lens, especially one left in overnight, prevents tears from lubricating the cornea and deprives the cornea of necessary oxygen. This results in hypoxia and hypercapnia of the epithelium and ultimately localized necrosis where bacteria can take hold (keratitis). Other risk factors contributing to ulcer development include diabetes, previous ocular surgery, chronic eye diseases, corticosteroids use, contaminated ocular medications, and farm/field work, and zoster. Infectious etiologies include Staphylococcus aureus/epidermidis/fusarium, Pseudomonas, HSV, CMV, and less commonly fungal, protozoan, and autoimmune disorders. Examination usually reveals a patient with some degree of pain, photophobia, and conjunctival injection. Occasionally, the cloudy infiltrate of the cornea can be seen with the naked eye or ophthalmoscope but is best seen with a slit lamp. HSV infections have the classic “dendritic lesions” while fungal infections have a gray-white “feathered” appearance with irregular, as opposed to the typical well-defined, margins.
Treatment should be coordinated with an ophthalmologist and typically involves starting a topical fluoroquinolone such as ciprofloxacin or the previously mentioned moxifloxacin, especially if contact lens use has occurred. A topical corticosteroid may also be recommended by ophthalmology. If the ulcer appears dendritic, topical antivirals such as trifluridine are recommended. The patient should be counseled on the importance of ophthalmology follow-up within no more than 24 hours and this follow-up should be coordinated by the provider. Permanent damage and vision loss is possible if not treated appropriately and with close follow-up to ensure improvement on the treatment that was started.
References:
1. https://www.ncbi.nlm.nih.gov/pmc/articles/PMC4170392/
2. https://www.ncbi.nlm.nih.gov/books/NBK539689/
3. UpToDateIncorrect
The correct answer is B. This image is consistent with a corneal ulcer which should be aggressively managed using a topical antibiotic which covers Pseudomonas. Erythromycin does not adequately cover Pseudomonas. Naphazoline-pheniramine is used for symptoms associated with allergic conjunctivitis, which is typically bilateral, itchy, and does not cause photophobia or staining on exam. Proparacaine is a topical anesthetic generally used in the evaluation of eye pain to help facilitate the exam but should not be used indefinitely in order to avoid secondary corneal epithelial defects or even perforation. The slit lamp exam does not demonstrate the presence of a rust ring.
Contact lens use is the most common etiology of a corneal ulcer and should be asked about when taking a history in a patient with eye pain. A contact lens, especially one left in overnight, prevents tears from lubricating the cornea and deprives the cornea of necessary oxygen. This results in hypoxia and hypercapnia of the epithelium and ultimately localized necrosis where bacteria can take hold (keratitis). Other risk factors contributing to ulcer development include diabetes, previous ocular surgery, chronic eye diseases, corticosteroids use, contaminated ocular medications, and farm/field work, and zoster. Infectious etiologies include Staphylococcus aureus/epidermidis/fusarium, Pseudomonas, HSV, CMV, and less commonly fungal, protozoan, and autoimmune disorders. Examination usually reveals a patient with some degree of pain, photophobia, and conjunctival injection. Occasionally, the cloudy infiltrate of the cornea can be seen with the naked eye or ophthalmoscope but is best seen with a slit lamp. HSV infections have the classic “dendritic lesions” while fungal infections have a gray-white “feathered” appearance with irregular, as opposed to the typical well-defined, margins.
Treatment should be coordinated with an ophthalmologist and typically involves starting a topical fluoroquinolone such as ciprofloxacin or the previously mentioned moxifloxacin, especially if contact lens use has occurred. A topical corticosteroid may also be recommended by ophthalmology. If the ulcer appears dendritic, topical antivirals such as trifluridine are recommended. The patient should be counseled on the importance of ophthalmology follow-up within no more than 24 hours and this follow-up should be coordinated by the provider. Permanent damage and vision loss is possible if not treated appropriately and with close follow-up to ensure improvement on the treatment that was started.
References:
1. https://www.ncbi.nlm.nih.gov/pmc/articles/PMC4170392/
2. https://www.ncbi.nlm.nih.gov/books/NBK539689/
3. UpToDate -
Question 5 of 56
5. Question
The following image will most likely be associated with which other finding?Correct
The correct answer is E. The image is a classic corneal dendritic ulcer known as herpes zoster ophthalmicus as seen after application of fluorescein under slit lamp. Shingles involving the ophthalmic division of the 5th cranial nerve should prompt the provider to rule-out ophthalmic involvement. Cell and flare is associated with uveitis and no stain uptake would occur. HSV of the cornea usually does not result in significant purulent discharge. Nystagmus and strabismus are not associated with corneal irregularities.
Patients with herpes zoster ophthalmicus may report unilateral headache, facial pain, or hypesthesia prior to the onset of a unilateral erythematous, turning vesicular, facial rash followed by a hyperemic ipsilateral and painful eye. Classically, if the rash reaches the tip of the nose (Hutchinson sign), corneal involvement in eminent. Ptosis is rare. In addition to staining and a slit lamp exam, tonometry should be considered as there is some risk of associated increased intraocular pressure. Risk factors include over 50-years-of-age, immunocompromised state, poorly controlled chronic diseases, acute illness, trauma, and emotional stressors.
The mainstay of treatment is via oral antivirals such as acyclovir, valacyclovir, or famciclovir for a minimum of 7 days. Topical antivirals may also be used but there is limited evidence of their effectiveness. Rather, topical antibiotics may be more useful in preventing a secondary bacterial infection. Topical corticosteroids may be used in consultation with an ophthalmologist. Other options for patient comfort include oral analgesics, artificial tears, and cool compresses. The only preventative measure is a zoster vaccination after 50-years-of-age which is approximately 97% effective up to age 70.
References:
A. https://www.ncbi.nlm.nih.gov/books/NBK557779/
B. UpToDateIncorrect
The correct answer is E. The image is a classic corneal dendritic ulcer known as herpes zoster ophthalmicus as seen after application of fluorescein under slit lamp. Shingles involving the ophthalmic division of the 5th cranial nerve should prompt the provider to rule-out ophthalmic involvement. Cell and flare is associated with uveitis and no stain uptake would occur. HSV of the cornea usually does not result in significant purulent discharge. Nystagmus and strabismus are not associated with corneal irregularities.
Patients with herpes zoster ophthalmicus may report unilateral headache, facial pain, or hypesthesia prior to the onset of a unilateral erythematous, turning vesicular, facial rash followed by a hyperemic ipsilateral and painful eye. Classically, if the rash reaches the tip of the nose (Hutchinson sign), corneal involvement in eminent. Ptosis is rare. In addition to staining and a slit lamp exam, tonometry should be considered as there is some risk of associated increased intraocular pressure. Risk factors include over 50-years-of-age, immunocompromised state, poorly controlled chronic diseases, acute illness, trauma, and emotional stressors.
The mainstay of treatment is via oral antivirals such as acyclovir, valacyclovir, or famciclovir for a minimum of 7 days. Topical antivirals may also be used but there is limited evidence of their effectiveness. Rather, topical antibiotics may be more useful in preventing a secondary bacterial infection. Topical corticosteroids may be used in consultation with an ophthalmologist. Other options for patient comfort include oral analgesics, artificial tears, and cool compresses. The only preventative measure is a zoster vaccination after 50-years-of-age which is approximately 97% effective up to age 70.
References:
A. https://www.ncbi.nlm.nih.gov/books/NBK557779/
B. UpToDate -
Question 6 of 56
6. Question
A patient reports waking in the middle of the night with severe bilateral eye pain. He states he works in an automotive repair shop but denies any known foreign body exposure or sensation. Denies contact lens use or prior similar symptoms. On exam, the patient appears to be in significant discomfort and has difficulty opening his eyes due to photophobia. Tearing is present but no discharge. Both eyes are injected without visible foreign body of the eye or lids. You decide to use fluorescein stain and under the Wood’s lamp there are numerous bilateral corneal areas of punctate staining. What is the most likely etiology of this patient’s symptoms
Correct
The correct answer is E. Given the patient’s occupation, onset of symptoms without any known exposure or injury, and bilateral punctate stain uptake, the patient was most likely incidentally exposed to welding arch. Bilateral acute glaucoma would be highly unlikely. A caustic substance exposure would cause immediate symptoms. Contact lens use generally results in a single corneal ulcer. Herpes simplex virus (HSV) results in a dendritic stain uptake and bilateral involvement would be highly unlikely.
Ultraviolet (UV) keratitis, also called photokeratitis, is caused by unprotected exposure to things like welder’s arc, reflections from snow, sunlamps, disinfecting lamps, or high-voltage power line arc. There is usually a delay of up to 12 hours from the time of exposure to onset of symptoms making a thorough history and differential diagnosis the key to making the diagnosis. The UV light destroys the corneal epithelial cells allowing uninterrupted stimulation of the subepithelial cells which generates the pain. These cells usually regenerate within 24-72 hours, at which point the symptoms resolve. The pain can be severe during this time period which is usually what prompts the patient to seek care. Tearing and photophobia accompany the pain and there may also be visible injection and chemosis of the conjunctiva as well as a slightly cloudy cornea. Depending on the duration and intensity of the UV exposure, there may be erythema of the face or lids. Fluorescein exam reveals the classically described “punctate staining”, usually to the areas that were not covered by the natural positioning of the eyelids. In most cases, the finding will be bilateral unless the patient was intentionally closing one eye or the source of the light was significantly off to the side of the patient.
Treatment is generally supportive with oral analgesics and topical antibiotic ophthalmic ointments which provide lubrication and prevent secondary infection. Pain can also be controlled using a drop of a cycloplegic such as cyclopentolate 1% (Cyclogyl) or homatropine 2 to 5% (Isopto homatropine). The patient should be notified that this will result in pupil dilation for several days. Reexamination in 48 hours to ensure improvement is recommended. Prevention using proper eye protection based on the anticipated exposure is the most important aspect of patient education.
References:
Incorrect
The correct answer is E. Given the patient’s occupation, onset of symptoms without any known exposure or injury, and bilateral punctate stain uptake, the patient was most likely incidentally exposed to welding arch. Bilateral acute glaucoma would be highly unlikely. A caustic substance exposure would cause immediate symptoms. Contact lens use generally results in a single corneal ulcer. Herpes simplex virus (HSV) results in a dendritic stain uptake and bilateral involvement would be highly unlikely.
Ultraviolet (UV) keratitis, also called photokeratitis, is caused by unprotected exposure to things like welder’s arc, reflections from snow, sunlamps, disinfecting lamps, or high-voltage power line arc. There is usually a delay of up to 12 hours from the time of exposure to onset of symptoms making a thorough history and differential diagnosis the key to making the diagnosis. The UV light destroys the corneal epithelial cells allowing uninterrupted stimulation of the subepithelial cells which generates the pain. These cells usually regenerate within 24-72 hours, at which point the symptoms resolve. The pain can be severe during this time period which is usually what prompts the patient to seek care. Tearing and photophobia accompany the pain and there may also be visible injection and chemosis of the conjunctiva as well as a slightly cloudy cornea. Depending on the duration and intensity of the UV exposure, there may be erythema of the face or lids. Fluorescein exam reveals the classically described “punctate staining”, usually to the areas that were not covered by the natural positioning of the eyelids. In most cases, the finding will be bilateral unless the patient was intentionally closing one eye or the source of the light was significantly off to the side of the patient.
Treatment is generally supportive with oral analgesics and topical antibiotic ophthalmic ointments which provide lubrication and prevent secondary infection. Pain can also be controlled using a drop of a cycloplegic such as cyclopentolate 1% (Cyclogyl) or homatropine 2 to 5% (Isopto homatropine). The patient should be notified that this will result in pupil dilation for several days. Reexamination in 48 hours to ensure improvement is recommended. Prevention using proper eye protection based on the anticipated exposure is the most important aspect of patient education.
References:
-
Question 7 of 56
7. Question
Which treatment option is the most effective for the findings in the attached image?Correct
The correct answer is D. The finding is consistent with a pterygium which usually requires excision once it impacts vision or ocular movement. Mast cell stabilizers and ocular decongestants are recommended as first-line options for allergic conjunctivitis. Cool compresses would provide no improvement for this disorder.
A pterygium is a fibrovascular growth of conjunctival tissue which usually has a triangular shape. The growth usually starts the nasal side of the conjunctiva and extends towards and possibly over the cornea. Most pterygiums are the result of environmental factors such as excess UV light, dust, smoke or excessively hot, dry, or windy environments. Other factors include abnormalities of tumor suppressor gene p53 or human leukocyte antigen (HLA) expression, and HPV infection. The disorder is not painful and the patient may present for concerns regarding the growth or if vision is being impacted.
A small pterygium can be treated with artificial tears and monitoring. Other options include topical ocular decongestants, ocular NSAIDs, and steroids however, they all have adverse effects which limit any long-term use. Some studies have shown that intralesional injections of vascular endothelial growth factor (VEGF) inhibitors may be beneficial. Surgical excision is usually indicated once the growth impacts vision. Despite excision, a high reoccurrence rate occurs. Potential complications of surgery include globe perforation, muscle damage, granuloma formation, retinal detachment, endophthalmitis, and corneal ulceration. Repeated excisions can result in permanent corneal scarring, astigmatism, impaired extraocular movements, or even the scarring of the eyelid to the globe (symblepharon).
References:
1. UpToDate
https://www.ncbi.nlm.nih.gov/pmc/articles/PMC5340105Incorrect
The correct answer is D. The finding is consistent with a pterygium which usually requires excision once it impacts vision or ocular movement. Mast cell stabilizers and ocular decongestants are recommended as first-line options for allergic conjunctivitis. Cool compresses would provide no improvement for this disorder.
A pterygium is a fibrovascular growth of conjunctival tissue which usually has a triangular shape. The growth usually starts the nasal side of the conjunctiva and extends towards and possibly over the cornea. Most pterygiums are the result of environmental factors such as excess UV light, dust, smoke or excessively hot, dry, or windy environments. Other factors include abnormalities of tumor suppressor gene p53 or human leukocyte antigen (HLA) expression, and HPV infection. The disorder is not painful and the patient may present for concerns regarding the growth or if vision is being impacted.
A small pterygium can be treated with artificial tears and monitoring. Other options include topical ocular decongestants, ocular NSAIDs, and steroids however, they all have adverse effects which limit any long-term use. Some studies have shown that intralesional injections of vascular endothelial growth factor (VEGF) inhibitors may be beneficial. Surgical excision is usually indicated once the growth impacts vision. Despite excision, a high reoccurrence rate occurs. Potential complications of surgery include globe perforation, muscle damage, granuloma formation, retinal detachment, endophthalmitis, and corneal ulceration. Repeated excisions can result in permanent corneal scarring, astigmatism, impaired extraocular movements, or even the scarring of the eyelid to the globe (symblepharon).
References:
1. UpToDate
https://www.ncbi.nlm.nih.gov/pmc/articles/PMC5340105 -
Question 8 of 56
8. Question
Which of the following is considered first-line treatment of chronic dacryocystitis?
Correct
The correct answer is D. Dacryocystitis is an inflammation of the nasolacrimal apparatus. Chronic recurrent cases are usually initially managed with probing +/- dilation of the lacrimal duct but will likely require additional surgical intervention. Causes of acute dacryocystitis include a dacryolith and surrounding cellulitis. Chronic cases are caused by scaring from prior infections/surgery/trauma, systemic disorders such as Wegener’s granulomatosis, sarcoidosis, and lupus, or local tumors. There are also cases of congenitally acquired cases which are due to incomplete expulsion of amniotic fluid from the duct at the time of delivery which then becomes purulent. The majority of cases occur at the extremes of age.
Physical exam findings may be subtle if presenting early and not completely obstructed but there is generally some degree of swelling along the crease between the nose and the cheek which is tender to palpation and may also be erythematous (not to include the entire periorbital area). Pus may also be expressed through the puncta with gentle massage. Most diagnoses are made clinically. Atypical presentations, high fever, headaches, ocular muscle entrapment or anything else of concern should prompt ordering labs and a facial CT scan.
Treatment is targeted at the underlying cause. Evidence of infection warrants antibiotic coverage directed primarily to cover Staphylococcus and Streptococcus. Haemophilus and Pseudomonas are also possible. Cases of chronic dacryocystitis may be treated initially with probing of the canal using a thin metal wire to assist with passage of the stone and open the canal. Additional surgical intervention will likely be necessary. Options include balloon dilation, stenting, or the creation of an alternative pathway. Neonatal or congenital cases will usually resolve within the first year of life using conservative measures such as massage of the canal by the parents in a downward direction and topical antibiotics for acute flares.
References:
A. https://www.ncbi.nlm.nih.gov/books/NBK470565/
B. https://www.ccjm.org/content/87/8/477
C. UpToDateIncorrect
The correct answer is D. Dacryocystitis is an inflammation of the nasolacrimal apparatus. Chronic recurrent cases are usually initially managed with probing +/- dilation of the lacrimal duct but will likely require additional surgical intervention. Causes of acute dacryocystitis include a dacryolith and surrounding cellulitis. Chronic cases are caused by scaring from prior infections/surgery/trauma, systemic disorders such as Wegener’s granulomatosis, sarcoidosis, and lupus, or local tumors. There are also cases of congenitally acquired cases which are due to incomplete expulsion of amniotic fluid from the duct at the time of delivery which then becomes purulent. The majority of cases occur at the extremes of age.
Physical exam findings may be subtle if presenting early and not completely obstructed but there is generally some degree of swelling along the crease between the nose and the cheek which is tender to palpation and may also be erythematous (not to include the entire periorbital area). Pus may also be expressed through the puncta with gentle massage. Most diagnoses are made clinically. Atypical presentations, high fever, headaches, ocular muscle entrapment or anything else of concern should prompt ordering labs and a facial CT scan.
Treatment is targeted at the underlying cause. Evidence of infection warrants antibiotic coverage directed primarily to cover Staphylococcus and Streptococcus. Haemophilus and Pseudomonas are also possible. Cases of chronic dacryocystitis may be treated initially with probing of the canal using a thin metal wire to assist with passage of the stone and open the canal. Additional surgical intervention will likely be necessary. Options include balloon dilation, stenting, or the creation of an alternative pathway. Neonatal or congenital cases will usually resolve within the first year of life using conservative measures such as massage of the canal by the parents in a downward direction and topical antibiotics for acute flares.
References:
A. https://www.ncbi.nlm.nih.gov/books/NBK470565/
B. https://www.ccjm.org/content/87/8/477
C. UpToDate -
Question 9 of 56
9. Question
A patient presents with the findings in the attached image and reports facial flushing during the review of systems questioning. What is the most likely etiology?Correct
The correct answer is D. The image is consistent with a diagnosis of blepharitis. Facial flushing is usually associated with rosacea, which is a common etiology of blepharitis. When the etiology is atopic dermatitis, a patient may report rashes in the creases of their skin or a family history of asthma. Parasitic (mite) infestation classically manifests with cylindrical scales that form around lash bases known as “collarettes”. A psoriasis etiology is usually associated with scaley erythematous plaques of the knees and elbows. Seborrheic dermatitis is a common cause of blepharitis, in which case, the patient may report having scalp dandruff.
Blepharitis is an inflammatory disorder involving the eyelid which is subdivided into anterior and posterior types. Posterior blepharitis effects the inner portion of the lid and is the more common of the two. Anterior effects the base of the lashes. Rosacea is a common cause of posterior-type while staphylococcal infections are more common causes of anterior-type. Seborrheic dermatitis, infestation, contact dermatitis, psoriasis, and eczema can cause either type. The patient may report itchy and/or flaking of the eyelids, a gritty irritation of the affected eye, or matting of the lashes in the mornings. Exam findings include crusting or flaking of the lid and lashes, conjunctival injection, and excess tearing. The diagnosis is made clinically and culture can be considered for severe cases or those not responding to treatment.
Treatment should target the suspected underlying cause. For systemic disorders such as psoriasis and rosacea, address per guidelines. Bacterial causes should respond to ophthalmic antibiotic ointments applied to the lids and the eye itself if secondary conjunctivitis is present, as opposed to drops. Mite infestations can be treated with oral ivermectin or topical tea tree solutions. All causes should benefit from warm compresses and lid massage which encourage emptying the meibomian glands, and artificial tears to help minimize eye dryness. For unresponsive cases, oral antibiotics, topical steroids, and topical cyclosporine may be considered. Patients with systemic disorders will benefit from regular lid hygiene which includes massage, warm compresses, and artificial tears.
References:
1. UpToDate
2. https://www.merckmanuals.com/professional/eye-disorders/eyelid-and-lacrimal-disorders/blepharitis#v954105Incorrect
The correct answer is D. The image is consistent with a diagnosis of blepharitis. Facial flushing is usually associated with rosacea, which is a common etiology of blepharitis. When the etiology is atopic dermatitis, a patient may report rashes in the creases of their skin or a family history of asthma. Parasitic (mite) infestation classically manifests with cylindrical scales that form around lash bases known as “collarettes”. A psoriasis etiology is usually associated with scaley erythematous plaques of the knees and elbows. Seborrheic dermatitis is a common cause of blepharitis, in which case, the patient may report having scalp dandruff.
Blepharitis is an inflammatory disorder involving the eyelid which is subdivided into anterior and posterior types. Posterior blepharitis effects the inner portion of the lid and is the more common of the two. Anterior effects the base of the lashes. Rosacea is a common cause of posterior-type while staphylococcal infections are more common causes of anterior-type. Seborrheic dermatitis, infestation, contact dermatitis, psoriasis, and eczema can cause either type. The patient may report itchy and/or flaking of the eyelids, a gritty irritation of the affected eye, or matting of the lashes in the mornings. Exam findings include crusting or flaking of the lid and lashes, conjunctival injection, and excess tearing. The diagnosis is made clinically and culture can be considered for severe cases or those not responding to treatment.
Treatment should target the suspected underlying cause. For systemic disorders such as psoriasis and rosacea, address per guidelines. Bacterial causes should respond to ophthalmic antibiotic ointments applied to the lids and the eye itself if secondary conjunctivitis is present, as opposed to drops. Mite infestations can be treated with oral ivermectin or topical tea tree solutions. All causes should benefit from warm compresses and lid massage which encourage emptying the meibomian glands, and artificial tears to help minimize eye dryness. For unresponsive cases, oral antibiotics, topical steroids, and topical cyclosporine may be considered. Patients with systemic disorders will benefit from regular lid hygiene which includes massage, warm compresses, and artificial tears.
References:
1. UpToDate
2. https://www.merckmanuals.com/professional/eye-disorders/eyelid-and-lacrimal-disorders/blepharitis#v954105 -
Question 10 of 56
10. Question
The accompanying image is most consistent with which diagnosis?Correct
The correct answer is B. The difference in presentation between a chalazion and a hordeolum can be subtle but a chalazion is a swollen area of the body of the lid while a hordeolum is a swelling of the eyelid margin at the base of an eyelash. Blepharitis is a diffuse flaking and crusting of the lid margin. Dacryocystitis is swelling and pain of the nasolacrimal duct apparatus. Periorbital, also called preseptal, cellulitis manifests as diffuse, opposed to localized, swelling and erythema of the eyelid extending up to the area below the eyebrow. Xanthelasma are soft, yellowish plaques of the eyelids, usually bilateral and relatively symmetrical. They are caused by cholesterol deposition.
The terms chalazion and hordeolum are frequently used interchangeably by providers despite the fact that they are associated with different origins and presentations. A chalazion is the result of an obstruction of a meibomian gland within the lid body which results in extravasation of lipid material into the surrounding soft tissues resulting in subsequent granulomatous inflammation presenting as a small nontender nodule which drains through the inner surface of the eyelid. Technically, this is not an infectious process. A hordeolum, commonly referred to as a “stye”, is an infection, typically staphylococcal, at the entry point of the lash into the lid margin. In this case, Zeis or Moll glands are involved instead of the meibomian. This process results in a pustule and surrounding erythema where the lash meets the lid. After a few days, this pustule spontaneously ruptures, allowing the pus to drain from the lid margin, as opposed to the internal surface.
Both chalazion and hordeolum usually respond well to supportive care which is primarily hot compresses several times per day. Persistent chalazion’s can be managed by an ophthalmologist using incision and curettage or intralesional glucocorticoid injection. In the case of a hordeolum, also advise the patient to avoid eye makeup, which could occlude the Zeis or Moll glands further and prevent drainage. Antibiotic ophthalmic ointment may be indicated if signs of conjunctivitis are present. If symptoms do not improve in 1-2 weeks despite using hot compresses regularly, a referral to an ophthalmologist for possible incision and drainage is warranted.
References:
1. https://www.merckmanuals.com/professional/eye-disorders/eyelid-and-lacrimal-disorders/chalazion-and-hordeolum-stye#:~:text=Chalazia%20and%20hordeola%20(styes)%20are,edema%2C%20swelling%2C%20and%20pain.
2. UpToDateIncorrect
The correct answer is B. The difference in presentation between a chalazion and a hordeolum can be subtle but a chalazion is a swollen area of the body of the lid while a hordeolum is a swelling of the eyelid margin at the base of an eyelash. Blepharitis is a diffuse flaking and crusting of the lid margin. Dacryocystitis is swelling and pain of the nasolacrimal duct apparatus. Periorbital, also called preseptal, cellulitis manifests as diffuse, opposed to localized, swelling and erythema of the eyelid extending up to the area below the eyebrow. Xanthelasma are soft, yellowish plaques of the eyelids, usually bilateral and relatively symmetrical. They are caused by cholesterol deposition.
The terms chalazion and hordeolum are frequently used interchangeably by providers despite the fact that they are associated with different origins and presentations. A chalazion is the result of an obstruction of a meibomian gland within the lid body which results in extravasation of lipid material into the surrounding soft tissues resulting in subsequent granulomatous inflammation presenting as a small nontender nodule which drains through the inner surface of the eyelid. Technically, this is not an infectious process. A hordeolum, commonly referred to as a “stye”, is an infection, typically staphylococcal, at the entry point of the lash into the lid margin. In this case, Zeis or Moll glands are involved instead of the meibomian. This process results in a pustule and surrounding erythema where the lash meets the lid. After a few days, this pustule spontaneously ruptures, allowing the pus to drain from the lid margin, as opposed to the internal surface.
Both chalazion and hordeolum usually respond well to supportive care which is primarily hot compresses several times per day. Persistent chalazion’s can be managed by an ophthalmologist using incision and curettage or intralesional glucocorticoid injection. In the case of a hordeolum, also advise the patient to avoid eye makeup, which could occlude the Zeis or Moll glands further and prevent drainage. Antibiotic ophthalmic ointment may be indicated if signs of conjunctivitis are present. If symptoms do not improve in 1-2 weeks despite using hot compresses regularly, a referral to an ophthalmologist for possible incision and drainage is warranted.
References:
1. https://www.merckmanuals.com/professional/eye-disorders/eyelid-and-lacrimal-disorders/chalazion-and-hordeolum-stye#:~:text=Chalazia%20and%20hordeola%20(styes)%20are,edema%2C%20swelling%2C%20and%20pain.
2. UpToDate -
Question 11 of 56
11. Question
Which of the following is the most appropriate term for the finding of an inward lower eyelid on physical exam?
Correct
The correct answer is C. The term ectropion is used to describe an outwardly sagging eyelid. Blepharitis is used with crusting, scaling, or infection of the lid along the insertion of the lashes is present. A hordeolum, also referred to as a stye, is an isolated swelling of the eyelid margin resulting from an occluded gland. Ptosis is a dropping or lagging of an upper eyelid which can be related to an issue involving the third crania nerve.
Entropion and ectropion most commonly involve the lower eyelid, although an entropion can occur to the upper lid as well. An entropion is usually the result of a muscle spasm but can also be caused by scarring or inflammation while an ectropion is the result of diminished muscle tone and generally occurs after the age of 60. Both can present with similar complaints from the patient. These include a gritty or sandy sensation, excess tearing, burning, or redness of the conjunctiva. The inward-turned entropion causes this because of lashes lying up against the conjunctiva. The outward sagging of the ectropion results in a loss of the normal tears which can dry the eye and also reflexively cause the patient to frequently wipe the eye and face with their hand or other object and further irritate the surface of the eye or cause the sagging to worsen resulting in further tear loss.
Either disorder can be temporarily improved by administering artificial tears regularly. Another temporary solution for a lower lid entropion is to have the patient tape the lower eyelid down, however, this can be annoying to the patient, is not cosmetically appealing, and can result in irritation of the skin due to the tape. The most effective treatment is surgical correction by an oculoplastic specialist.
References:
A. https://www.eye7.in/oculoplasty/ptosis-ectropion-and-entropion/
B. https://www.nhs.uk/conditions/ectropioIncorrect
The correct answer is C. The term ectropion is used to describe an outwardly sagging eyelid. Blepharitis is used with crusting, scaling, or infection of the lid along the insertion of the lashes is present. A hordeolum, also referred to as a stye, is an isolated swelling of the eyelid margin resulting from an occluded gland. Ptosis is a dropping or lagging of an upper eyelid which can be related to an issue involving the third crania nerve.
Entropion and ectropion most commonly involve the lower eyelid, although an entropion can occur to the upper lid as well. An entropion is usually the result of a muscle spasm but can also be caused by scarring or inflammation while an ectropion is the result of diminished muscle tone and generally occurs after the age of 60. Both can present with similar complaints from the patient. These include a gritty or sandy sensation, excess tearing, burning, or redness of the conjunctiva. The inward-turned entropion causes this because of lashes lying up against the conjunctiva. The outward sagging of the ectropion results in a loss of the normal tears which can dry the eye and also reflexively cause the patient to frequently wipe the eye and face with their hand or other object and further irritate the surface of the eye or cause the sagging to worsen resulting in further tear loss.
Either disorder can be temporarily improved by administering artificial tears regularly. Another temporary solution for a lower lid entropion is to have the patient tape the lower eyelid down, however, this can be annoying to the patient, is not cosmetically appealing, and can result in irritation of the skin due to the tape. The most effective treatment is surgical correction by an oculoplastic specialist.
References:
A. https://www.eye7.in/oculoplasty/ptosis-ectropion-and-entropion/
B. https://www.nhs.uk/conditions/ectropio -
Question 12 of 56
12. Question
A patient presents with dizziness and nausea. On exam, which type of nystagmus would only occur if the symptoms are purely the result of a central, as opposed to a peripheral, cause?
Correct
The correct answer is D. Nystagmus is an involuntary oscillatory movement of the eyes which is rhythmic in nature and can be fast, slow, or have a faster movement in one direction than another. It can be constant, intermittent, or triggered by movements of the eyes or head. The movements can impact coordination, including balance, as well as depth perception and vision. The etiology of nystagmus is either the peripheral (vestibular structures within the ear) or central (vestibular nuclear complex and cerebellum within the brain). There are primarily two descriptors of nystagmus, jerk and pendular, in addition to the direction. Jerk nystagmus describes a pattern of a slow drift in one direction followed by a rapid “jerk” in the opposite. Pendular is used to describe a pattern of slow, consistent movements that appear similar to a pendulum freely swinging back and forth. Jerk nystagmus can be caused by either peripheral or central etiologies (see below) while pendular may be the result of a brain matter disorder such as multiple sclerosis or congenital problems.
Peripheral vertigo is the result of inflammation, excess endolymphatic fluid or an autoimmune process effecting the otolithic organs or semicircular canals. Common disorders associated with peripheral vertigo include benign paroxysmal positional vertigo, Meniere disease, vestibular neuritis, labyrinthitis, acoustic neuroma, and viral or bacterial infection of the inner ear.
Central vertigo is the result of a disruption in the cerebellar blood flow which causes an asymmetric disturbance in the network which stabilizes vertical gaze. The network includes the cerebello-vestibular tract, vestibulo-ocular reflexes, and the vertical smooth pursuit system. Common disorders associated with central vertigo include brainstem stroke, vertebrobasilar insufficiency, masses, demyelination syndromes, migraine, and Chiari malformation.
The work-up and treatment of nystagmus depend on the most likely underlying cause which makes the history and physical exam the key to narrowing down the possibilities. A MRI may be the most useful but will not rule out many of the diagnoses.
References:
A. UpToDate
B. https://www.ncbi.nlm.nih.gov/books/NBK539711/
C. https://academic.oup.com/brain/article/128/6/1237/432048
D. https://www.ncbi.nlm.nih.gov/books/NBK430797Incorrect
The correct answer is D. Nystagmus is an involuntary oscillatory movement of the eyes which is rhythmic in nature and can be fast, slow, or have a faster movement in one direction than another. It can be constant, intermittent, or triggered by movements of the eyes or head. The movements can impact coordination, including balance, as well as depth perception and vision. The etiology of nystagmus is either the peripheral (vestibular structures within the ear) or central (vestibular nuclear complex and cerebellum within the brain). There are primarily two descriptors of nystagmus, jerk and pendular, in addition to the direction. Jerk nystagmus describes a pattern of a slow drift in one direction followed by a rapid “jerk” in the opposite. Pendular is used to describe a pattern of slow, consistent movements that appear similar to a pendulum freely swinging back and forth. Jerk nystagmus can be caused by either peripheral or central etiologies (see below) while pendular may be the result of a brain matter disorder such as multiple sclerosis or congenital problems.
Peripheral vertigo is the result of inflammation, excess endolymphatic fluid or an autoimmune process effecting the otolithic organs or semicircular canals. Common disorders associated with peripheral vertigo include benign paroxysmal positional vertigo, Meniere disease, vestibular neuritis, labyrinthitis, acoustic neuroma, and viral or bacterial infection of the inner ear.
Central vertigo is the result of a disruption in the cerebellar blood flow which causes an asymmetric disturbance in the network which stabilizes vertical gaze. The network includes the cerebello-vestibular tract, vestibulo-ocular reflexes, and the vertical smooth pursuit system. Common disorders associated with central vertigo include brainstem stroke, vertebrobasilar insufficiency, masses, demyelination syndromes, migraine, and Chiari malformation.
The work-up and treatment of nystagmus depend on the most likely underlying cause which makes the history and physical exam the key to narrowing down the possibilities. A MRI may be the most useful but will not rule out many of the diagnoses.
References:
A. UpToDate
B. https://www.ncbi.nlm.nih.gov/books/NBK539711/
C. https://academic.oup.com/brain/article/128/6/1237/432048
D. https://www.ncbi.nlm.nih.gov/books/NBK430797 -
Question 13 of 56
13. Question
A 25-year-old female reports progressive vision loss over the past 3 days associated with moderate eye discomfort. She notices that it is most profound in the central portion of the visual field. Review of systems negative for headache or neck pain and positive for generalized intermittent paresthesia’s. Which test is most likely to confirm your top differential diagnosis?
Correct
The correct answer is D. The case scenario most likely represents a new diagnosis of multiple sclerosis (MS) which is manifesting as optic neuritis. A MRI would be most likely to demonstrate both the optic neuritis as well as any associated brain plaques. A CTA would be used in the workup of an aneurysm or ischemic stroke, neither of which should cause generalized paresthesia’s or central vision loss. An electroencephalogram is used in the work-up of a suspected seizure which is inconsistent with the history. Tonometry is used to assess intraocular pressure and diagnose glaucoma which does not fit with this scenario.
Optic neuritis is an inflammatory and demyelinating process of the optic nerve. It is the symptom that prompts presentation and subsequent diagnosis in 15-20% of patients who have underlying MS and occurs at some point in approximately 50% of MS patients’ lifetime. The majority of cases occur in females age 20-40. Incidents also seem higher the further away from the equator. Other etiologies of optic neuritis include syphilis, Lyme, cat-scratch disease, sarcoidosis and paraneoplastic syndrome. Approximately 90% of optic neuritis cases involve central visual field loss in only 1 eye and are painful (periorbital or retro-ocular) to some degree. The vision loss occurs over hours to days and peaks by 2 weeks. Other symptoms a patient may report include decreased color vision and flashing or flickering lights. Physical exam findings include papilledema, distended optic veins, or a positive swinging flashlight sign demonstrating an afferent pupillary defect.
The diagnosis in most cases can be made based on history and physical. Confirmation of the diagnosis can be made with a MRI of the brain and orbits. If the diagnosis is uncertain, additional test that should be considered include erythrocyte sedimentation rate (ESR), antinuclear antibodies (ANA), and angiotensin converting enzyme (ACE) levels and tests for Lyme disease and syphilis. A lumbar puncture with cerebrospinal fluid (CSF) analysis will show elevated lymphocytes and protein in the majority of cases. The primary treatment option is early high dose intravenous corticosteroid with transition to oral. The suggested regiment consists of IV methylprednisolone 250 mg four times per day for three days followed by oral prednisone at 1 mg/kg/day) for 7 days followed by a four-day taper. The prognosis for vision recovery is very good, however, approximately 35% will have another event within 10 years. The treatment also prolongs onset of MS by a few years.
References:
A. UpToDate
B. https://bestpractice.bmj.com/topics/en-us/966
C. https://www.ncbi.nlm.nih.gov/pmc/articles/PMC6370553/Incorrect
The correct answer is D. The case scenario most likely represents a new diagnosis of multiple sclerosis (MS) which is manifesting as optic neuritis. A MRI would be most likely to demonstrate both the optic neuritis as well as any associated brain plaques. A CTA would be used in the workup of an aneurysm or ischemic stroke, neither of which should cause generalized paresthesia’s or central vision loss. An electroencephalogram is used in the work-up of a suspected seizure which is inconsistent with the history. Tonometry is used to assess intraocular pressure and diagnose glaucoma which does not fit with this scenario.
Optic neuritis is an inflammatory and demyelinating process of the optic nerve. It is the symptom that prompts presentation and subsequent diagnosis in 15-20% of patients who have underlying MS and occurs at some point in approximately 50% of MS patients’ lifetime. The majority of cases occur in females age 20-40. Incidents also seem higher the further away from the equator. Other etiologies of optic neuritis include syphilis, Lyme, cat-scratch disease, sarcoidosis and paraneoplastic syndrome. Approximately 90% of optic neuritis cases involve central visual field loss in only 1 eye and are painful (periorbital or retro-ocular) to some degree. The vision loss occurs over hours to days and peaks by 2 weeks. Other symptoms a patient may report include decreased color vision and flashing or flickering lights. Physical exam findings include papilledema, distended optic veins, or a positive swinging flashlight sign demonstrating an afferent pupillary defect.
The diagnosis in most cases can be made based on history and physical. Confirmation of the diagnosis can be made with a MRI of the brain and orbits. If the diagnosis is uncertain, additional test that should be considered include erythrocyte sedimentation rate (ESR), antinuclear antibodies (ANA), and angiotensin converting enzyme (ACE) levels and tests for Lyme disease and syphilis. A lumbar puncture with cerebrospinal fluid (CSF) analysis will show elevated lymphocytes and protein in the majority of cases. The primary treatment option is early high dose intravenous corticosteroid with transition to oral. The suggested regiment consists of IV methylprednisolone 250 mg four times per day for three days followed by oral prednisone at 1 mg/kg/day) for 7 days followed by a four-day taper. The prognosis for vision recovery is very good, however, approximately 35% will have another event within 10 years. The treatment also prolongs onset of MS by a few years.
References:
A. UpToDate
B. https://bestpractice.bmj.com/topics/en-us/966
C. https://www.ncbi.nlm.nih.gov/pmc/articles/PMC6370553/ -
Question 14 of 56
14. Question
The attached image is most consistent with which disorder?Correct
The correct answer is C. The image demonstrates papilledema which is usually the manifestation of increased intracranial pressure. Bacterial endocarditis can cause an abnormal finding called a Roth spot. A Roth spot is a round or oval area with a pale, round center surrounded by the hemorrhage. Diabetes causes hard exudates, cotton-wool spots and hemorrhages. Hypertension results in arteriovenous nicking, copper or silver wiring, as well as hemorrhages and exudates similar to diabetic retinopathy. Retinal detachment causes a gray elevation in the temporal area with folds in the detached section while vessels appear tortuous and elevated over the detached retina.
Increased intracranial pressure disturbs flow within the optic nerve resulting in leakage of water and protein into the extracellular space of the optic disc known as papilledema. An absence of venous pulsations may occur early in the process along with splinter hemorrhages (also seen in the attached image) at or beyond the disc margin. If the pressure is not relieved, the edema continues to build causing the optic disc to become elevated as well as loss of the cup and disc margins. Venous dilation and telangiectasias occur next. Etiologies of elevated intracranial pressure include brain tumors, traumatic brain injury, increased CSF production or decreased absorption, obstructive hydrocephalus, venous sinus thrombosis, jugular vein obstruction, and pseudotumor cerebri.
Positional headache (worse in recumbent position) is the primary symptom of increased intracranial pressure along with nausea and vomiting. The headaches are also typically worse in the early morning. Double vision, visual field defects, and a pulsatile noise in the ear are also reported by patients. The initial work-up of suspected elevated intracranial pressure includes either CT or MRI of the brain. MRI is the preferred modality due to the level of detail it provides but if unavailable or if the patient is not stable enough to undergo the lengthier test, a CT is very reasonable. If the MRI or CT is unremarkable, a lumbar puncture should be considered in order to measure the opening pressure and send CSF for analysis. Ocular bedside ultrasound is also a modality that can be used to quickly determine if papilledema is present and if the patient is potentially suffering form an elevated intracranial pressure.
Treatment is targeted at the specific underlying etiology causing the increased pressure. The prognosis of vision recovery depends on how quickly the patient sought medical attention and the responsiveness to treatment. Patients presenting with high-grade disc edema, hemorrhages, visual field loss, older age, or a past medical history of anemia, glaucoma, extreme myopia, or hypertension do not fare as well.
References:
1. UpToDate
2. https://www.ncbi.nlm.nih.gov/pmc/articles/PMC5398730/Incorrect
The correct answer is C. The image demonstrates papilledema which is usually the manifestation of increased intracranial pressure. Bacterial endocarditis can cause an abnormal finding called a Roth spot. A Roth spot is a round or oval area with a pale, round center surrounded by the hemorrhage. Diabetes causes hard exudates, cotton-wool spots and hemorrhages. Hypertension results in arteriovenous nicking, copper or silver wiring, as well as hemorrhages and exudates similar to diabetic retinopathy. Retinal detachment causes a gray elevation in the temporal area with folds in the detached section while vessels appear tortuous and elevated over the detached retina.
Increased intracranial pressure disturbs flow within the optic nerve resulting in leakage of water and protein into the extracellular space of the optic disc known as papilledema. An absence of venous pulsations may occur early in the process along with splinter hemorrhages (also seen in the attached image) at or beyond the disc margin. If the pressure is not relieved, the edema continues to build causing the optic disc to become elevated as well as loss of the cup and disc margins. Venous dilation and telangiectasias occur next. Etiologies of elevated intracranial pressure include brain tumors, traumatic brain injury, increased CSF production or decreased absorption, obstructive hydrocephalus, venous sinus thrombosis, jugular vein obstruction, and pseudotumor cerebri.
Positional headache (worse in recumbent position) is the primary symptom of increased intracranial pressure along with nausea and vomiting. The headaches are also typically worse in the early morning. Double vision, visual field defects, and a pulsatile noise in the ear are also reported by patients. The initial work-up of suspected elevated intracranial pressure includes either CT or MRI of the brain. MRI is the preferred modality due to the level of detail it provides but if unavailable or if the patient is not stable enough to undergo the lengthier test, a CT is very reasonable. If the MRI or CT is unremarkable, a lumbar puncture should be considered in order to measure the opening pressure and send CSF for analysis. Ocular bedside ultrasound is also a modality that can be used to quickly determine if papilledema is present and if the patient is potentially suffering form an elevated intracranial pressure.
Treatment is targeted at the specific underlying etiology causing the increased pressure. The prognosis of vision recovery depends on how quickly the patient sought medical attention and the responsiveness to treatment. Patients presenting with high-grade disc edema, hemorrhages, visual field loss, older age, or a past medical history of anemia, glaucoma, extreme myopia, or hypertension do not fare as well.
References:
1. UpToDate
2. https://www.ncbi.nlm.nih.gov/pmc/articles/PMC5398730/ -
Question 15 of 56
15. Question
The symptoms of pain or limitation with moving an eye, proptosis, and double vision are most consistent with which diagnosis?
Correct
The correct answer is D. Orbital and preseptal (periorbital) cellulitis both have erythema and edema surrounding an eye and some degree of pain, however, preseptal is a superficial infection which does not involve the deeper structures resulting in the symptoms mentioned. Chemosis is a diffuse swelling of the conjunctiva that can accompany orbital cellulitis, but of itself does not involve pain, vision changes, or movement limitations. Glaucoma, especially acute angle closure-type, causes significant eye pain and diminished vision but usually not range of motion limitations or double vision. Iritis is a painful inflammatory condition of the iris usually caused by trauma, infection, or autoimmune etiologies. In addition to pain, blurred vision, photophobia, and redness occur but usually without ocular entrapment or proptosis.
Preseptal cellulitis is more common than orbital (post-septal), however, orbital is the more severe disorder which can lead to cavernous sinus thrombosis, intracranial abscess, or permanent vision loss. The pain from orbital cellulitis is a deeper pain and the patient generally appears more ill and uncomfortable. Orbital cellulitis occurs most commonly in children and is usually a complication of ethmoid rhinosinusitis due to staph or strep bacteria. Other causes include dental infections, orbital trauma, ophthalmic surgery, and dacryocystitis.
The provider should maintain a high level of suspicion for a diagnosis of orbital cellulitis in anyone who presents with swelling and erythema surrounding an eye, unexplained deep orbital pain, or restricted eye movements. The diagnosis can be confirmed with CT or MRI, with the former being adequate in most cases. Treatment with intravenous antibiotics should be promptly initiated. Standard treatment is vancomycin plus a third-generation cephalosporin. If intracranial involvement is apparent on imaging or if the etiology is thought to be from chronic sinusitis or a dental infection, metronidazole should be added. Once physical symptoms have begun to improve, a switch to oral antibiotics can occur. Trimethoprim-sulfamethoxazole or Clindamycin plus amoxicillin-clavulanic acid or a cephalosporin are the drugs of choice with total duration of antibiotics of at least 2 weeks. Surgical drainage of any abscess is usually indicated for the antibiotics to be most effective.
References:
1. UpToDate
2. https://www.ncbi.nlm.nih.gov/books/NBK507901/Incorrect
The correct answer is D. Orbital and preseptal (periorbital) cellulitis both have erythema and edema surrounding an eye and some degree of pain, however, preseptal is a superficial infection which does not involve the deeper structures resulting in the symptoms mentioned. Chemosis is a diffuse swelling of the conjunctiva that can accompany orbital cellulitis, but of itself does not involve pain, vision changes, or movement limitations. Glaucoma, especially acute angle closure-type, causes significant eye pain and diminished vision but usually not range of motion limitations or double vision. Iritis is a painful inflammatory condition of the iris usually caused by trauma, infection, or autoimmune etiologies. In addition to pain, blurred vision, photophobia, and redness occur but usually without ocular entrapment or proptosis.
Preseptal cellulitis is more common than orbital (post-septal), however, orbital is the more severe disorder which can lead to cavernous sinus thrombosis, intracranial abscess, or permanent vision loss. The pain from orbital cellulitis is a deeper pain and the patient generally appears more ill and uncomfortable. Orbital cellulitis occurs most commonly in children and is usually a complication of ethmoid rhinosinusitis due to staph or strep bacteria. Other causes include dental infections, orbital trauma, ophthalmic surgery, and dacryocystitis.
The provider should maintain a high level of suspicion for a diagnosis of orbital cellulitis in anyone who presents with swelling and erythema surrounding an eye, unexplained deep orbital pain, or restricted eye movements. The diagnosis can be confirmed with CT or MRI, with the former being adequate in most cases. Treatment with intravenous antibiotics should be promptly initiated. Standard treatment is vancomycin plus a third-generation cephalosporin. If intracranial involvement is apparent on imaging or if the etiology is thought to be from chronic sinusitis or a dental infection, metronidazole should be added. Once physical symptoms have begun to improve, a switch to oral antibiotics can occur. Trimethoprim-sulfamethoxazole or Clindamycin plus amoxicillin-clavulanic acid or a cephalosporin are the drugs of choice with total duration of antibiotics of at least 2 weeks. Surgical drainage of any abscess is usually indicated for the antibiotics to be most effective.
References:
1. UpToDate
2. https://www.ncbi.nlm.nih.gov/books/NBK507901/ -
Question 16 of 56
16. Question
What is the most common cause of adult blindness?
Correct
The correct answer is D. All others listed are causes of adult blindness but not as common a cause as macular degeneration. The exact mechanism that leads to macular degeneration is poorly understood. There may be a genetic component which results in variations of the compliment pathway. Other risk factors include advancing age, smoking, hypertension, obesity, excess sun exposure, and a low fatty acid, dark green vegetable diet. There are two types of macular degeneration, atrophic (also called dry or geographic) and neovascular (also called wet or exudative). Atrophic degeneration is the result of degeneration of the perimeter of the retina and retinal pigment epithelium. Visual loss is usually bilateral and gradual. The neovascular-type is caused by excess vessel growth which eventually results in leakage of serous fluid or blood and fibrosis. The vision loss is more rapid with this type and usually one eye is effected first, followed by the other. The primary symptoms a patient may report include straight line distortion/waviness and gradual central visual field loss while maintaining peripheral visual fields. On exam, atrophic-type will cause visible drusen (yellowish deposits) on funduscopic exam. There may also be areas of decreased pigmentation which represent retinal atrophy. Cases of neovascular origin will demonstrate subretinal edema or blood as well as a grayish-green appearance in the macular which represents the neovascular collection. In addition to the dilated eye funduscopic exam, fluorescein dye retinal angiography and optical coherence tomography are specialized tests that can help to confirm the diagnosis.
Treatment is aimed at slowing the progression. For cases of atrophic degeneration, smoking cessation, strict blood pressure control, and a vitamin supplement regiment. Suggested supplements vitamins C & E, lutein, zeaxanthin, zinc oxide, and copper (cupric oxide). Patients suffering from neovascular degeneration will also benefit from the same supportive care recommendations as well as intravitreous injection of a vascular endothelial growth factor inhibitor and/or photodynamic therapy. Prevention recommendations include smoking cessation, adequate blood pressure control, healthy diet and physical activity. Vitamin B supplements may have some preventative benefits.
References:
1. UpToDate
2. https://www.merckmanuals.com/professional/SearchResults?query=macular+degeneration
3. Current Medical Diagnosis and TreatmentIncorrect
The correct answer is D. All others listed are causes of adult blindness but not as common a cause as macular degeneration. The exact mechanism that leads to macular degeneration is poorly understood. There may be a genetic component which results in variations of the compliment pathway. Other risk factors include advancing age, smoking, hypertension, obesity, excess sun exposure, and a low fatty acid, dark green vegetable diet. There are two types of macular degeneration, atrophic (also called dry or geographic) and neovascular (also called wet or exudative). Atrophic degeneration is the result of degeneration of the perimeter of the retina and retinal pigment epithelium. Visual loss is usually bilateral and gradual. The neovascular-type is caused by excess vessel growth which eventually results in leakage of serous fluid or blood and fibrosis. The vision loss is more rapid with this type and usually one eye is effected first, followed by the other. The primary symptoms a patient may report include straight line distortion/waviness and gradual central visual field loss while maintaining peripheral visual fields. On exam, atrophic-type will cause visible drusen (yellowish deposits) on funduscopic exam. There may also be areas of decreased pigmentation which represent retinal atrophy. Cases of neovascular origin will demonstrate subretinal edema or blood as well as a grayish-green appearance in the macular which represents the neovascular collection. In addition to the dilated eye funduscopic exam, fluorescein dye retinal angiography and optical coherence tomography are specialized tests that can help to confirm the diagnosis.
Treatment is aimed at slowing the progression. For cases of atrophic degeneration, smoking cessation, strict blood pressure control, and a vitamin supplement regiment. Suggested supplements vitamins C & E, lutein, zeaxanthin, zinc oxide, and copper (cupric oxide). Patients suffering from neovascular degeneration will also benefit from the same supportive care recommendations as well as intravitreous injection of a vascular endothelial growth factor inhibitor and/or photodynamic therapy. Prevention recommendations include smoking cessation, adequate blood pressure control, healthy diet and physical activity. Vitamin B supplements may have some preventative benefits.
References:
1. UpToDate
2. https://www.merckmanuals.com/professional/SearchResults?query=macular+degeneration
3. Current Medical Diagnosis and Treatment -
Question 17 of 56
17. Question
A patient who has the findings seen on the above image obtained from a bedside ocular ultrasound would most likely be presenting with which of the following symptoms?
Correct
The correct answer is B. The ultrasound image is consistent with a retinal detachment. Visual floaters and painless vision loss are the most common symptoms. Most cases of diplopia (double vision) are secondary to problems involving the muscles that control the eye or ophthalmoplegic migraine. An intraocular foreign body is usually visible on ultrasound but is unlikely to be this shape or this large. Ultrasound is not generally used for evaluation of a corneal foreign body. Causes of color blindness would not result in abnormal ultrasound images.
The retina is a highly organized structure of neurons located along the posterior aspect of the inside of the eye. The tissue underneath (choroid) is highly vascular. If the retina begins to detach from the choroid, the blood supply to the neurons is compromised and fluid is allowed to accumulate between the two layers contributing to further detachment. Risk factors for retinal detachment include advancing age, underlying history of nearsightedness (myopia), family history, intraocular surgeries, trauma, diabetes, and sickle cell disease. After age 70 the risk goes up due to the natural course of posterior vitreous shrinkage which pulls on the retina. This pulling away from the retina is what causes a patient to experience floaters, flashes of light, and/or a flurry of black spots. Floaters are the result of collagen fibers from the vitreous gel clumping together. Flashes of light occur as the vitreous pulls against the retina, depolarizing the neurons within the retina. Black spots are the result of hemorrhage within the vitreous. Vision loss generally expands starting from one area, usually the periphery. Once the macula is reached, central vision loss occurs. All of these processes are generally painless.
The diagnosis should be high on the differential in a patient presenting with these symptoms. A dilated exam and slit lamp evaluation, usually conducted by an ophthalmologist, is the process of confirming the diagnosis in most cases. A wrinkled or wavy appearance of the retina is usually visible. If available, a low power bedside ultrasound can be performed and has been shown to highly sensitive and specific (94%; 96%) to rule-in the diagnosis but should not be used to rule-out if the exam is unremarkable in a patient with these complaints. An abnormal finding would reveal a wavy, folded, or sometimes curved membrane towards the middle to back of the posterior chamber.
Patients with floaters or flashes without visible detachment or vision loss can be treated conservatively and reassessed by an ophthalmologist within 2-3 months or immediately if vision loss starts to occur. Floaters may take up to 12 months to resolve. Patients with peripheral vision loss should be urgently seen by a specialist while patients with central loss should be seen emergently (24-48 hours). Treatment options include using laser or cryoretinopexy with the goal of creating an adhesion between the retina and choroid, thus preventing additional detachment or surgical reattachment for larger detachments, especially involving the macula.
References:
1. UpToDate
2. https://onlinelibrary.wiley.com/doi/10.1111/acem.13682
3. Current Medical Diagnosis and TreatmentIncorrect
The correct answer is B. The ultrasound image is consistent with a retinal detachment. Visual floaters and painless vision loss are the most common symptoms. Most cases of diplopia (double vision) are secondary to problems involving the muscles that control the eye or ophthalmoplegic migraine. An intraocular foreign body is usually visible on ultrasound but is unlikely to be this shape or this large. Ultrasound is not generally used for evaluation of a corneal foreign body. Causes of color blindness would not result in abnormal ultrasound images.
The retina is a highly organized structure of neurons located along the posterior aspect of the inside of the eye. The tissue underneath (choroid) is highly vascular. If the retina begins to detach from the choroid, the blood supply to the neurons is compromised and fluid is allowed to accumulate between the two layers contributing to further detachment. Risk factors for retinal detachment include advancing age, underlying history of nearsightedness (myopia), family history, intraocular surgeries, trauma, diabetes, and sickle cell disease. After age 70 the risk goes up due to the natural course of posterior vitreous shrinkage which pulls on the retina. This pulling away from the retina is what causes a patient to experience floaters, flashes of light, and/or a flurry of black spots. Floaters are the result of collagen fibers from the vitreous gel clumping together. Flashes of light occur as the vitreous pulls against the retina, depolarizing the neurons within the retina. Black spots are the result of hemorrhage within the vitreous. Vision loss generally expands starting from one area, usually the periphery. Once the macula is reached, central vision loss occurs. All of these processes are generally painless.
The diagnosis should be high on the differential in a patient presenting with these symptoms. A dilated exam and slit lamp evaluation, usually conducted by an ophthalmologist, is the process of confirming the diagnosis in most cases. A wrinkled or wavy appearance of the retina is usually visible. If available, a low power bedside ultrasound can be performed and has been shown to highly sensitive and specific (94%; 96%) to rule-in the diagnosis but should not be used to rule-out if the exam is unremarkable in a patient with these complaints. An abnormal finding would reveal a wavy, folded, or sometimes curved membrane towards the middle to back of the posterior chamber.
Patients with floaters or flashes without visible detachment or vision loss can be treated conservatively and reassessed by an ophthalmologist within 2-3 months or immediately if vision loss starts to occur. Floaters may take up to 12 months to resolve. Patients with peripheral vision loss should be urgently seen by a specialist while patients with central loss should be seen emergently (24-48 hours). Treatment options include using laser or cryoretinopexy with the goal of creating an adhesion between the retina and choroid, thus preventing additional detachment or surgical reattachment for larger detachments, especially involving the macula.
References:
1. UpToDate
2. https://onlinelibrary.wiley.com/doi/10.1111/acem.13682
3. Current Medical Diagnosis and Treatment -
Question 18 of 56
18. Question
A 50-year-old male presents for a new patient visit after having no PCP for his adult life. A thorough physical exam reveals the above image on funduscopic visualization. In addition to diabetes, what other disorder is this most consistent with?Correct
The correct answer is B. The image shows hemorrhages, exudates, cotton-wool spots, aneurysms, and neovascularization. Although classically associated with diabetes, all these findings can occur in patients with lupus as well. Cotton-wool spots are the most common retinopathy findings in patients with lupus. Glaucoma does not cause changes of the retina. Macular degeneration does not result in cotton-wool spots or aneurysms but can result in decreased pigmentation of the retinal or a grayish-green appearance in the macula area. Sickle cell disease results mostly in neovascularization but can also cause vitreous hemorrhages and retinal detachments.
Any disorder capable of damaging endothelial tissue will eventually cause a compromise of the capillary walls and allow microaneurysms (black arrow) to form. These microaneurysms can then rupture leading to visible hemorrhages (yellow arrow). Cotton-wool spots (green arrow) occur when a disease process blocks capillaries and subsequently infarcts the nerve fiber layer. The blocked capillaries or arterioles also trigger the stimulation of pro-angiogenic factors which result in areas of neovascularization (purple arrow). Hard exudates (blue arrow) form as the result of endothelial damage allowing for increased wall permeability and subsequent fluid leakage under the macula. Once the fluid is reabsorbed, the lipid component is left behind and forms waxy, yellow deposits.
A dilated retinopathy screening performed by an ophthalmologist or optometrist is recommended for type 2 diabetics as soon as possible after initial diagnosis of diabetes. Type 1 diabetics should be screened for retinopathy within 5 years of diagnosis. Most should be reassessed annually unless there is no evidence for disease and diabetes is well managed, in which case, every other year exams may be considered. For children who have had diabetes for 3-5 years, screening should begin after age 11 and repeated every two years. Patient education on the importance of disease (diabetes, hypertension, lupus, etc.) control is the key to minimizing the advancement of disease. Treatment options based on the advancement of the disease can include laser panretinal photocoagulation, intravitreal anti-vascular endothelial growth factor (VEGF) agents, intravitreal glucocorticoids, or vitrectomy.
References:
A. UpToDate
B. https://www.reviewofophthalmology.com/article/posterior-segment-findings-in-sle
C. https://eyewiki.aao.org/Diabetic_Retinopathy_PathophysiologyIncorrect
The correct answer is B. The image shows hemorrhages, exudates, cotton-wool spots, aneurysms, and neovascularization. Although classically associated with diabetes, all these findings can occur in patients with lupus as well. Cotton-wool spots are the most common retinopathy findings in patients with lupus. Glaucoma does not cause changes of the retina. Macular degeneration does not result in cotton-wool spots or aneurysms but can result in decreased pigmentation of the retinal or a grayish-green appearance in the macula area. Sickle cell disease results mostly in neovascularization but can also cause vitreous hemorrhages and retinal detachments.
Any disorder capable of damaging endothelial tissue will eventually cause a compromise of the capillary walls and allow microaneurysms (black arrow) to form. These microaneurysms can then rupture leading to visible hemorrhages (yellow arrow). Cotton-wool spots (green arrow) occur when a disease process blocks capillaries and subsequently infarcts the nerve fiber layer. The blocked capillaries or arterioles also trigger the stimulation of pro-angiogenic factors which result in areas of neovascularization (purple arrow). Hard exudates (blue arrow) form as the result of endothelial damage allowing for increased wall permeability and subsequent fluid leakage under the macula. Once the fluid is reabsorbed, the lipid component is left behind and forms waxy, yellow deposits.
A dilated retinopathy screening performed by an ophthalmologist or optometrist is recommended for type 2 diabetics as soon as possible after initial diagnosis of diabetes. Type 1 diabetics should be screened for retinopathy within 5 years of diagnosis. Most should be reassessed annually unless there is no evidence for disease and diabetes is well managed, in which case, every other year exams may be considered. For children who have had diabetes for 3-5 years, screening should begin after age 11 and repeated every two years. Patient education on the importance of disease (diabetes, hypertension, lupus, etc.) control is the key to minimizing the advancement of disease. Treatment options based on the advancement of the disease can include laser panretinal photocoagulation, intravitreal anti-vascular endothelial growth factor (VEGF) agents, intravitreal glucocorticoids, or vitrectomy.
References:
A. UpToDate
B. https://www.reviewofophthalmology.com/article/posterior-segment-findings-in-sle
C. https://eyewiki.aao.org/Diabetic_Retinopathy_Pathophysiology -
Question 19 of 56
19. Question
An adult presents after being struck in the eye by a batted baseball. Eye movement in which direction is most likely to be limited if a fracture has occurred?
Correct
The correct answer is D. The inferior rectus muscle of the eye is the muscle most commonly involved in orbital floor fractures. This muscle is responsible for downward gaze which would lead one to think that inferior gaze is restricted, however, the muscle is usually entrapped in the bony fragments causing the eye to be stuck looking down and unable to move upward.
Fractures of the orbit inferior wall, or “floor”, are common consequents of an adult being struck in the periorbital area, usually by a hard object under moderate force, such as a baseball or fist. Children are more likely to sustain superior wall, or “roof”, injuries from the same mechanism, likely due to a large head to midface ratio at this age as well as incomplete development of the sinuses. In addition to fractures of any of the bones, other structures that may be involved in an injury include the muscles (levator, superior rectus, superior oblique, lateral rectus, medial rectus, inferior rectus and inferior oblique), lacrimal duct and canthus, nerves (supraorbital or infraorbital), and periorbital fat in addition to possible globe or intracranial damage. Injury to the infraorbital nerve will result in diminished sensation to the cheek while supraorbital nerve involvement results in numbness to the forehead. Other findings include abnormal pupil shape/reactivity which may indicated a ruptured globe or foreign body penetration, hyphema or vitreous hemorrhage, lens displacement,
proptosis or enophthalmos, or excessively firm globe.CT scan of the orbits without contrast is the imaging modality of choice. Ophthalmology or oculoplastic surgery should be consulted for possible surgical release of the entrapped fat or muscle and stabilization of the fracture(s). Prophylactic antibiotics to cover typical sinus pathogens as well as corticosteroids to reduce swelling should be discussed at the time of consultation.
References:
A. UpToDate
B. https://www.ncbi.nlm.nih.gov/books/NBK534825/
C. https://www.sciencedirect.com/science/article/pii/S2214854X21000157
D. https://www.sciencedirect.com/science/article/pii/S2214541919300252Incorrect
The correct answer is D. The inferior rectus muscle of the eye is the muscle most commonly involved in orbital floor fractures. This muscle is responsible for downward gaze which would lead one to think that inferior gaze is restricted, however, the muscle is usually entrapped in the bony fragments causing the eye to be stuck looking down and unable to move upward.
Fractures of the orbit inferior wall, or “floor”, are common consequents of an adult being struck in the periorbital area, usually by a hard object under moderate force, such as a baseball or fist. Children are more likely to sustain superior wall, or “roof”, injuries from the same mechanism, likely due to a large head to midface ratio at this age as well as incomplete development of the sinuses. In addition to fractures of any of the bones, other structures that may be involved in an injury include the muscles (levator, superior rectus, superior oblique, lateral rectus, medial rectus, inferior rectus and inferior oblique), lacrimal duct and canthus, nerves (supraorbital or infraorbital), and periorbital fat in addition to possible globe or intracranial damage. Injury to the infraorbital nerve will result in diminished sensation to the cheek while supraorbital nerve involvement results in numbness to the forehead. Other findings include abnormal pupil shape/reactivity which may indicated a ruptured globe or foreign body penetration, hyphema or vitreous hemorrhage, lens displacement,
proptosis or enophthalmos, or excessively firm globe.CT scan of the orbits without contrast is the imaging modality of choice. Ophthalmology or oculoplastic surgery should be consulted for possible surgical release of the entrapped fat or muscle and stabilization of the fracture(s). Prophylactic antibiotics to cover typical sinus pathogens as well as corticosteroids to reduce swelling should be discussed at the time of consultation.
References:
A. UpToDate
B. https://www.ncbi.nlm.nih.gov/books/NBK534825/
C. https://www.sciencedirect.com/science/article/pii/S2214854X21000157
D. https://www.sciencedirect.com/science/article/pii/S2214541919300252 -
Question 20 of 56
20. Question
Which of the following should be performed based on the above image?
Correct
The correct answer is B. The image demonstrates multiple faint linear areas of fluorescein uptake in addition to the larger areas. While some of the larger appear dendritic, the very straight vertical areas of staining indicate this is likely due to a foreign body trapped under the upper eyelid. Burr scraping is indicated for a residual rust ring which can occur if a metal foreign body is stuck in the cornea. There is no evidence for a rust ring on this image, which would typically be best seen using a slit lamp. a pH test would be performed if a chemical injury was reported. This would be unlikely given the vertical abrasions. Tonometry is used when glaucoma is suspected to assess the intraocular pressure, however, corneal defects are unlikely to be present with glaucoma. Ultrasound is useful if intraocular foreign body, lens displacement or retinal detachment are suspected.
Corneal abrasions are usually caused by foreign bodies but can be the result of improper contact lens use or, less commonly, spontaneous. Photophobia and a foreign body sensation are the most common presenting complaints. The foreign body sensation may persist even if the object has spontaneously fallen out due to the richly innervated corneal epithelium.
Physical exam should start with a visual acuity using corrective lenses if used and available, ideally prior to instilling any drops or stain into the eye. The exam may be facilitated by administering a topical anesthetic such as tetracaine or proparacaine. The pain should significantly improve within a few seconds but may wear off in 10-15 minutes. Re-administration is acceptable however, the patient should not be sent home with the anesthetic due to the possibility of developing an ulceration or perforation which can lead to scaring or blindness. An irregularly shaped or nonreactive pupil as well as the presence of blood in the anterior chamber (hyphema) raises concern for a penetrating injury. Assess for a visible retained foreign body on the cornea. Tangential lighting may help. Also assess the undersurface of the upper and lower eyelids for a foreign body. The lower lid can easily be pulled down using a finger, however, the upper lid requires use of cotton swab (tip or wooden shaft) placed over the lid followed by grasping the lashes and lifting the lid over the swab while slightly pushing down with the swab. Once the lid is everted, the swab can be removed to minimize patient discomfort. If a foreign body is seen, it is easily removed from the lids using the swab. A corneal foreign body may come off with a cotton swab but frequently will required a needle or burr to extricate from the cornea. After the general assessment, fluorescein dye can be instilled to highlight any abrasions. Dye can be instilled by placing one drop of saline or anesthetic on the paper strip and placing on the inside of the lower lid. Ask the patient to blink after removing the strip and the dye will disburse over the surface of the eye and settle into any areas of epithelial defect, allowing the basement membrane to be stained. The retained stain on the basement membrane becomes a yellow-green color when exposed to a cobalt blue filter or a Wood’s lamp. If the stain appears to be visibly streaming down and away from the area, penetration into the anterior chamber is likely. This is referred to as a Seidel sign.
Topical antibiotic solution or ointment is usually all that is needed for the treatment of a simple abrasion. Three to four days is usually an adequate duration. Pain management with oral analgesics or a one-time administration of a cycloplegic drop should be considered. Always determine the patient’s tetanus vaccine status and update if over 5 years. If discomfort persists or worsens, follow-up with ophthalmology is warranted.
References:
A. UpTodate
B. https://www.ncbi.nlm.nih.gov/books/NBK532960/Incorrect
The correct answer is B. The image demonstrates multiple faint linear areas of fluorescein uptake in addition to the larger areas. While some of the larger appear dendritic, the very straight vertical areas of staining indicate this is likely due to a foreign body trapped under the upper eyelid. Burr scraping is indicated for a residual rust ring which can occur if a metal foreign body is stuck in the cornea. There is no evidence for a rust ring on this image, which would typically be best seen using a slit lamp. a pH test would be performed if a chemical injury was reported. This would be unlikely given the vertical abrasions. Tonometry is used when glaucoma is suspected to assess the intraocular pressure, however, corneal defects are unlikely to be present with glaucoma. Ultrasound is useful if intraocular foreign body, lens displacement or retinal detachment are suspected.
Corneal abrasions are usually caused by foreign bodies but can be the result of improper contact lens use or, less commonly, spontaneous. Photophobia and a foreign body sensation are the most common presenting complaints. The foreign body sensation may persist even if the object has spontaneously fallen out due to the richly innervated corneal epithelium.
Physical exam should start with a visual acuity using corrective lenses if used and available, ideally prior to instilling any drops or stain into the eye. The exam may be facilitated by administering a topical anesthetic such as tetracaine or proparacaine. The pain should significantly improve within a few seconds but may wear off in 10-15 minutes. Re-administration is acceptable however, the patient should not be sent home with the anesthetic due to the possibility of developing an ulceration or perforation which can lead to scaring or blindness. An irregularly shaped or nonreactive pupil as well as the presence of blood in the anterior chamber (hyphema) raises concern for a penetrating injury. Assess for a visible retained foreign body on the cornea. Tangential lighting may help. Also assess the undersurface of the upper and lower eyelids for a foreign body. The lower lid can easily be pulled down using a finger, however, the upper lid requires use of cotton swab (tip or wooden shaft) placed over the lid followed by grasping the lashes and lifting the lid over the swab while slightly pushing down with the swab. Once the lid is everted, the swab can be removed to minimize patient discomfort. If a foreign body is seen, it is easily removed from the lids using the swab. A corneal foreign body may come off with a cotton swab but frequently will required a needle or burr to extricate from the cornea. After the general assessment, fluorescein dye can be instilled to highlight any abrasions. Dye can be instilled by placing one drop of saline or anesthetic on the paper strip and placing on the inside of the lower lid. Ask the patient to blink after removing the strip and the dye will disburse over the surface of the eye and settle into any areas of epithelial defect, allowing the basement membrane to be stained. The retained stain on the basement membrane becomes a yellow-green color when exposed to a cobalt blue filter or a Wood’s lamp. If the stain appears to be visibly streaming down and away from the area, penetration into the anterior chamber is likely. This is referred to as a Seidel sign.
Topical antibiotic solution or ointment is usually all that is needed for the treatment of a simple abrasion. Three to four days is usually an adequate duration. Pain management with oral analgesics or a one-time administration of a cycloplegic drop should be considered. Always determine the patient’s tetanus vaccine status and update if over 5 years. If discomfort persists or worsens, follow-up with ophthalmology is warranted.
References:
A. UpTodate
B. https://www.ncbi.nlm.nih.gov/books/NBK532960/ -
Question 21 of 56
21. Question
A patient presents with an injury to the eye resulting in decreased vision. Physical exam reveals a subconjunctival hemorrhage and a teardrop shaped pupil. Which of the following is the next most appropriate step?
Correct
The correct answer is D. The presence of an abnormal pupil shape raises the concern for possible globe rupture. When globe rupture is suspected, avoid putting anything into the eye or putting pressure on the eye itself or the surrounding structures. Placing a metal shield over the eye will provide some protection while awaiting definitive treatment. Avoid using cloth patches as they may apply pressure to the globe and cause expulsion of contents.
A ruptured globe occurs as the result of a blunt or penetrating trauma to the eye which creates a defect in the cornea and/or sclera. The weakest area of the sclera, and therefore the most common area responsible for rupture, is along the posterior equator of the eye near the area where the rectus muscles insert. This disruption to the structure of the globe allows for an abnormal pupil shape. Other findings of globe rupture include vision impairment, hyphema, subconjunctival hemorrhage, shallow anterior chamber depth, relative afferent pupillary defect, and possible proptosis if retrobulbar hematoma is present. Depending on the mechanism of injury, swelling and ecchymosis surrounding the eye may be present.
The workup when globe rupture is suspected a CT scan of the orbits without contrast. Images may show an intraocular foreign body or air, deformity or volume loss of the globe, vitreous hemorrhage, decreased depth of the anterior chamber, or an irregular contour of the sclera. Ophthalmology consultation should be obtained when the diagnosis is confirmed. IV antiemetics should be administered early to minimize increasing the pressure exerted to the eye if vomiting occurs. IV analgesics along with sedation should also be considered. Update tetanus if warranted along with IV antibiotics if an open injury is present. Surgical repair of the globe defect is the usual definitive treatment. The majority of cases will have preserved vision, to some extent. The primary complication of a globe rupture is the formation of a traumatic cataract. Other complications include corneal scaring or retinal detachment.
References:
A. UpTodate
B. https://www.ncbi.nlm.nih.gov/books/NBK551637/
C. https://www.ncbi.nlm.nih.gov/books/NBK470379/Incorrect
The correct answer is D. The presence of an abnormal pupil shape raises the concern for possible globe rupture. When globe rupture is suspected, avoid putting anything into the eye or putting pressure on the eye itself or the surrounding structures. Placing a metal shield over the eye will provide some protection while awaiting definitive treatment. Avoid using cloth patches as they may apply pressure to the globe and cause expulsion of contents.
A ruptured globe occurs as the result of a blunt or penetrating trauma to the eye which creates a defect in the cornea and/or sclera. The weakest area of the sclera, and therefore the most common area responsible for rupture, is along the posterior equator of the eye near the area where the rectus muscles insert. This disruption to the structure of the globe allows for an abnormal pupil shape. Other findings of globe rupture include vision impairment, hyphema, subconjunctival hemorrhage, shallow anterior chamber depth, relative afferent pupillary defect, and possible proptosis if retrobulbar hematoma is present. Depending on the mechanism of injury, swelling and ecchymosis surrounding the eye may be present.
The workup when globe rupture is suspected a CT scan of the orbits without contrast. Images may show an intraocular foreign body or air, deformity or volume loss of the globe, vitreous hemorrhage, decreased depth of the anterior chamber, or an irregular contour of the sclera. Ophthalmology consultation should be obtained when the diagnosis is confirmed. IV antiemetics should be administered early to minimize increasing the pressure exerted to the eye if vomiting occurs. IV analgesics along with sedation should also be considered. Update tetanus if warranted along with IV antibiotics if an open injury is present. Surgical repair of the globe defect is the usual definitive treatment. The majority of cases will have preserved vision, to some extent. The primary complication of a globe rupture is the formation of a traumatic cataract. Other complications include corneal scaring or retinal detachment.
References:
A. UpTodate
B. https://www.ncbi.nlm.nih.gov/books/NBK551637/
C. https://www.ncbi.nlm.nih.gov/books/NBK470379/ -
Question 22 of 56
22. Question
A patient sustains a blunt injury to the eye and became concerned after looking in the mirror and noticing the findings on the above image. What is the most appropriate initial treatment?Correct
The correct answer is E. The findings are consistent with a collection of blood in the anterior chamber, called a hyphema. There is no evidence on this image consistent with an injury to the cornea requiring topical antibiotics. Surgical evacuation of the clot is rarely necessary. Aspirin would allow for additional bleeding and should be avoided. Older treatment recommendations included placing the patient in a 90-degree upright position for several days. This has since been determined to decrease arterial blood flow to the eye which is not beneficial. Thirty degrees of elevation has been shown to be sufficient to allow blood to pool at the bottom of the chamber and avoid obstructing the trabecular outflow with clots.
A hyphema can occur after either blunt or penetrating trauma which tears vessels within the ciliary body or iris. This blood then collects in the anterior chamber which consists of the space between the lens, cornea, iris, and angle. The same area can have pus instead of blood when an infection is present. This is referred to as a hypopyon. Differentiating between in the two clinically is important. A hyphema is best visualized using a good light source with the patient sitting upright. The patient may have unequal pupil size, called anisocoria. Vision impact range from mild blurriness to only being able to appreciate changes in light intensity. In addition to the visible hyphema, the injury may also result in an orbital compartment syndrome. In this case, proptosis may be noticed, or a feeling of tight eyelids may be reported by the patient. This can be confirmed with tonometry.
A spontaneous hyphema (atraumatic) warrants a workup for inherited bleeding disorders if not currently on any anticoagulants for other medical conditions. African Americans should be tested for sickle cell disease if not previously tested. A hyphema is a clinical diagnosis, however, in the setting of trauma or if the hyphema is large enough to obscure visualization of the posterior chamber, imaging should be considered to assess for more extensive damage. CT of the orbits without contrast is a good choice. Ultrasound can be used if there is no evidence for globe rupture, otherwise, the direct pressure required to perform the exam would be contraindicated. Treatment for a simple hyphema involves rest and reclining to a 30-degree angle when possible for one week. Intraocular pressure should be checked by ophthalmology daily. Increased pressure can be treated with topical beta adrenergic blockers or carbonic anhydrase inhibitors. In patients without a history of narrow angle glaucoma, a cycloplegic such as cyclopentolate or scopolamine can be administered for pain. Oral analgesics, except NSAIDS, can also be given. Topical glucocorticoid drops can be used in conjunction with an ophthalmologist to help slow bleeding. Most hyphema’s will reabsorb however some will require surgical clot evacuation. Patients with a history of sickle cell disease or coagulopathies are more likely to require evacuation.
References:
1. UpToDate
2. https://www.ncbi.nlm.nih.gov/books/NBK507802/Incorrect
The correct answer is E. The findings are consistent with a collection of blood in the anterior chamber, called a hyphema. There is no evidence on this image consistent with an injury to the cornea requiring topical antibiotics. Surgical evacuation of the clot is rarely necessary. Aspirin would allow for additional bleeding and should be avoided. Older treatment recommendations included placing the patient in a 90-degree upright position for several days. This has since been determined to decrease arterial blood flow to the eye which is not beneficial. Thirty degrees of elevation has been shown to be sufficient to allow blood to pool at the bottom of the chamber and avoid obstructing the trabecular outflow with clots.
A hyphema can occur after either blunt or penetrating trauma which tears vessels within the ciliary body or iris. This blood then collects in the anterior chamber which consists of the space between the lens, cornea, iris, and angle. The same area can have pus instead of blood when an infection is present. This is referred to as a hypopyon. Differentiating between in the two clinically is important. A hyphema is best visualized using a good light source with the patient sitting upright. The patient may have unequal pupil size, called anisocoria. Vision impact range from mild blurriness to only being able to appreciate changes in light intensity. In addition to the visible hyphema, the injury may also result in an orbital compartment syndrome. In this case, proptosis may be noticed, or a feeling of tight eyelids may be reported by the patient. This can be confirmed with tonometry.
A spontaneous hyphema (atraumatic) warrants a workup for inherited bleeding disorders if not currently on any anticoagulants for other medical conditions. African Americans should be tested for sickle cell disease if not previously tested. A hyphema is a clinical diagnosis, however, in the setting of trauma or if the hyphema is large enough to obscure visualization of the posterior chamber, imaging should be considered to assess for more extensive damage. CT of the orbits without contrast is a good choice. Ultrasound can be used if there is no evidence for globe rupture, otherwise, the direct pressure required to perform the exam would be contraindicated. Treatment for a simple hyphema involves rest and reclining to a 30-degree angle when possible for one week. Intraocular pressure should be checked by ophthalmology daily. Increased pressure can be treated with topical beta adrenergic blockers or carbonic anhydrase inhibitors. In patients without a history of narrow angle glaucoma, a cycloplegic such as cyclopentolate or scopolamine can be administered for pain. Oral analgesics, except NSAIDS, can also be given. Topical glucocorticoid drops can be used in conjunction with an ophthalmologist to help slow bleeding. Most hyphema’s will reabsorb however some will require surgical clot evacuation. Patients with a history of sickle cell disease or coagulopathies are more likely to require evacuation.
References:
1. UpToDate
2. https://www.ncbi.nlm.nih.gov/books/NBK507802/ -
Question 23 of 56
23. Question
A 60-year-old male with a history of hypertension, diabetes, and tobacco abuse presents with abrupt onset of painless vision loss in the right eye. He is only able to see shadows out of the affected eye. The visible structures of the eye are unremarkable on inspection. A swinging flashlight test is abnormal, and the funduscopic exam is shown in the above image. What is the most likely diagnosis?
Correct
The correct answer is B. All of the listed disorders can result in an abnormal swinging flashlight sign. Glaucoma should not result in the funduscopic findings. In cases of macular degeneration, drusen, areas of decreased pigmentation, or a grayish-green appearance in the macula can occur. Retinal detachment results in a wavy or wrinkled appearance. Optic neuritis usually presents with an unremarkable funduscopic exam.
A relative afferent pupillary defect is detected using the swinging flashlight test. Start by dimming the lights and asking the patient to focus on a distant object. Shine a light directly into one eye and then the other for approximately 3 seconds each, keeping it the same distance from each pupil. In a patient with bilateral intact retina and optic nerves, both eyes constrict equally with direct and consensual light. In a patient with retinal or optic nerve disease, shining the light into either eye will initially cause both pupils to incompletely constrict. When the light is shone into the unaffected eye, both pupils will constrict further. Finally, when the light is shone back into the affected eye, both pupils will dilate. This test may also be useful in patients with glaucoma to determine if optic nerve damage is occurring secondary to the increase in pressure.
Retinal artery occlusions usually occur secondary to carotid atherosclerosis, cardiac emboli, sickle cell disease, hypercoagulopathy, and inflammatory vasculitis disorders. Vision loss is abrupt and painless. Funduscopic exam demonstrates pallor, as seen in the attached image. A “cherry red spot” (seen in image) is usually visible in the macula due to the retinal ischemia allowing it to become more translucent. Retinal emboli can sometime be seen as well. Intravenous fluorescein angiography can be used to confirm the diagnosis. Labs should be considered based on the history, exam and risk factors. These include ESR, CRP, ANA, lipid profile, fluorescent treponemal antibody absorption test, and hypercoagulability labs. Other modalities to consider include carotid artery ultrasound, EKG, echocardiogram, and Holter or event monitoring. Primary treatment is with intraarterial thrombolysis. Other options include globe massage, medications to reduce intraocular pressure, ocular vasodilator medications, hyperventilation or hyperbaric oxygen.
References:
1. UpToDate
2. https://www.ncbi.nlm.nih.gov/pmc/articles/PMC3588138/#:~:text=The%20'swinging%20light%20test'%20is,one%20eye%20at%20a%20time.
3. https://accessmedicine.mhmedical.com/content.aspx?bookid=1763§ionid=125433189
4. https://www.ncbi.nlm.nih.gov/books/NBK470354/Incorrect
The correct answer is B. All of the listed disorders can result in an abnormal swinging flashlight sign. Glaucoma should not result in the funduscopic findings. In cases of macular degeneration, drusen, areas of decreased pigmentation, or a grayish-green appearance in the macula can occur. Retinal detachment results in a wavy or wrinkled appearance. Optic neuritis usually presents with an unremarkable funduscopic exam.
A relative afferent pupillary defect is detected using the swinging flashlight test. Start by dimming the lights and asking the patient to focus on a distant object. Shine a light directly into one eye and then the other for approximately 3 seconds each, keeping it the same distance from each pupil. In a patient with bilateral intact retina and optic nerves, both eyes constrict equally with direct and consensual light. In a patient with retinal or optic nerve disease, shining the light into either eye will initially cause both pupils to incompletely constrict. When the light is shone into the unaffected eye, both pupils will constrict further. Finally, when the light is shone back into the affected eye, both pupils will dilate. This test may also be useful in patients with glaucoma to determine if optic nerve damage is occurring secondary to the increase in pressure.
Retinal artery occlusions usually occur secondary to carotid atherosclerosis, cardiac emboli, sickle cell disease, hypercoagulopathy, and inflammatory vasculitis disorders. Vision loss is abrupt and painless. Funduscopic exam demonstrates pallor, as seen in the attached image. A “cherry red spot” (seen in image) is usually visible in the macula due to the retinal ischemia allowing it to become more translucent. Retinal emboli can sometime be seen as well. Intravenous fluorescein angiography can be used to confirm the diagnosis. Labs should be considered based on the history, exam and risk factors. These include ESR, CRP, ANA, lipid profile, fluorescent treponemal antibody absorption test, and hypercoagulability labs. Other modalities to consider include carotid artery ultrasound, EKG, echocardiogram, and Holter or event monitoring. Primary treatment is with intraarterial thrombolysis. Other options include globe massage, medications to reduce intraocular pressure, ocular vasodilator medications, hyperventilation or hyperbaric oxygen.
References:
1. UpToDate
2. https://www.ncbi.nlm.nih.gov/pmc/articles/PMC3588138/#:~:text=The%20'swinging%20light%20test'%20is,one%20eye%20at%20a%20time.
3. https://accessmedicine.mhmedical.com/content.aspx?bookid=1763§ionid=125433189
4. https://www.ncbi.nlm.nih.gov/books/NBK470354/ -
Question 24 of 56
24. Question
A patient reports transient binocular visual loss. Which of the following is the most likely etiology?
Correct
The correct answer is C. The other disorders listed would usually result in monocular transient vision loss.
Amaurosis fugax is the general term used to describe monocular or binocular transient vision loss. It is not specific to a monocular vascular cause as some might think, although it is frequently used in this context. When only one eye is involved, the pathophysiology is usually located anterior to the optic chiasm. When both are involved, a disorder posterior to the optic chiasm should be considered. In addition to obtaining unilateral or bilateral involvement, some other specifics in the history can help narrow down the differential diagnosis. Episodes of a few seconds are typically due to papilledema while thromboembolic events generally last 1-15 minutes and a migraine aura etiology typically lasts 10-30 minutes.
When the cause is retinal ischemia or detachment, the vision loss is usually described as a curtain or shade coming down. Reporting of bright, flickering lights marching across the visual field are suggestive of a migraine etiology. Onset immediately after turning the neck, a posture change, or even immediately postprandial suggests carotid artery stenosis. If the symptoms are reproduced by gazing in a certain direction an orbital mass should be considered. And vision loss after exercise or a hot shower may be related to multiple sclerosis.Binocular transient vision loss has a much shorter differential diagnosis than monocular causes. Migraine, underlying seizure disorder, and brainstem stroke are the primary causes of transient binocular vision loss. Ocular migraines are the most common cause in young to middle-aged adults. The visual involvement can occur without an actual accompanying headache or can be an aura preceding the pain. The likely explanation regarding the pathophysiology is that a vasospasm occurs within the retinal or ciliary vasculature or that an area of the brain cortex is excited followed by neuron depression. Patients with vision loss secondary to a brainstem stroke will also usually have associated vertigo and nausea. Monocular transient vision loss etiologies include carotid artery plaque rupture or embolism from carotid dissection, giant cell (temporal) arteritis, cardiac embolism, hypotension, retinal vein occlusion, optic neuropathy or nerve compression, or papilledema. The workup can be narrowed based on history and any specific findings on initial physical exam but consideration should be given to a detailed funduscopic exam, ESR, CRP, coagulopathy labs, carotid duplex ultrasound, echocardiogram, MRI, and/or EEG.
References:
1. UpToDate
2. https://www.ncbi.nlm.nih.gov/books/NBK470528/Incorrect
The correct answer is C. The other disorders listed would usually result in monocular transient vision loss.
Amaurosis fugax is the general term used to describe monocular or binocular transient vision loss. It is not specific to a monocular vascular cause as some might think, although it is frequently used in this context. When only one eye is involved, the pathophysiology is usually located anterior to the optic chiasm. When both are involved, a disorder posterior to the optic chiasm should be considered. In addition to obtaining unilateral or bilateral involvement, some other specifics in the history can help narrow down the differential diagnosis. Episodes of a few seconds are typically due to papilledema while thromboembolic events generally last 1-15 minutes and a migraine aura etiology typically lasts 10-30 minutes.
When the cause is retinal ischemia or detachment, the vision loss is usually described as a curtain or shade coming down. Reporting of bright, flickering lights marching across the visual field are suggestive of a migraine etiology. Onset immediately after turning the neck, a posture change, or even immediately postprandial suggests carotid artery stenosis. If the symptoms are reproduced by gazing in a certain direction an orbital mass should be considered. And vision loss after exercise or a hot shower may be related to multiple sclerosis.Binocular transient vision loss has a much shorter differential diagnosis than monocular causes. Migraine, underlying seizure disorder, and brainstem stroke are the primary causes of transient binocular vision loss. Ocular migraines are the most common cause in young to middle-aged adults. The visual involvement can occur without an actual accompanying headache or can be an aura preceding the pain. The likely explanation regarding the pathophysiology is that a vasospasm occurs within the retinal or ciliary vasculature or that an area of the brain cortex is excited followed by neuron depression. Patients with vision loss secondary to a brainstem stroke will also usually have associated vertigo and nausea. Monocular transient vision loss etiologies include carotid artery plaque rupture or embolism from carotid dissection, giant cell (temporal) arteritis, cardiac embolism, hypotension, retinal vein occlusion, optic neuropathy or nerve compression, or papilledema. The workup can be narrowed based on history and any specific findings on initial physical exam but consideration should be given to a detailed funduscopic exam, ESR, CRP, coagulopathy labs, carotid duplex ultrasound, echocardiogram, MRI, and/or EEG.
References:
1. UpToDate
2. https://www.ncbi.nlm.nih.gov/books/NBK470528/ -
Question 25 of 56
25. Question
Parents report their 2-year-old child seems to repeatedly hold their head tilted to one side while awake for the past few months. The child was premature with a low birth weight but has been thriving since. Which test would be most likely to provide a definitive diagnosis?
Correct
The correct answer is C. The most likely diagnosis is congenital amblyopia which a cover-uncover test is used for evaluation at this age. A brain MRI would be useful if multiple sclerosis, stroke or mass were at the top of the differential, however, there is nothing in this scenario suggestive of that. The scenario describes a torticollis-like scenario which the patient using to compensate for poor vision in one eye. Torticollis is a soft tissue disorder which is not diagnosed with x-ray. A Dix-Hallpike test is used to evaluate for benign positional vertigo which would be a very unlikely diagnosis at this age and with this presentation. An electroencephalogram (EEG) is used to diagnose seizures and the movements described seem intentional.
Amblyopia it the term used to describe poor visual acuity due to abnormal development of structures involved in sight during infancy or early childhood. If developmental abnormalities are present, the resulting reduction in stimulation of the visual pathways further leads to a decline in the development of the visual cortex, advancing amblyopia. It is sometimes referred to as having a “lazy eye.” Risk factors associated with amblyopia include lid ptosis, preterm delivery (<30 weeks), low birth weight (<3 lb, 5 oz), cerebral palsy, Down syndrome), or a family history of amblyopia or strabismus.
Amblyopia is most frequently unilateral but can be bilateral. Due to patient age and associated immaturity, the history is very limited, therefore, asking the parent about specific irregularities during routine exams is the key to early diagnosis and preservation of vision. Ask about noticeable squinting or wandering of one eye which may indicate exotropia. Torticollis (head tilting to the side) may indicate an alignment problem while intentional neck extension may be compensating for a ptosis. Intentionally turning, as opposed to tilting, of the head may be a way to compensate for nystagmus. Physical exam findings can range from normal to visible opacifications of the cornea or lens, lid ptosis, or noticeable malalignment.
Because adult level visual acuity is usually achieved by 3-5 years of age, early recognition and referral is crucial in preventing or minimizing vision loss. Initial screening should occur during the newborn period and then again around 3-5 years of age or sooner if parental concerns occur. Newborn screening should include assessing the general appearance of the eyes and lids, pupil reactivity, ocular mobility, presence or absence of a red reflex, and gross ocular malalignment (tropia or strabismus) by assessing corneal light reflection. In a cooperative child under the age of 3, a cover-uncover test is used to evaluate for a phoria which is noticed when both eyes are not allowed to fixate on an object at the same time. After the age of 3 a visual acuity exam should be performed using an age-based eye chart. Treatment of amblyopia should start as soon as it is recognized and a specific cause is determined. Surgical removal of a congenital cataract or strabismus surgery may be necessary. Patching of the unaffected eye for 2 hours per day can be very effective. Alternatively, if a patch is not well tolerated, using atropine 1% drops on 2 consecutive days per week in the "normal" eye will cause pupil dilation resulting in blurring which forces the amblyopic eye to work harder at focusing on near distances. Correcting lens refractive errors with glasses should be done when warranted.
References:
1. https://www.aafp.org/afp/2019/1215/p745.html
2. UpToDateIncorrect
The correct answer is C. The most likely diagnosis is congenital amblyopia which a cover-uncover test is used for evaluation at this age. A brain MRI would be useful if multiple sclerosis, stroke or mass were at the top of the differential, however, there is nothing in this scenario suggestive of that. The scenario describes a torticollis-like scenario which the patient using to compensate for poor vision in one eye. Torticollis is a soft tissue disorder which is not diagnosed with x-ray. A Dix-Hallpike test is used to evaluate for benign positional vertigo which would be a very unlikely diagnosis at this age and with this presentation. An electroencephalogram (EEG) is used to diagnose seizures and the movements described seem intentional.
Amblyopia it the term used to describe poor visual acuity due to abnormal development of structures involved in sight during infancy or early childhood. If developmental abnormalities are present, the resulting reduction in stimulation of the visual pathways further leads to a decline in the development of the visual cortex, advancing amblyopia. It is sometimes referred to as having a “lazy eye.” Risk factors associated with amblyopia include lid ptosis, preterm delivery (<30 weeks), low birth weight (<3 lb, 5 oz), cerebral palsy, Down syndrome), or a family history of amblyopia or strabismus.
Amblyopia is most frequently unilateral but can be bilateral. Due to patient age and associated immaturity, the history is very limited, therefore, asking the parent about specific irregularities during routine exams is the key to early diagnosis and preservation of vision. Ask about noticeable squinting or wandering of one eye which may indicate exotropia. Torticollis (head tilting to the side) may indicate an alignment problem while intentional neck extension may be compensating for a ptosis. Intentionally turning, as opposed to tilting, of the head may be a way to compensate for nystagmus. Physical exam findings can range from normal to visible opacifications of the cornea or lens, lid ptosis, or noticeable malalignment.
Because adult level visual acuity is usually achieved by 3-5 years of age, early recognition and referral is crucial in preventing or minimizing vision loss. Initial screening should occur during the newborn period and then again around 3-5 years of age or sooner if parental concerns occur. Newborn screening should include assessing the general appearance of the eyes and lids, pupil reactivity, ocular mobility, presence or absence of a red reflex, and gross ocular malalignment (tropia or strabismus) by assessing corneal light reflection. In a cooperative child under the age of 3, a cover-uncover test is used to evaluate for a phoria which is noticed when both eyes are not allowed to fixate on an object at the same time. After the age of 3 a visual acuity exam should be performed using an age-based eye chart. Treatment of amblyopia should start as soon as it is recognized and a specific cause is determined. Surgical removal of a congenital cataract or strabismus surgery may be necessary. Patching of the unaffected eye for 2 hours per day can be very effective. Alternatively, if a patch is not well tolerated, using atropine 1% drops on 2 consecutive days per week in the "normal" eye will cause pupil dilation resulting in blurring which forces the amblyopic eye to work harder at focusing on near distances. Correcting lens refractive errors with glasses should be done when warranted.
References:
1. https://www.aafp.org/afp/2019/1215/p745.html
2. UpToDate -
Question 26 of 56
26. Question
A 70-year-old female presents with peripheral vision loss. Which of the following is most likely to be found if the patient’s symptoms are the result of open-angle glaucoma?
Correct
The correct answer is E. Intraocular pressure can be elevated but up to 50% of cases of open-angle glaucoma will have normal pressures. A cloudy cornea occurs in many cases of angle-closure glaucoma. Cotton wool spots are associated with hypertension and diabetes, not with glaucoma. Open-angle glaucoma is usually painless, as opposed to angle-closure. Glaucoma is the leading cause of blindness worldwide.
Open-angle glaucoma is more specifically related to an optic neuropathy than elevated intraocular pressure. The disorder results in progressive peripheral visual field loss with subsequent central field loss. Risk factors include advancing age, African descent, family history, myopia, diabetes, hypertension, and previously elevated intraocular pressure. Disease progression is slow and most cases are diagnosed incidentally when a comprehensive eye exam is performed. The physical exam should focus on the funduscopic exam, specifically the appearance of the disc and the cup to disc ratio. The disc is the end of the optic nerve with the denser central portion called the cup. The normal cup is approximately 1/3rd the size of the disc and both are round or slightly oval in shape. The normal disc has a pink-orange color with a slightly more pale, yellow center representing the cup. With advancing glaucoma, the cup asymmetrically enlarges, and becomes a more pale, white and the disc appears “hollowed-out”, known as “cupping”. When the cup exceeds 50% of the diameter of the disc, glaucoma is present. This process also causes the vessels to narrow as they cross over the cup. Notching, or an abrupt ending to a vessel also develops as the cup enlarges.
Screening with routine measurement of intraocular pressures is not recommended as many patients will have normal pressures making it an unreliable test and possibly leading to a false sense of security for the patient and provider. It is advised that African Americans, Hispanics, or those with diabetes, myopia or a family history of glaucoma should have a comprehensive exam every 1-3 years starting at or before age 40. Others without risk factors should be evaluated at least once before 40 and then every 2-3 years after.
The goal of treatment is to prevent continued vision loss. All patients with open-angle glaucoma should receive medication to lower the initial pressure by 25-30%, even if the initial pressure was withing normal limits. Topical prostaglandins such as latanoprost or bimatoprost are generally considered first-line, however, cost or side effects may limit use. Topical beta blockers such as timolol or betaxolol are also reasonable choices. Laser therapy and surgery are also options. Patients with open-angle glaucoma should avoid glucocorticoids in any form.
References:
A. https://www.ncbi.nlm.nih.gov/books/NBK441887/
B. UpToDate
C. https://www.reviewofophthalmology.com/article/how-to-evaluate-the-suspicious-optic-discIncorrect
The correct answer is E. Intraocular pressure can be elevated but up to 50% of cases of open-angle glaucoma will have normal pressures. A cloudy cornea occurs in many cases of angle-closure glaucoma. Cotton wool spots are associated with hypertension and diabetes, not with glaucoma. Open-angle glaucoma is usually painless, as opposed to angle-closure. Glaucoma is the leading cause of blindness worldwide.
Open-angle glaucoma is more specifically related to an optic neuropathy than elevated intraocular pressure. The disorder results in progressive peripheral visual field loss with subsequent central field loss. Risk factors include advancing age, African descent, family history, myopia, diabetes, hypertension, and previously elevated intraocular pressure. Disease progression is slow and most cases are diagnosed incidentally when a comprehensive eye exam is performed. The physical exam should focus on the funduscopic exam, specifically the appearance of the disc and the cup to disc ratio. The disc is the end of the optic nerve with the denser central portion called the cup. The normal cup is approximately 1/3rd the size of the disc and both are round or slightly oval in shape. The normal disc has a pink-orange color with a slightly more pale, yellow center representing the cup. With advancing glaucoma, the cup asymmetrically enlarges, and becomes a more pale, white and the disc appears “hollowed-out”, known as “cupping”. When the cup exceeds 50% of the diameter of the disc, glaucoma is present. This process also causes the vessels to narrow as they cross over the cup. Notching, or an abrupt ending to a vessel also develops as the cup enlarges.
Screening with routine measurement of intraocular pressures is not recommended as many patients will have normal pressures making it an unreliable test and possibly leading to a false sense of security for the patient and provider. It is advised that African Americans, Hispanics, or those with diabetes, myopia or a family history of glaucoma should have a comprehensive exam every 1-3 years starting at or before age 40. Others without risk factors should be evaluated at least once before 40 and then every 2-3 years after.
The goal of treatment is to prevent continued vision loss. All patients with open-angle glaucoma should receive medication to lower the initial pressure by 25-30%, even if the initial pressure was withing normal limits. Topical prostaglandins such as latanoprost or bimatoprost are generally considered first-line, however, cost or side effects may limit use. Topical beta blockers such as timolol or betaxolol are also reasonable choices. Laser therapy and surgery are also options. Patients with open-angle glaucoma should avoid glucocorticoids in any form.
References:
A. https://www.ncbi.nlm.nih.gov/books/NBK441887/
B. UpToDate
C. https://www.reviewofophthalmology.com/article/how-to-evaluate-the-suspicious-optic-disc -
Question 27 of 56
27. Question
A 60-year-old woman presents with a complaint of right eye pain and seeing “halos around lights”. Red reflex is intact. What is the most likely diagnosis?
Correct
The correct answer is A. The two most common causes of a “halo around lights” complaint are cataract and acute angle-closure glaucoma. Since there is a pain component and the red reflex is intact, a cataract is unlikely. Macular degeneration typically causes a complaint of straight-line distortion or waviness and gradual central visual field loss while maintaining peripheral visual fields. Open-angle glaucoma typically results in progressive vision loss from the periphery to the center and is painless. Retinal detachment is also painless and causes flashes of light or “curtain” or shadow covering part of the visual field.
Angle-closure glaucoma differs from open-angle in that it is more often acute, painful, and always associated with elevated intraocular pressure. The normal physiology of the eye is that the ciliary body produces aqueous humor which flows through the pupil and exits the eye at the anterior chamber angle. The normal process results in a relatively static, “normal” pressure. Angle-closure glaucoma can be divided into primary, chronic, or secondary types. Primary is associated with a lens which is positioned too anterior, causing it to rest against the iris and block the pupil which prevents normal flow and results in an increase in pressure behind the iris and ultimately, pushing the iris to cover the anterior chamber angle. If only a portion of the angle is blocked on multiple occasions, scaring of the angle occurs over time. Secondary-type is the result of outside forces pushing or pulling the iris or ciliary body to the point that it interferes with the functionality of the angle. Risk factors include age > 60, female > male, Asian descent, and family history.
Acute angle-closure is a fairly rapid process which results in pain which can frequently be associated nausea and/or vomiting, the “halos around lights” phenomenon, and blurred vision. Exam will show a patient in moderate to severe pain, conjunctival injection, edema or cloudy appearance of the cornea, shallow anterior chamber with tangential lighting, and/or a mid-dilated, fixed pupil. Most episodes occur in low light environments which causes pupil dilation and pushes the iris into the angle, therefore, patients with a history of angle-closure should be educated to avoid low light scenarios. When examining a patient with a high likelihood of acute-angle closure glaucoma, avoid instilling drops that dilate the pupil since this may exacerbate the condition.
The diagnosis is confirmed by obtaining an elevated intraocular pressure (normal = 10-21 mm Hg). An eye expert can do this with slit lamp gonioscopy or slit lamp grading of the anterior chamber depth. Ultrasound biomicroscopy or anterior segment optical coherence tomography are also used by experts. The diagnosis is usually made in the office, urgent care, or emergency department setting using a handheld tonometer. Types of tonometers include applanation, indentation, rebound, and dynamic contour. Accuracy of these devices depends on proper calibration, provider familiarity, avoidance of pressing on the globe with the hand not holding the device, and underlying patient anatomy (corneal thickness, elasticity, hysteresis).
Emergent ophthalmology consultation is warranted. Initial treatment usually involves instilling pressure-lowering drops with the goal of stopping production of aqueous humor (beta-blocker, alpha 2-agonist) and increasing the outflow through the angle (miotic, alpha 2-agonist). An example of a typical protocol would be 1 drop each 0.5% timolol maleate, 1% apraclonidine; and 2% pilocarpine administered approximately 1 minute apart. Intravenous acetazolamide 500 mg is also given, which also blocks production of aqueous humor. Pressure should be reassessed every hour until definitive treatment via laser or surgical iridectomy can be performed.
References:
A. https://www.ncbi.nlm.nih.gov/books/NBK430857/
B. UpToDate
C. https://www.ncbi.nlm.nih.gov/books/NBK493225/Incorrect
The correct answer is A. The two most common causes of a “halo around lights” complaint are cataract and acute angle-closure glaucoma. Since there is a pain component and the red reflex is intact, a cataract is unlikely. Macular degeneration typically causes a complaint of straight-line distortion or waviness and gradual central visual field loss while maintaining peripheral visual fields. Open-angle glaucoma typically results in progressive vision loss from the periphery to the center and is painless. Retinal detachment is also painless and causes flashes of light or “curtain” or shadow covering part of the visual field.
Angle-closure glaucoma differs from open-angle in that it is more often acute, painful, and always associated with elevated intraocular pressure. The normal physiology of the eye is that the ciliary body produces aqueous humor which flows through the pupil and exits the eye at the anterior chamber angle. The normal process results in a relatively static, “normal” pressure. Angle-closure glaucoma can be divided into primary, chronic, or secondary types. Primary is associated with a lens which is positioned too anterior, causing it to rest against the iris and block the pupil which prevents normal flow and results in an increase in pressure behind the iris and ultimately, pushing the iris to cover the anterior chamber angle. If only a portion of the angle is blocked on multiple occasions, scaring of the angle occurs over time. Secondary-type is the result of outside forces pushing or pulling the iris or ciliary body to the point that it interferes with the functionality of the angle. Risk factors include age > 60, female > male, Asian descent, and family history.
Acute angle-closure is a fairly rapid process which results in pain which can frequently be associated nausea and/or vomiting, the “halos around lights” phenomenon, and blurred vision. Exam will show a patient in moderate to severe pain, conjunctival injection, edema or cloudy appearance of the cornea, shallow anterior chamber with tangential lighting, and/or a mid-dilated, fixed pupil. Most episodes occur in low light environments which causes pupil dilation and pushes the iris into the angle, therefore, patients with a history of angle-closure should be educated to avoid low light scenarios. When examining a patient with a high likelihood of acute-angle closure glaucoma, avoid instilling drops that dilate the pupil since this may exacerbate the condition.
The diagnosis is confirmed by obtaining an elevated intraocular pressure (normal = 10-21 mm Hg). An eye expert can do this with slit lamp gonioscopy or slit lamp grading of the anterior chamber depth. Ultrasound biomicroscopy or anterior segment optical coherence tomography are also used by experts. The diagnosis is usually made in the office, urgent care, or emergency department setting using a handheld tonometer. Types of tonometers include applanation, indentation, rebound, and dynamic contour. Accuracy of these devices depends on proper calibration, provider familiarity, avoidance of pressing on the globe with the hand not holding the device, and underlying patient anatomy (corneal thickness, elasticity, hysteresis).
Emergent ophthalmology consultation is warranted. Initial treatment usually involves instilling pressure-lowering drops with the goal of stopping production of aqueous humor (beta-blocker, alpha 2-agonist) and increasing the outflow through the angle (miotic, alpha 2-agonist). An example of a typical protocol would be 1 drop each 0.5% timolol maleate, 1% apraclonidine; and 2% pilocarpine administered approximately 1 minute apart. Intravenous acetazolamide 500 mg is also given, which also blocks production of aqueous humor. Pressure should be reassessed every hour until definitive treatment via laser or surgical iridectomy can be performed.
References:
A. https://www.ncbi.nlm.nih.gov/books/NBK430857/
B. UpToDate
C. https://www.ncbi.nlm.nih.gov/books/NBK493225/ -
Question 28 of 56
28. Question
A patient presents with a gradually worsening, painful, red eye for the past 3 days. The globe is tender to palpation over the red area (see photo above). The patient never sees a PCP and has no medical diagnoses. Which underlying disease is the patient most likely to have?Correct
The correct answer is D. All of these disorders can result in different types of eye findings, but rheumatoid arthritis is most commonly associated with scleritis, which is the most likely diagnosis given the information provided. Ankylosing spondylitis can cause anterior uveitis, cataracts, and increased intraocular pressure.
Neurofibromatosis causes yellow deposits with the iris, called Lisch nodules, in 95% of patients with the disease. Osteoarthritis is associated with macular degeneration. Scleroderma can affect the eyelids and lashes as well as thinning of the iris, and exudates or hemorrhages of the retina.The sclera lies just inferior to the conjunctiva and episclera and extends from the limbus to the optic canal. The exact pathophysiology of why this layer becomes inflamed is poorly understood but is highly associated with systemic inflammatory and autoimmune vasculitis disorders. The vast majority of cases effect the anterior portion of the sclera and are subdivided into diffuse, nodular, and necrotizing. A small percentage are isolated to the posterior portion. Other disorders associated with scleritis include reactive arthritis, lupus, polyarteritis nodosa, syphilis, lyme, zoster, sarcoidosis, and inflammatory bowel disease, amongst others.
The hallmark of episcleritis is redness and injections of the sclera vessels associated with a constant, moderate-severe boring pain which progresses over several days. The pain may increase at night, in the early morning, or with extraocular movements. Other symptoms can include headache, tearing, and photophobia. The primary findings on physical exam are injection and edema of the sclera along with reproducible tenderness to palpation of the globe over a closed eyelid. Isolated cases of posterior scleritis will have noticeable inflammation only at the extremes of gaze. Other potential findings include retinal detachment, optic disc edema, posterior uveitis, or glaucoma.
Scleritis is a clinical diagnosis, however, if not previously diagnosed, a workup for possible systemic inflammatory and vasculitis disorders. Initial treatment is usually with oral NSAIDS until resolution of symptoms. If unsuccessful, oral glucocorticoids with a taper should be given over 4-6 weeks. Immunosuppressants can also be considered, including rituximab, cyclophosphamide, and cyclosporine. Scleritis has the potential to cause permanent vision loss if not recognized and treated early. Glaucoma and cataracts can also occur.Episcleritis is an inflammation of the episcleral and conjunctival layers. It is usually more bright red injection of vessels than the bluish-violet appearance of scleritis, but they can look very similar. Instilling phenylephrine 2.5% or 10% drops should cause episcleritis to blanch. If blanching occurs, episcleritis is diagnosed. If there is no change in the redness and injection, scleritis is likely. Unlike scleritis, episcleritis is usually nonpainful or minimally painful.
References:
1. UpToDate
2. https://www.ncbi.nlm.nih.gov/books/NBK499944/
3. https://link.springer.com/article/10.1007/s10792-019-01183-9
4. https://www.reviewofoptometry.com/article/ro1117-a-red-eye-scleritis-or-episcleritisIncorrect
The correct answer is D. All of these disorders can result in different types of eye findings, but rheumatoid arthritis is most commonly associated with scleritis, which is the most likely diagnosis given the information provided. Ankylosing spondylitis can cause anterior uveitis, cataracts, and increased intraocular pressure.
Neurofibromatosis causes yellow deposits with the iris, called Lisch nodules, in 95% of patients with the disease. Osteoarthritis is associated with macular degeneration. Scleroderma can affect the eyelids and lashes as well as thinning of the iris, and exudates or hemorrhages of the retina.The sclera lies just inferior to the conjunctiva and episclera and extends from the limbus to the optic canal. The exact pathophysiology of why this layer becomes inflamed is poorly understood but is highly associated with systemic inflammatory and autoimmune vasculitis disorders. The vast majority of cases effect the anterior portion of the sclera and are subdivided into diffuse, nodular, and necrotizing. A small percentage are isolated to the posterior portion. Other disorders associated with scleritis include reactive arthritis, lupus, polyarteritis nodosa, syphilis, lyme, zoster, sarcoidosis, and inflammatory bowel disease, amongst others.
The hallmark of episcleritis is redness and injections of the sclera vessels associated with a constant, moderate-severe boring pain which progresses over several days. The pain may increase at night, in the early morning, or with extraocular movements. Other symptoms can include headache, tearing, and photophobia. The primary findings on physical exam are injection and edema of the sclera along with reproducible tenderness to palpation of the globe over a closed eyelid. Isolated cases of posterior scleritis will have noticeable inflammation only at the extremes of gaze. Other potential findings include retinal detachment, optic disc edema, posterior uveitis, or glaucoma.
Scleritis is a clinical diagnosis, however, if not previously diagnosed, a workup for possible systemic inflammatory and vasculitis disorders. Initial treatment is usually with oral NSAIDS until resolution of symptoms. If unsuccessful, oral glucocorticoids with a taper should be given over 4-6 weeks. Immunosuppressants can also be considered, including rituximab, cyclophosphamide, and cyclosporine. Scleritis has the potential to cause permanent vision loss if not recognized and treated early. Glaucoma and cataracts can also occur.Episcleritis is an inflammation of the episcleral and conjunctival layers. It is usually more bright red injection of vessels than the bluish-violet appearance of scleritis, but they can look very similar. Instilling phenylephrine 2.5% or 10% drops should cause episcleritis to blanch. If blanching occurs, episcleritis is diagnosed. If there is no change in the redness and injection, scleritis is likely. Unlike scleritis, episcleritis is usually nonpainful or minimally painful.
References:
1. UpToDate
2. https://www.ncbi.nlm.nih.gov/books/NBK499944/
3. https://link.springer.com/article/10.1007/s10792-019-01183-9
4. https://www.reviewofoptometry.com/article/ro1117-a-red-eye-scleritis-or-episcleritis -
Question 29 of 56
29. Question
The above image is most consistent with which terminology?
Correct
The correct answer is B. The first four terms are all used in the description of strabismus. Esotropia refers to an eye turned nasally. Exotropia is an outward splay. Hypertropia is when an eye is positioned superiorly while the patient is looking straight forward. Hypotropia is a inferior displacement. While this eye does appear to be looking up as well, it seems to be centrally located between the upper and lower lids. Nystagmus is used when a rhythmic oscillation of an eye(s) occur, usually exacerbated when testing extraocular movements. This is unable to be determined using a still photo.
Strabismus is a malalignment of an eye or eyes which is usually the result of one or more of the six ocular muscles being out of balance with the others. This is usually due to an underlying issue with the muscle or cranial nerve III, IV, or VI. A congenital malformation of the globe, mass or prior trauma can also give the appearance of strabismus. Risk factors include preterm birth, compensation for an underlying vision defect, retinoblastoma, cerebral palsy, and various types of muscular dystrophy. The majority of cases occur during childhood making early recognition and intervention important to preserve normal vision. General inspection screenings should begin at birth with more specific physical exam or instrument testing starting around 6 months or when the child is able to cooperate. Followed by yearly assessments until age 5 and then every 1-2 years thereafter.
There are several physical exam tests that are used in evaluating for strabismus. The corneal light reflex test (Hirschberg) is performed by having the patient stare at an object held directly in line with their nose and shining a light from the object toward the patient. Normal alignment is indicated by a small, white light reflecting back from the central portion of each eye. In the attached image, the left eye reflection is at approximately 8 o’clock over the iris instead of the pupil. The cover and cover-uncover tests are also utilized. When performing the cover test, the provider asks the patient to stare at a stationary object and places a card over one eye while watching for any repositioning of the opposite eye. Repositioning is abnormal. The test is performed bilaterally. The cover-uncover test is performed in the same manner with the provider observing the covered eye as the cover is removed. Any repositioning is abnormal. Photoscreeners and autorefractors are examples of instruments that can be used by the trained expert to evaluate for strabismus as well as other things like vision loss, astigmatism, etc. Patients with less than 6 degrees of strabismus can be closely followed. Others will usually require intervention with treatment options including vision correction, ocular exercises, patching the “good” eye in patients with amblyopia, miotic drop therapy, Botox injections, or surgery. Prognosis is good if detected early.
References:
1. UpToDate
2. https://www.ncbi.nlm.nih.gov/pmc/articles/PMC6317790/
3. https://www.aao.org/eye-health/tips-prevention/children-eye-screening
4. https://www.ncbi.nlm.nih.gov/books/NBK560782/Incorrect
The correct answer is B. The first four terms are all used in the description of strabismus. Esotropia refers to an eye turned nasally. Exotropia is an outward splay. Hypertropia is when an eye is positioned superiorly while the patient is looking straight forward. Hypotropia is a inferior displacement. While this eye does appear to be looking up as well, it seems to be centrally located between the upper and lower lids. Nystagmus is used when a rhythmic oscillation of an eye(s) occur, usually exacerbated when testing extraocular movements. This is unable to be determined using a still photo.
Strabismus is a malalignment of an eye or eyes which is usually the result of one or more of the six ocular muscles being out of balance with the others. This is usually due to an underlying issue with the muscle or cranial nerve III, IV, or VI. A congenital malformation of the globe, mass or prior trauma can also give the appearance of strabismus. Risk factors include preterm birth, compensation for an underlying vision defect, retinoblastoma, cerebral palsy, and various types of muscular dystrophy. The majority of cases occur during childhood making early recognition and intervention important to preserve normal vision. General inspection screenings should begin at birth with more specific physical exam or instrument testing starting around 6 months or when the child is able to cooperate. Followed by yearly assessments until age 5 and then every 1-2 years thereafter.
There are several physical exam tests that are used in evaluating for strabismus. The corneal light reflex test (Hirschberg) is performed by having the patient stare at an object held directly in line with their nose and shining a light from the object toward the patient. Normal alignment is indicated by a small, white light reflecting back from the central portion of each eye. In the attached image, the left eye reflection is at approximately 8 o’clock over the iris instead of the pupil. The cover and cover-uncover tests are also utilized. When performing the cover test, the provider asks the patient to stare at a stationary object and places a card over one eye while watching for any repositioning of the opposite eye. Repositioning is abnormal. The test is performed bilaterally. The cover-uncover test is performed in the same manner with the provider observing the covered eye as the cover is removed. Any repositioning is abnormal. Photoscreeners and autorefractors are examples of instruments that can be used by the trained expert to evaluate for strabismus as well as other things like vision loss, astigmatism, etc. Patients with less than 6 degrees of strabismus can be closely followed. Others will usually require intervention with treatment options including vision correction, ocular exercises, patching the “good” eye in patients with amblyopia, miotic drop therapy, Botox injections, or surgery. Prognosis is good if detected early.
References:
1. UpToDate
2. https://www.ncbi.nlm.nih.gov/pmc/articles/PMC6317790/
3. https://www.aao.org/eye-health/tips-prevention/children-eye-screening
4. https://www.ncbi.nlm.nih.gov/books/NBK560782/ -
Question 30 of 56
30. Question
Which of the following is accurate with regards to a diagnosis of retinal vein occlusion?
Correct
The correct answer is D. All other options are associated with retinal artery occlusions. A retinal vein occlusion is usually the result of pinching where an artery and vein cross over each other or a thrombus formation due to typical risk factors associated with cardiovascular disease or an underlying hypercoagulable state. Retinal vein occlusions present with painless blurred vision progressing over hours to weeks, eventually resulting in blindness as opposed to a more abrupt vision loss from a retinal artery occlusion.
Funduscopic exam typically reveals hemorrhages, retinal or optic disc edema, dilation of retinal vein(s), and cotton wool spots. Retinal hemorrhage can be localized in a wedge-shape or diffuse in all quadrants depending on whether a branch or central vein is involved. Central vein involvement has been classically described as a “blood and thunder” fundus. Fluorescence angiography is the definitive modality for diagnosing the disorder.
First-line treatment is with intraocular injections of a vascular endothelial growth factor inhibitor which generally continues for over 1 year. Second-line treatment is with intraocular glucocorticoid injection. Laser photocoagulation therapy is utilized when neovascular complications have developed. Complications of retinal vein occlusion include macular edema, neovascularization, and possible vitreous hemorrhage. Patients with retinal vein occlusion have higher incidents of stroke and cardiovascular disease.
References:
1. UpToDate
2. https://www.consultant360.com/article/consultant360/retinal-vein-occlusions-diagnosis-and-managementIncorrect
The correct answer is D. All other options are associated with retinal artery occlusions. A retinal vein occlusion is usually the result of pinching where an artery and vein cross over each other or a thrombus formation due to typical risk factors associated with cardiovascular disease or an underlying hypercoagulable state. Retinal vein occlusions present with painless blurred vision progressing over hours to weeks, eventually resulting in blindness as opposed to a more abrupt vision loss from a retinal artery occlusion.
Funduscopic exam typically reveals hemorrhages, retinal or optic disc edema, dilation of retinal vein(s), and cotton wool spots. Retinal hemorrhage can be localized in a wedge-shape or diffuse in all quadrants depending on whether a branch or central vein is involved. Central vein involvement has been classically described as a “blood and thunder” fundus. Fluorescence angiography is the definitive modality for diagnosing the disorder.
First-line treatment is with intraocular injections of a vascular endothelial growth factor inhibitor which generally continues for over 1 year. Second-line treatment is with intraocular glucocorticoid injection. Laser photocoagulation therapy is utilized when neovascular complications have developed. Complications of retinal vein occlusion include macular edema, neovascularization, and possible vitreous hemorrhage. Patients with retinal vein occlusion have higher incidents of stroke and cardiovascular disease.
References:
1. UpToDate
2. https://www.consultant360.com/article/consultant360/retinal-vein-occlusions-diagnosis-and-management -
Question 31 of 56
31. Question
A 34 year-old woman presents to urgent care with a sore throat and fatigue for four days. She is also complains of back pain, chills, and fever. She denies any sick contacts but did admit to having a completed miscarriage 6 days prior and thought her symptoms may be a result from that. She denies abdominal pain or pelvic pain, cough, vomiting, rash, or dysuria. On physical exam, she is dressed in layers with rigors. Temp 104 ˚F, orally. She has a mildly erythematous pharynx with exudate and tonsillar hypertrophy. She has tender bilateral anterior lymphadenopathy without nuchal rigidity. Lung sounds and cardiac exam normal. Abdomen and mid thoracic back mildly tender to palpation without organomegaly. The remainder of the exam is unremarkable. Based on the information provided, which of the following diagnostic tests would be the most appropriate next step?
Correct
The correct answer is C. This patient is presenting with the typical symptoms of streptococcal pharyngitis. Although the most common cause of pharyngitis is viral, this patient has a bacterial infection. This is most commonly caused by Group A Beta Hemolytic Streptococcus.1 The Centor Criteria for strep throat consists of: Fever (>100.4 degrees Fahrenheit), tender anterior cervical lymphadenopathy, absence of a cough, and pharyngotonsillar exudate.1 Each symptom is worth 1 point. Any patient with 2-3 points should get a throat culture.1 This patient presents with 3 of the 4 symptoms (fever, anterior cervical lymphadenopathy, and no cough), giving her a score of 3 points and therefore needing a throat culture.
Choice A, a rapid influenza test via nasopharyngeal swab is incorrect. The common symptoms for influenza do include fever, myalgias, and chills but also a cough, nasal congestion, coryza, and headaches.2 This patient is not experiencing a majority of these symptoms, so therefore a rapid influenza test would not be the best choice.
Choice B, MonoSpot test followed by Epstein Barr titers is incorrect. Mononucleosis is a good option to keep in the differential, but a key symptom that would be a negative indication of mononucleosis is “anterior” cervical lymphadenopathy. Most patients with mononucleosis will have posterior cervical lymphadenopathy.1 Although patients with mononucleosis will have a sore throat and fever, they will also complain of severe fatigue as well as abdominal discomfort. This patient denied these symptoms. The abdominal pain is due to the inflammation of the spleen and liver that occurs during the disease. Patients who have an enlarged spleen or liver should avoid contact sports and trauma for at least 1 month in order to avoid possible rupture.1 On physical exam, bilateral lid edema known as Hoagland sign and conjunctival hemorrhaging may be present as well.2
Choice D, do nothing because it’s viral, is not the correct answer. This patient has been experiencing high fevers for multiple days which would indicate more of a bacterial infection rather than viral. Most viral infections have lower grade fevers as opposed to bacterial infections where fevers tend to be higher.3 Also, in bacterial infections the fever will get worse a few days into the illness rather than getting better.3
Resources
- Williams D.A. PANCE Prep Pearls. CreateSpace. 2014.
- Papadakis M, McPhee S, Rabow M. Current Medical Diagnosis & Treatment 2015. 2015
- Duke University Health System. Is it a Bacterial Infection or Virus?. DukeMedicine. https://www.dukemedicine.org/blog/it-bacterial-infection-or-virus. Published Oct. 1, 2013. Accessed Jan. 25, 2016.
- Wald, E. R. Approach to diagnosis of acute infectious pharyngitis in children and adolescents. UpToDate. http://www.uptodate.com/contents/approach-to-diagnosis-of-acute-infectious-pharyngitis-in-children-and-adolescents?source=search_result. Published Aug. 11, 2015. Accessed Jan. 28, 2016.
- Marquez-Carrillo M.A. Bacterial Pharyngitis. Medscape. http://emedicine.medscape.com/article/225243-overview. Published Jul. 28, 2015. Accessed Feb. 1, 2016.
Incorrect
The correct answer is C. This patient is presenting with the typical symptoms of streptococcal pharyngitis. Although the most common cause of strep is viral, this patient has a bacterial infection. This is most commonly caused by Group A Beta Hemolytic Streptococcus.1 The Centor Criteria for strep throat consists of: Fever (>100.4 degrees Fahrenheit), tender anterior cervical lymphadenopathy, absence of a cough, and pharyngotonsillar exudate.1 Each symptom is worth 1 point. Any patient with 2-3 points should get a throat culture.1 This patient presents with 3 of the 4 symptoms (fever, anterior cervical lymphadenopathy, and no cough), giving her a score of 3 points and therefore needing a throat culture.
Choice A, a rapid influenza test via nasopharyngeal swab is incorrect. The common symptoms for influenza do include fever, myalgias, and chills but also a cough, nasal congestion, coryza, and headaches.2 This patient is not experiencing a majority of these symptoms, so therefore a rapid influenza test would not be the best choice.
Choice B, MonoSpot test followed by Epstein Barr titers is incorrect. Mononucleosis is a good option to keep in the differential, but a key symptom that would be a negative indication of mononucleosis is “anterior” cervical lymphadenopathy. Most patients with mononucleosis will have posterior cervical lymphadenopathy.1 Although patients with mononucleosis will have a sore throat and fever, they will also complain of severe fatigue as well as abdominal discomfort. This patient denied these symptoms. The abdominal pain is due to the inflammation of the spleen and liver that occurs during the disease. Patients who have an enlarged spleen or liver should avoid contact sports and trauma for at least 1 month in order to avoid possible rupture.1 On physical exam, bilateral lid edema known as Hoagland sign and conjunctival hemorrhaging may be present as well.2
Choice D, do nothing because it’s viral, is not the correct answer. This patient has been experiencing high fevers for multiple days which would indicate more of a bacterial infection rather than viral. Most viral infections have lower grade fevers as opposed to bacterial infections where fevers tend to be higher.3 Also, in bacterial infections the fever will get worse a few days into the illness rather than getting better.3
Resources
- Williams D.A. PANCE Prep Pearls. CreateSpace. 2014.
- Papadakis M, McPhee S, Rabow M. Current Medical Diagnosis & Treatment 2015. 2015
- Duke University Health System. Is it a Bacterial Infection or Virus?. DukeMedicine. https://www.dukemedicine.org/blog/it-bacterial-infection-or-virus. Published Oct. 1, 2013. Accessed Jan. 25, 2016.
- Wald, E. R. Approach to diagnosis of acute infectious pharyngitis in children and adolescents. UpToDate. http://www.uptodate.com/contents/approach-to-diagnosis-of-acute-infectious-pharyngitis-in-children-and-adolescents?source=search_result. Published Aug. 11, 2015. Accessed Jan. 28, 2016.
- Marquez-Carrillo M.A. Bacterial Pharyngitis. Medscape. http://emedicine.medscape.com/article/225243-overview. Published Jul. 28, 2015. Accessed Feb. 1, 2016.
-
Question 32 of 56
32. Question
A 59 year-old female comes to primary care office complaining of cold symptoms for the past three weeks. She has productive cough, purulent nasal drainage, post nasal drip, and maxillary sinus pressure. On exam, patient has temperature of 100.4F, has tenderness to palpation and percussion on the right maxillary sinus, erythematous nasal mucosa. Her heart and lungs clear on auscultation. The patient has been taking Dextromethorphan PRN for cough with temporary relief. She denies SOB, dysphagia, sore throat, rash, abdominal tenderness. Which of the following is the most likely diagnosis?
Correct
The correct answer is D, acute bacterial sinusitis. Chronic sinusitis is a distracter because chronic sinusitis is defined as symptoms lasting for 12 weeks or more. Here, the patient has symptoms for only three weeks. Initial management of acute sinusitis is to observe for 7 days and treat with antibiotics only if there is no clinical improvement and the patient is febrile. Option B, Acute viral sinusitis should have been resolved within 7-10 days if it was viral in etiology. In this case, patient symptoms are going on from past 3 weeks. Option C, allergic rhinitis would cause symptoms including rhinorrhea, nasal congestion, but would not consist of purulent nasal discharge, sinus pressure or tenderness, low grade fever, or productive cough.
In the United States, 1 out of every 7 adults have an episode of acute rhinosinusitis (ARS) each year and approximately 30 million individuals are diagnosed each year. It is more common in women than men. Highest incidence among adults is seen between ages 45y.o to 64yo. Risk factors include older age, smoking, allergies and asthma, dental disease, and immunodeficiency. Acute bacterial rhinosinusitis (ABRS) comprise of 0.5 – 2.0% of ARS episodes. The pathophysiology of acute rhinosinusitis is associated with three factors including obstruction of the ostia or sinus drainage pathways, ciliary impairment, and altered mucus quantity and quality. Allergic, non-allergic or viral insults can predispose the sinuses to obstruction and reduced ciliary function leading to inflammation and edema of the sinus and nasal mucosa. The inflammation and edema results in decreased ciliary clearance of sinus secretions and/ or obstruction of the ostium leading to retention of secretions, negative sinus pressure, and decrease in partial pressure of oxygen, thereby, allowing infection by bacteria.
Acute bacterial rhinosinusitis (ABRS) is most commonly associated with viral URI. Additionally, it can result as a complication of trauma, granulomatous diseases, dental infection, mechanical obstruction, allergy or non-allergic rhinitis, immunodeficiency, ciliary dysfunction, cystic fibrosis, and other factors that disrupt sinus drainage. Approximately 75% cases of ABRS comprise of Streptococcus pneumoniae, and Haemophilus influenza as the pathogens. Other pathogens include Moraxella catarrhalis, Pseudomonas aeruginosa, Staphylococcus aureus, Escherichia coli, Klebsiella pneumoniae. Acute sinusitis pathogens can also be very commonly viral and rarely fungal agents. Patient can have a history of recent URI that lasts more than 7 days or worsens after 7 days, in this case, consider acute sinusitis. ABRS has three common clinical presentations including onset with persistent, severe, or worsening symptoms (nasal drainage, cough). Symptoms of ABRS include sinus pressure, tenderness over the sinus areas, hyposmia, nasal congestion, thick and purulent nasal drainage, postnasal drip, cough, sore throat, fever, fatigue, maxillary dental pain, ear fullness. On physical exam, common findings include tenderness on palpation and percussion of the maxillary and/or frontal sinuses, purulent nasal secretions, mucosal and facial erythema, periorbital edema. Transillumination may show opacification of the maxillary or frontal sinuses.
ABRS is a clinical diagnosis and does not require any imaging or tests unless complications, alternate diagnosis or treatment failure is suspected. One of the following is required to diagnose a patient with ABRS: (1) symptoms or signs of acute rhinosinusitis are present 10 days or more beyond the onset of upper respiratory symptoms, 1 or (2) symptoms or signs of acute rhinosinusitis worsen within 10 days after an initial improvement. X-ray or CT-scan may be used to determine etiology or for confirmation of sinusitis. Sinus aspiration can be performed to determine etiology as well. For treatment or management, Antibiotics are the mainstay for treatment of ABRS, but watchful waiting can be considered for up to 7 days if follow-up is assured. Antibiotic after 7 days should be started if patient’s condition fails to improve. If a decision is made to treat ABRS with an antibiotic agent, prescribe amoxicillin with or without clavulanic acid as first-line therapy for 5 to 10 days for most adults according to current guidelines. Doxycycline or levofloxacin or moxifloxacin is recommended as alternative first line agents for patients with penicillin allergy. Other commonly used antibiotics for second line treatment include third-generation cephalosporins (eg, cefuroxime, cefpodoxime, cefdinir), macrolides (ie, clarithromycin), fluoroquinolones (eg, ciprofloxacin, levofloxacin, moxifloxacin), and clindamycin.3Additionally, recommendations include drinking plenty of fluids, rest, analgesics, topical intranasal steroids, and/or nasal saline irrigation for symptomatic relief of ABRS.References
Rosenfeld RM, Piccirillo JF, Chandrasekhar SS, et al. Clinical practice guideline (update): Adult sinusitis. Official Journal of American Academy of Otolaryngology—Head and Neck Surgery Foundation. 2015; 152(2S): S1–S39. doi: 10.1177/0194599815572097
JAMA. 2009; 301(17):1844. doi:10.1001/jama.301.17.1844.
Brook I. Acute sinusitis. Medscape website. http://emedicine.medscape.com/article/232670-overview. Updated July 29, 2015. Accessed February 4, 2016.
Hwang PH, Patel ZM. Acute sinusitis and rhinosinusitis in adults: Clinical manifestation and diagnosis. UpToDate website. http://www.uptodate.com/contents/acute-sinusitis-and-rhinosinusitis-in-adults-clinical-manifestations-and-diagnosis?source=search_result&search=acute+sinustis&selectedTitle=2~150. Updated January 22, 2016. Accessed February 2, 2016.
Incorrect
The correct answer is D, acute bacterial sinusitis. Chronic sinusitis is a distracter because chronic sinusitis is defined as symptoms lasting for 12 weeks or more. Here, the patient has symptoms for only three weeks. Initial management of acute sinusitis is to observe for 7 days and treat with antibiotics only if there is no clinical improvement and the patient is febrile. Option B, Acute viral sinusitis should have been resolved within 7-10 days if it was viral in etiology. In this case, patient symptoms are going on from past 3 weeks. Option C, allergic rhinitis would cause symptoms including rhinorrhea, nasal congestion, but would not consist of purulent nasal discharge, sinus pressure or tenderness, low grade fever, or productive cough.
In the United States, 1 out of every 7 adults have an episode of acute rhinosinusitis (ARS) each year and approximately 30 million individuals are diagnosed each year. It is more common in women than men. Highest incidence among adults is seen between ages 45y.o to 64yo. Risk factors include older age, smoking, allergies and asthma, dental disease, and immunodeficiency. Acute bacterial rhinosinusitis (ABRS) comprise of 0.5 – 2.0% of ARS episodes. The pathophysiology of acute rhinosinusitis is associated with three factors including obstruction of the ostia or sinus drainage pathways, ciliary impairment, and altered mucus quantity and quality. Allergic, non-allergic or viral insults can predispose the sinuses to obstruction and reduced ciliary function leading to inflammation and edema of the sinus and nasal mucosa. The inflammation and edema results in decreased ciliary clearance of sinus secretions and/ or obstruction of the ostium leading to retention of secretions, negative sinus pressure, and decrease in partial pressure of oxygen, thereby, allowing infection by bacteria.
Acute bacterial rhinosinusitis (ABRS) is most commonly associated with viral URI. Additionally, it can result as a complication of trauma, granulomatous diseases, dental infection, mechanical obstruction, allergy or non-allergic rhinitis, immunodeficiency, ciliary dysfunction, cystic fibrosis, and other factors that disrupt sinus drainage. Approximately 75% cases of ABRS comprise of Streptococcus pneumoniae, and Haemophilus influenza as the pathogens. Other pathogens include Moraxella catarrhalis, Pseudomonas aeruginosa, Staphylococcus aureus, Escherichia coli, Klebsiella pneumoniae. Acute sinusitis pathogens can also be very commonly viral and rarely fungal agents. Patient can have a history of recent URI that lasts more than 7 days or worsens after 7 days, in this case, consider acute sinusitis. ABRS has three common clinical presentations including onset with persistent, severe, or worsening symptoms (nasal drainage, cough). Symptoms of ABRS include sinus pressure, tenderness over the sinus areas, hyposmia, nasal congestion, thick and purulent nasal drainage, postnasal drip, cough, sore throat, fever, fatigue, maxillary dental pain, ear fullness. On physical exam, common findings include tenderness on palpation and percussion of the maxillary and/or frontal sinuses, purulent nasal secretions, mucosal and facial erythema, periorbital edema. Transillumination may show opacification of the maxillary or frontal sinuses.
ABRS is a clinical diagnosis and does not require any imaging or tests unless complications, alternate diagnosis or treatment failure is suspected. One of the following is required to diagnose a patient with ABRS: (1) symptoms or signs of acute rhinosinusitis are present 10 days or more beyond the onset of upper respiratory symptoms, 1 or (2) symptoms or signs of acute rhinosinusitis worsen within 10 days after an initial improvement. X-ray or CT-scan may be used to determine etiology or for confirmation of sinusitis. Sinus aspiration can be performed to determine etiology as well. For treatment or management, Antibiotics are the mainstay for treatment of ABRS, but watchful waiting can be considered for up to 7 days if follow-up is assured. Antibiotic after 7 days should be started if patient’s condition fails to improve. If a decision is made to treat ABRS with an antibiotic agent, prescribe amoxicillin with or without clavulanic acid as first-line therapy for 5 to 10 days for most adults according to current guidelines. Doxycycline or levofloxacin or moxifloxacin is recommended as alternative first line agents for patients with penicillin allergy. Other commonly used antibiotics for second line treatment include third-generation cephalosporins (eg, cefuroxime, cefpodoxime, cefdinir), macrolides (ie, clarithromycin), fluoroquinolones (eg, ciprofloxacin, levofloxacin, moxifloxacin), and clindamycin.3Additionally, recommendations include drinking plenty of fluids, rest, analgesics, topical intranasal steroids, and/or nasal saline irrigation for symptomatic relief of ABRS.References
Rosenfeld RM, Piccirillo JF, Chandrasekhar SS, et al. Clinical practice guideline (update): Adult sinusitis. Official Journal of American Academy of Otolaryngology—Head and Neck Surgery Foundation. 2015; 152(2S): S1–S39. doi: 10.1177/0194599815572097
JAMA. 2009; 301(17):1844. doi:10.1001/jama.301.17.1844.
Brook I. Acute sinusitis. Medscape website. http://emedicine.medscape.com/article/232670-overview. Updated July 29, 2015. Accessed February 4, 2016.
Hwang PH, Patel ZM. Acute sinusitis and rhinosinusitis in adults: Clinical manifestation and diagnosis. UpToDate website. http://www.uptodate.com/contents/acute-sinusitis-and-rhinosinusitis-in-adults-clinical-manifestations-and-diagnosis?source=search_result&search=acute+sinustis&selectedTitle=2~150. Updated January 22, 2016. Accessed February 2, 2016.
-
Question 33 of 56
33. Question
A 68-year-old female presents with difficulty swallowing that has gotten progressively worse over the last couple of months. She feels like food gets stuck in her throat when eating. She also gets frequent heartburn. The patient notices the most difficulty when she eats meats. She does not struggle to swallow liquids. The patient has a history of COPD for which she is on a steroid inhaler. She has no pertinent physical exam findings. Which diagnosis is the most likely cause of the patient’s dysphagia?
Correct
The correct answer is C. Schatzki/esophageal ring. Esophageal ring is the correct diagnosis based on the patient’s presentation and lack of physical exam findings. The patient has intermittent dysphagia to solid foods only and exhibits “steakhouse syndrome” as she has the most dysphagia when eating steak and bread with others.4 The patient has uncontrolled heartburn and reflux, which has been linked to the development of Schatzki rings. An esophageal motility disorder would be incorrect in this scenario because the patient does not have dysphagia to liquids. With motility disorders, the patient will have difficulty swallowing both liquid and solid foods. Candidiasis is possible in individuals who take inhaled steroids and can cause dysphagia and chest discomfort. Despite this, Candida esophagitis is not correct in this scenario. The patient had no evidence of candidiasis on physical exam. There were no fluffy white patches in her mouth or oropharynx.4 Lastly, GERD is not the correct answer. The patient exhibits symptoms of GERD, but it is unlikely that it is the primary reason for the patient’s dysphagia. GERD has been linked to the development of esophageal strictures, but in this case it would not be the most correct diagnosis.
References
1.Kafrouni M. Schatzki’s Ring. Memorial Hermann. http://www.memorialhermann.org/digestive/schatzkis-ring/. Published July 8, 2015. Accessed February 6, 2017.
2.Liu JJ, Kahrilas PJ. Pharyngeal and esophageal diverticula, rings, and webs. GI Motility Online. May 2006. doi:10.1038/gimo41.
3.Lundell L. Reflux esophagitis and peptic strictures. GI Motility Online. May 2006. doi:10.1038/gimo43.
4.Schatzki Ring. Schatzki Ring: Background, Pathophysiology, Epidemiology. http://emedicine.medscape.com/article/182647-overview. Accessed February 1, 2017.
5.Smith MS. Diagnosis and Management of Esophageal Rings and Webs. Gastroenterology & Hepatology. https://www.ncbi.nlm.nih.gov/pmc/articles/PMC3033540/. Published November 2010. Accessed February 5, 2017.Incorrect
The correct answer is C. Schatzki/esophageal ring. Esophageal ring is the correct diagnosis based on the patient’s presentation and lack of physical exam findings. The patient has intermittent dysphagia to solid foods only and exhibits “steakhouse syndrome” as she has the most dysphagia when eating steak and bread with others.4 The patient has uncontrolled heartburn and reflux, which has been linked to the development of Schatzki rings. An esophageal motility disorder would be incorrect in this scenario because the patient does not have dysphagia to liquids. With motility disorders, the patient will have difficulty swallowing both liquid and solid foods. Candidiasis is possible in individuals who take inhaled steroids and can cause dysphagia and chest discomfort. Despite this, Candida esophagitis is not correct in this scenario. The patient had no evidence of candidiasis on physical exam. There were no fluffy white patches in her mouth or oropharynx.4 Lastly, GERD is not the correct answer. The patient exhibits symptoms of GERD, but it is unlikely that it is the primary reason for the patient’s dysphagia. GERD has been linked to the development of esophageal strictures, but in this case it would not be the most correct diagnosis.
References
1.Kafrouni M. Schatzki’s Ring. Memorial Hermann. http://www.memorialhermann.org/digestive/schatzkis-ring/. Published July 8, 2015. Accessed February 6, 2017.
2.Liu JJ, Kahrilas PJ. Pharyngeal and esophageal diverticula, rings, and webs. GI Motility Online. May 2006. doi:10.1038/gimo41.
3.Lundell L. Reflux esophagitis and peptic strictures. GI Motility Online. May 2006. doi:10.1038/gimo43.
4.Schatzki Ring. Schatzki Ring: Background, Pathophysiology, Epidemiology. http://emedicine.medscape.com/article/182647-overview. Accessed February 1, 2017.
5.Smith MS. Diagnosis and Management of Esophageal Rings and Webs. Gastroenterology & Hepatology. https://www.ncbi.nlm.nih.gov/pmc/articles/PMC3033540/. Published November 2010. Accessed February 5, 2017. -
Question 34 of 56
34. Question
A 34 year old woman presents to her primary care complaining of episodic hearing loss and vertigo for the past year. Once every couple of weeks she gets “dizzy spells” where she needs to lie down for about an hour until they pass. She thought they were related to her migraine headaches but she hasn’t had a migraine in many months and still has been experiencing the above symptoms. She also states her right ear sometimes feels full and she often has to turn her head to the left side to understand what people are saying. She has some nausea with the dizzy spells but denies any fever/chills, head trauma, or recent infection/illness. Her PCP sends her to get an audiogram which shows a low frequency hearing loss in her R ear with normal hearing in the left, as well as an MRI which shows no abnormalities. The provider also attempted to lay the patient on her back, head turned to the side and lower than the body but observed no signs of any nystagmus and it did not provoke symptoms. What is the most likely diagnosis?
Correct
Although all of the above answers can cause peripheral vertigo, the correct answer in the above vignette after eliminating the other choices is choice B, Meniere’s Disease. The classic presentation for this illness consists of episodic vertigo and hearing loss that tends to fluctuate (and often is low or high frequency loss), as well as tinnitus. Some data has also shown it can be more common in people who have a history of migraines.
Choice A (BPPV) would also cause episodic periods of vertigo, but it is not linked to any hearing loss. These periods of vertigo are also most commonly 10-20 seconds long, rarely as long as a minute. In addition, the provider performed a Dix-Hallpike Maneuver during the visit, which with BPPV would likely cause some fatigable nystagmus, of which the patient had none.
Choices C and D are diseases involving inflammation of the vestibular portion of eighth cranial nerve thought to be associated with viral illnesses. These would also cause dizziness, but in this case the symptoms of vertigo would be more constant instead of fluctuating. In addition, acoustic neuritis is not associated with hearing loss until the inflammation involves the bony labyrinth of the inner ear, when it becomes labyrinthitis.Choice E is unlikely as well, since the patient underwent an MRI which did not show any sign of mass suggesting an acoustic neuroma. Also, the hearing loss associated with an acoustic neuroma is irreversible and thus would not be coming and going as the above patient described.
Discussion:
Meniere’s Disease is a frustrating and complicated diagnosis, and while it may lack life-threatening qualities it can absolutely have a devastating effect on the patient’s quality of life. The typical patient presents with Meniere’s Disease in the 3rd-5th decade of life but people of all ages can be affected.1 Children with Meniere’s Disease will often have congenital problems, specifically malformations of the cochlea and vestibular organs of the inner ear. It affects as many as 150 out of 100,000 people in the general population, although it is difficult to definitively measure due to nonstandard and obscure reporting of cases.The inner ear is made up of not only the cochlea, which is responsible for interpreting air waves conducted through the external and middle ear and transmitting them to the brain, but also the vestibular organs which include the semicircular canals, the utricle, and the saccule. All of these together make up the bony labyrinth of the inner ear. These structures contain endolymph within them which facilitates the various vestibular and auditory functions in order to properly hear and balance. It has been demonstrated that patients who suffer from Meniere’s Disease display “endolymphatic hydrops,” or excess fluid build-up within these inner ear structures.1 Unfortunately the cause of this imbalance is not clearly understood. It is possible that there is some sort of blockage of the endolymphatic duct or narrowing of the vestibular aqueduct that causes the back-up, and other various immunologic, genetic, viral, and vascular etiologies hypotheses exist as well.1
The clinical triad associated with Meniere’s Disease is episodic vertigo, sensorineural hearing loss, and tinnitus.1 It is very important to distinguish true vertigo from other types of “dizziness” during a patient interview. When a patient reports dizziness, the true complaint can range from vertigo (a spinning sensation with central and peripheral causes), presyncope (associated with vascular causes), disequilibrium (musculoskeletal, neuropathies, or cerebellar causes), or even vague/nonspecific causes (associated with psychiatric disorders/anxiety).2 While aural fullness, nausea, and tinnitus may or may not be present with this condition, it is necessary for diagnosis to have episodic vertigo as well as hearing loss.1 This defines the condition from various differentials, such as BPPV which would have similar episodic vertigo without any hearing loss, and vestibular neuritis/labyrinthitis, which would have more continuous symptoms of vertigo. The hearing loss experienced usually comes and goes and is primarily a lower frequency loss that progresses over the course of the disease. The intensity of hearing loss and vertigo vary from patient to patient.It is almost impossible to measure the amount of endolymph within the inner ear without risking permanent damage, and imaging leaves a lot to be desired. Often it can only be truly assessed post-mortem, which is little help to the patient. Therefore, the diagnosis of Meniere’s Disease is almost always a clinical one. Often the first diagnostic test that will be done is an audiogram to assess the patient’s level of hearing loss and differentiate between a sensorineural or conductive hearing loss. Again, this is commonly a low frequency loss, or occasionally high frequency with normal mid-frequency hearing.1 Additionally vestibular testing can be done, such as an electronystagmogram. This test measures inner ear response to caloric stimulation and various movements which can provide insight towards the source of the vertigo. The ear is irrigated with warm air followed by cold while the patient lies down, and as the temperature change triggers movement of fluid in the semicircular canals, nystagmic movement of the eyes follows.3 A normal test would result in equal nystagmic response in each eye. With Meniere’s, often times the affected side will display a lesser response to testing, while in contrast some sort of inner ear lesion or irritation may result in an increased response.3 Labs and imaging studies such as an MRI for diagnosis of Meniere’s Disease are primarily used to rule out other possible causes, most importantly brain lesions that can cause similar symptoms of vertigo. Some controversial tests attempt to assess endolymphatic hydrops such as glycerine, urea, and sorbitol stress tests but are not widely used due to their low specificity and sensitivity.1
The treatment for Meniere’s usually doesn’t aim to cure the disease but rather to control the chronic symptoms associated with it. Hearing loss connected with this diagnosis does not have any treatment. However, the vertigo spells have several options to reduce severity and frequency. First line treatments for vertigo spells are anti-emetics (Zofran or Promethazine) to control nausea, and motion sickness medications (Valium or Antivert) to reduce spinning sensations. The most non-invasive option for long-term control is strict diet control. Often Meniere’s patients will be affected by certain dietary triggers, and it usually helps to restrict salt consumption as well as caffeine, alcohol, and nicotine, which reduce blood flow to the inner ear and also cause imbalances to fluid and electrolytes.1 Diuretics such as acetazolamide may also reduce the frequency of vertigo attacks.4 There are also several more “destructive” options to control refractory vertigo. These consist of injecting dexamethasone, a steroid, or gentamicin, an ototoxic antibiotic, into the inner ear. Gentamicin can be especially damaging to the cochlea and semicircular canals but may alleviate some chronic symptoms of Meniere’s. Most times it has the added complication of further reducing hearing ability. The most invasive options include surgical routes of endolymphatic sac procedures, vestibular nerve sections, or even a total labyrinthectomy, which would only be performed if the hearing loss in the affected ear was already completely lost.
References:
1. Moskowitz HS et al. Meniere Disease. UpToDate. https://www.uptodate.com/contents/meniere-disease?source=search_result&search=meniere%20disease&selectedTitle=1~29#H4. Updated 3/22/16. Accessed 4/22/17.
2. Branch WT et al. Approach to the patient with dizziness. UpToDate. https://www.uptodate.com/contents/approach-to-the-patient-with-dizziness?source=search_result&search=disequilibrium&selectedTitle=1~79#H7. Updated 10/20/14. Accessed 4/22/17.
3. Li JC. Meniere Disease (Idiopathic Endolymphatic Hydrops) Workup. UpToDate. http://emedicine.medscape.com/article/1159069-workup#c1. Updated 11/29/16. Accessed 4/22/17.
4. Mayo Clinic Staff. Meniere’s Disease – Treatment and drugs. Mayo Clinic. http://www.mayoclinic.org/diseases-conditions/menieres-disease/basics/treatment/con-20028251. Published 11/26/15. Accessed 4/22/17.Incorrect
Although all of the above answers can cause peripheral vertigo, the correct answer in the above vignette after eliminating the other choices is choice B, Meniere’s Disease. The classic presentation for this illness consists of episodic vertigo and hearing loss that tends to fluctuate (and often is low or high frequency loss), as well as tinnitus. Some data has also shown it can be more common in people who have a history of migraines.
Choice A (BPPV) would also cause episodic periods of vertigo, but it is not linked to any hearing loss. These periods of vertigo are also most commonly 10-20 seconds long, rarely as long as a minute. In addition, the provider performed a Dix-Hallpike Maneuver during the visit, which with BPPV would likely cause some fatigable nystagmus, of which the patient had none.
Choices C and D are diseases involving inflammation of the vestibular portion of eighth cranial nerve thought to be associated with viral illnesses. These would also cause dizziness, but in this case the symptoms of vertigo would be more constant instead of fluctuating. In addition, acoustic neuritis is not associated with hearing loss until the inflammation involves the bony labyrinth of the inner ear, when it becomes labyrinthitis.Choice E is unlikely as well, since the patient underwent an MRI which did not show any sign of mass suggesting an acoustic neuroma. Also, the hearing loss associated with an acoustic neuroma is irreversible and thus would not be coming and going as the above patient described.
Discussion:
Meniere’s Disease is a frustrating and complicated diagnosis, and while it may lack life-threatening qualities it can absolutely have a devastating effect on the patient’s quality of life. The typical patient presents with Meniere’s Disease in the 3rd-5th decade of life but people of all ages can be affected.1 Children with Meniere’s Disease will often have congenital problems, specifically malformations of the cochlea and vestibular organs of the inner ear. It affects as many as 150 out of 100,000 people in the general population, although it is difficult to definitively measure due to nonstandard and obscure reporting of cases.The inner ear is made up of not only the cochlea, which is responsible for interpreting air waves conducted through the external and middle ear and transmitting them to the brain, but also the vestibular organs which include the semicircular canals, the utricle, and the saccule. All of these together make up the bony labyrinth of the inner ear. These structures contain endolymph within them which facilitates the various vestibular and auditory functions in order to properly hear and balance. It has been demonstrated that patients who suffer from Meniere’s Disease display “endolymphatic hydrops,” or excess fluid build-up within these inner ear structures.1 Unfortunately the cause of this imbalance is not clearly understood. It is possible that there is some sort of blockage of the endolymphatic duct or narrowing of the vestibular aqueduct that causes the back-up, and other various immunologic, genetic, viral, and vascular etiologies hypotheses exist as well.1
The clinical triad associated with Meniere’s Disease is episodic vertigo, sensorineural hearing loss, and tinnitus.1 It is very important to distinguish true vertigo from other types of “dizziness” during a patient interview. When a patient reports dizziness, the true complaint can range from vertigo (a spinning sensation with central and peripheral causes), presyncope (associated with vascular causes), disequilibrium (musculoskeletal, neuropathies, or cerebellar causes), or even vague/nonspecific causes (associated with psychiatric disorders/anxiety).2 While aural fullness, nausea, and tinnitus may or may not be present with this condition, it is necessary for diagnosis to have episodic vertigo as well as hearing loss.1 This defines the condition from various differentials, such as BPPV which would have similar episodic vertigo without any hearing loss, and vestibular neuritis/labyrinthitis, which would have more continuous symptoms of vertigo. The hearing loss experienced usually comes and goes and is primarily a lower frequency loss that progresses over the course of the disease. The intensity of hearing loss and vertigo vary from patient to patient.It is almost impossible to measure the amount of endolymph within the inner ear without risking permanent damage, and imaging leaves a lot to be desired. Often it can only be truly assessed post-mortem, which is little help to the patient. Therefore, the diagnosis of Meniere’s Disease is almost always a clinical one. Often the first diagnostic test that will be done is an audiogram to assess the patient’s level of hearing loss and differentiate between a sensorineural or conductive hearing loss. Again, this is commonly a low frequency loss, or occasionally high frequency with normal mid-frequency hearing.1 Additionally vestibular testing can be done, such as an electronystagmogram. This test measures inner ear response to caloric stimulation and various movements which can provide insight towards the source of the vertigo. The ear is irrigated with warm air followed by cold while the patient lies down, and as the temperature change triggers movement of fluid in the semicircular canals, nystagmic movement of the eyes follows.3 A normal test would result in equal nystagmic response in each eye. With Meniere’s, often times the affected side will display a lesser response to testing, while in contrast some sort of inner ear lesion or irritation may result in an increased response.3 Labs and imaging studies such as an MRI for diagnosis of Meniere’s Disease are primarily used to rule out other possible causes, most importantly brain lesions that can cause similar symptoms of vertigo. Some controversial tests attempt to assess endolymphatic hydrops such as glycerine, urea, and sorbitol stress tests but are not widely used due to their low specificity and sensitivity.1
The treatment for Meniere’s usually doesn’t aim to cure the disease but rather to control the chronic symptoms associated with it. Hearing loss connected with this diagnosis does not have any treatment. However, the vertigo spells have several options to reduce severity and frequency. First line treatments for vertigo spells are anti-emetics (Zofran or Promethazine) to control nausea, and motion sickness medications (Valium or Antivert) to reduce spinning sensations. The most non-invasive option for long-term control is strict diet control. Often Meniere’s patients will be affected by certain dietary triggers, and it usually helps to restrict salt consumption as well as caffeine, alcohol, and nicotine, which reduce blood flow to the inner ear and also cause imbalances to fluid and electrolytes.1 Diuretics such as acetazolamide may also reduce the frequency of vertigo attacks.4 There are also several more “destructive” options to control refractory vertigo. These consist of injecting dexamethasone, a steroid, or gentamicin, an ototoxic antibiotic, into the inner ear. Gentamicin can be especially damaging to the cochlea and semicircular canals but may alleviate some chronic symptoms of Meniere’s. Most times it has the added complication of further reducing hearing ability. The most invasive options include surgical routes of endolymphatic sac procedures, vestibular nerve sections, or even a total labyrinthectomy, which would only be performed if the hearing loss in the affected ear was already completely lost.
References:
1. Moskowitz HS et al. Meniere Disease. UpToDate. https://www.uptodate.com/contents/meniere-disease?source=search_result&search=meniere%20disease&selectedTitle=1~29#H4. Updated 3/22/16. Accessed 4/22/17.
2. Branch WT et al. Approach to the patient with dizziness. UpToDate. https://www.uptodate.com/contents/approach-to-the-patient-with-dizziness?source=search_result&search=disequilibrium&selectedTitle=1~79#H7. Updated 10/20/14. Accessed 4/22/17.
3. Li JC. Meniere Disease (Idiopathic Endolymphatic Hydrops) Workup. UpToDate. http://emedicine.medscape.com/article/1159069-workup#c1. Updated 11/29/16. Accessed 4/22/17.
4. Mayo Clinic Staff. Meniere’s Disease – Treatment and drugs. Mayo Clinic. http://www.mayoclinic.org/diseases-conditions/menieres-disease/basics/treatment/con-20028251. Published 11/26/15. Accessed 4/22/17. -
Question 35 of 56
35. Question
A 20 year-old female presents with sore throat, odynophagia, headache, and fever for the past two days. She reports ill contacts at work only. She complains of nausea, loss of appetite, fatigue, tinnitus, and odynophagia. On examination, she has bilaterally tender, swollen anterior cervical lymphadenopathy, bilateral tonsillar hypertrophy with exudates, and fever of 100.9°F oral. According to Centor criteria, which of the following is an essential clinical feature consistent with the diagnosis of Group A Streptococcal pharyngitis?
Correct
The answer is C. In the diagnosis of Group A streptococcal pharyngitis, Centor’s criteria is an important clinical tool that physicians use for the management and treatment of GABHS. Centor’s criteria consists of: anterior cervical lymphadenopathy, pharyngotonsillar exudate, a temperature of 100.9°F, and the absence of cough. Streptococcal pharyngitis is diagnosed in over 11 million patients each year1, with the highest incidence occurring among children 5 to 15 years of age. This infection is present most commonly in the winter and the early spring. Risk factors for developing streptococcal pharyngitis include chronic medical illnesses such as diabetes, immunodeficiency, being within crowded environments such as schools, and exposure through infected persons.
Although viruses are the most common causes of acute pharyngitis, Group A beta-hemolytic Streptococcus (GABHS) is the most common bacterial cause of pharyngitis. Some of the respiratory viruses that cause acute pharyngitis include adenovirus, inflenza virus, RSV, and rhinovirus, among many others. However, GABHS accounts for about 15-30% of cases of acute pharyngitis in children and 5-20% of cases in adults. This infection is transmitted through respiratory droplets, which means that the bacteria is spread through the droplets after an infected person coughs or sneezes. It is important in preventing the spread of infection by practicing good hand washing techniques, refraining from drinking from the same glass as an infected person, or eating food that an infected person had eaten.
The microorganism Group A beta-hemolytic streptococci (Streptococcus pyogenes), is a gram positive extracellular bacterial pathogen that colonizes the throat or skin. There are few individuals that have chronic GABHS colonization, but they do not have clinical symptoms of the disease, and these individuals are at low risk of transmitting the disease to others. In the majority of individuals, the proliferation of GABHS in the pharynx causes the clinical signs and symptoms of the infection. This is due to the fact that GABHS has many surface proteins and it produces extracellular products that cause evasion of the immune system. Specifically, the M protein is responsible for the virulence and clinical symptoms that are seen in infected individuals.
In the clinical diagnosis of GABHS in patients, it is important to first differentiate the clinical differences between the viral and bacterial etiologies of acute pharyngitis. Patients with bacterial etiology (GABHS) present with sudden onset of a sore throat, odynophagia, and fever. In children, headache, nausea, vomiting, and abdominal pain may be present. On examination, there is inflammation of the pharynx and tonsils with or without exudates and enlarged anterior cervical lymph nodes. Clinicians often use the “Centor Criteria” for the diagnosis which includes temperature >100.4°F, anterior cervical lymphadenopathy, absence of a cough, and pharyngotonsillar exudate. Conversely, patients with acute pharyngitis of viral origin present with conjunctivitis, cough, hoarseness, coryza, viral exanthems, diarrhea, and the absence of a fever. It is also important to ask the patient if they have a history of exposure to an infected person with GABHS, as this is significant information to aid in the diagnosis.
The diagnostic tests that are useful in detecting GABHS include the rapid antigen detection test (RADT) and throat culture. The culture of a throat swab for detecting the presence of GABHS has a sensitivity of 90-95%6, whereas the RADT for strep is slightly less sensitive. Using the Centor criteria, clinicians identify which patients need a rapid antigen test or throat culture. When the patients have met >2 out of the 4 criteria, these patients require further testing. When >3 out of the 4 criteria are present, then the rapid antigen testing sensitivity is over 90%. With less than 2 out of the 4 criteria present, GABHS is not likely. This being said, if the patient has more than 2 out of the 4 criteria, the clinician will perform the RADT or throat culture. If the RADT is positive, then it is not necessary to perform a throat culture. However, if a RADT is negative in high prevalence settings and clinical suspicion is high for the infection, a throat culture should be performed. For children and adolescents, all negative RADT results should be followed with a throat culture. Due to the lower incidence of streptococcal infection in adults and low risk of rheumatic fever, it is not necessary to confirm the diagnosis of the infection with a throat culture following a negative RADT.
Group A streptococcal pharyngitis is a often a self-limited disease, which means that the fever and constitutional symptoms reside in 3-4 days without the use of antibiotic therapy. However, the use of antibiotics is indicated for those patients experiencing symptoms of the infection with a positive throat culture or rapid antigen detection test. In addition, if there is a high index of suspicion for the infection, and the clinician is waiting for laboratory results, the patient should also be placed on antibiotics. The Centor criteria is also used in the management of the disease, thus when patients have 4 criteria met, antibiotics should be started. The antibiotics that are effective in treating GABHS include penicillin, ampicillin, amoxicillin, clindamycin, certain cephalosporin and macrolides. The course of treatment for the antibiotics is usually 10 days. The treatment of choice for GABHS pharyngitis remains penicillin due to its efficacy and safety, low cost, and narrow spectrum6. However, amoxicillin can be used instead with equal effectiveness and more tolerable taste. For non-compliant patients who are not able to finish a full 10 day course of antibiotics, it is recommended that they be treated with a single IM dose of penicillin G benzathine. Also, treatment guidelines state that those with a penicillin allergy should be treated with erythromycin or second generation macrolides azithromycin or clarithromycin. Rarely, there are patients with erythromycin-resistant strain of GABHS and when they are unable to tolerate beta lactam antibiotics, clindamycin is the appropriate choice. In addition, symptomatic treatment includes acetaminophen for reducing fever, cough drops as needed for throat discomfort, rest, and plenty of fluids.
References
Choby, B. Diagnosis and Treatment of Streptococcal Pharyngitis. American Family Physician . 2009;79(5):383–390.
Gerber, M, Baltimore, R, Eaton, C, et al. Prevention of Rheumatic Fever and Diagnosis and Treatment of Acute Streptococcal Pharyngitis. American Heart Association. 2009;119:1541–1551.
Chahine, E, Chamoun, J, Sucher, A. Update on the Management of Streptococcal Pharyngitis. US Pharmacist. 2013;38(7):51–56.
Strep Throat. National Center for Biotechnology Information. Available at: http://www.ncbi.nlm.nih.gov/pubmedhealth/pmht0024690/. Accessed March 10, 2016.
Cunningham, M. Pathogenesis of Group A Streptococcal Infections. Clinical Microbiology Reviews. 2000;13(3):470–511.
Alan, B., Gerber, M., Gwaltney, J., Kaplan, E., Schwartz, RH. Practice Guidelines For The Diagnosis And Management Of Group A Streptococcal Pharyngitis. Infectious Diseases in Clinical Practice. 2002;35:113–123.
Papadakis MA, McPhee SJ, Rabow MW. 2015 Current Medical Diagnosis and Treatment. New York: McGraw-Hill Education/Medical 2015.
Incorrect
The answer is C. In the diagnosis of Group A streptococcal pharyngitis, Centor’s criteria is an important clinical tool that physicians use for the management and treatment of GABHS. Centor’s criteria consists of: anterior cervical lymphadenopathy, pharyngotonsillar exudate, a temperature of 100.9°F, and the absence of cough. Streptococcal pharyngitis is diagnosed in over 11 million patients each year1, with the highest incidence occurring among children 5 to 15 years of age. This infection is present most commonly in the winter and the early spring. Risk factors for developing streptococcal pharyngitis include chronic medical illnesses such as diabetes, immunodeficiency, being within crowded environments such as schools, and exposure through infected persons.
Although viruses are the most common causes of acute pharyngitis, Group A beta-hemolytic Streptococcus (GABHS) is the most common bacterial cause of pharyngitis. Some of the respiratory viruses that cause acute pharyngitis include adenovirus, inflenza virus, RSV, and rhinovirus, among many others. However, GABHS accounts for about 15-30% of cases of acute pharyngitis in children and 5-20% of cases in adults. This infection is transmitted through respiratory droplets, which means that the bacteria is spread through the droplets after an infected person coughs or sneezes. It is important in preventing the spread of infection by practicing good hand washing techniques, refraining from drinking from the same glass as an infected person, or eating food that an infected person had eaten.
The microorganism Group A beta-hemolytic streptococci (Streptococcus pyogenes), is a gram positive extracellular bacterial pathogen that colonizes the throat or skin. There are few individuals that have chronic GABHS colonization, but they do not have clinical symptoms of the disease, and these individuals are at low risk of transmitting the disease to others. In the majority of individuals, the proliferation of GABHS in the pharynx causes the clinical signs and symptoms of the infection. This is due to the fact that GABHS has many surface proteins and it produces extracellular products that cause evasion of the immune system. Specifically, the M protein is responsible for the virulence and clinical symptoms that are seen in infected individuals.
In the clinical diagnosis of GABHS in patients, it is important to first differentiate the clinical differences between the viral and bacterial etiologies of acute pharyngitis. Patients with bacterial etiology (GABHS) present with sudden onset of a sore throat, odynophagia, and fever. In children, headache, nausea, vomiting, and abdominal pain may be present. On examination, there is inflammation of the pharynx and tonsils with or without exudates and enlarged anterior cervical lymph nodes. Clinicians often use the “Centor Criteria” for the diagnosis which includes temperature >100.4°F, anterior cervical lymphadenopathy, absence of a cough, and pharyngotonsillar exudate. Conversely, patients with acute pharyngitis of viral origin present with conjunctivitis, cough, hoarseness, coryza, viral exanthems, diarrhea, and the absence of a fever. It is also important to ask the patient if they have a history of exposure to an infected person with GABHS, as this is significant information to aid in the diagnosis.
The diagnostic tests that are useful in detecting GABHS include the rapid antigen detection test (RADT) and throat culture. The culture of a throat swab for detecting the presence of GABHS has a sensitivity of 90-95%6, whereas the RADT for strep is slightly less sensitive. Using the Centor criteria, clinicians identify which patients need a rapid antigen test or throat culture. When the patients have met >2 out of the 4 criteria, these patients require further testing. When >3 out of the 4 criteria are present, then the rapid antigen testing sensitivity is over 90%. With less than 2 out of the 4 criteria present, GABHS is not likely. This being said, if the patient has more than 2 out of the 4 criteria, the clinician will perform the RADT or throat culture. If the RADT is positive, then it is not necessary to perform a throat culture. However, if a RADT is negative in high prevalence settings and clinical suspicion is high for the infection, a throat culture should be performed. For children and adolescents, all negative RADT results should be followed with a throat culture. Due to the lower incidence of streptococcal infection in adults and low risk of rheumatic fever, it is not necessary to confirm the diagnosis of the infection with a throat culture following a negative RADT.
Group A streptococcal pharyngitis is a often a self-limited disease, which means that the fever and constitutional symptoms reside in 3-4 days without the use of antibiotic therapy. However, the use of antibiotics is indicated for those patients experiencing symptoms of the infection with a positive throat culture or rapid antigen detection test. In addition, if there is a high index of suspicion for the infection, and the clinician is waiting for laboratory results, the patient should also be placed on antibiotics. The Centor criteria is also used in the management of the disease, thus when patients have 4 criteria met, antibiotics should be started. The antibiotics that are effective in treating GABHS include penicillin, ampicillin, amoxicillin, clindamycin, certain cephalosporin and macrolides. The course of treatment for the antibiotics is usually 10 days. The treatment of choice for GABHS pharyngitis remains penicillin due to its efficacy and safety, low cost, and narrow spectrum6. However, amoxicillin can be used instead with equal effectiveness and more tolerable taste. For non-compliant patients who are not able to finish a full 10 day course of antibiotics, it is recommended that they be treated with a single IM dose of penicillin G benzathine. Also, treatment guidelines state that those with a penicillin allergy should be treated with erythromycin or second generation macrolides azithromycin or clarithromycin. Rarely, there are patients with erythromycin-resistant strain of GABHS and when they are unable to tolerate beta lactam antibiotics, clindamycin is the appropriate choice. In addition, symptomatic treatment includes acetaminophen for reducing fever, cough drops as needed for throat discomfort, rest, and plenty of fluids.
References
Choby, B. Diagnosis and Treatment of Streptococcal Pharyngitis. American Family Physician . 2009;79(5):383–390.
Gerber, M, Baltimore, R, Eaton, C, et al. Prevention of Rheumatic Fever and Diagnosis and Treatment of Acute Streptococcal Pharyngitis. American Heart Association. 2009;119:1541–1551.
Chahine, E, Chamoun, J, Sucher, A. Update on the Management of Streptococcal Pharyngitis. US Pharmacist. 2013;38(7):51–56.
Strep Throat. National Center for Biotechnology Information. Available at: http://www.ncbi.nlm.nih.gov/pubmedhealth/pmht0024690/. Accessed March 10, 2016.
Cunningham, M. Pathogenesis of Group A Streptococcal Infections. Clinical Microbiology Reviews. 2000;13(3):470–511.
Alan, B., Gerber, M., Gwaltney, J., Kaplan, E., Schwartz, RH. Practice Guidelines For The Diagnosis And Management Of Group A Streptococcal Pharyngitis. Infectious Diseases in Clinical Practice. 2002;35:113–123.
Papadakis MA, McPhee SJ, Rabow MW. 2015 Current Medical Diagnosis and Treatment. New York: McGraw-Hill Education/Medical 2015.
-
Question 36 of 56
36. Question
A 57 year-old female patient with a history of controlled hypertension and vertigo complains of dizziness with movement, “heavy-headedness”, and fatigue. Her usual dose of Meclizine provided no relief. She has been taking HCTZ and Atenolol without any changes in dosage in the past 2 years. On exam, she shows a positive Romberg, positive Dix-Hallpike, and 4/5 UE motor strength, while the remainder of her neurological exam is unremarkable. How would you treat her vertigo?
Correct
Correct answer: D. 1-2 sessions of in-office Canalith repositioning. Canalith repositioning (CRP) is the treatment of choice for confirmed BPPV by positive Dix-Hallpike. The most common CRP is the Epley maneuver which general principle is to return the particles in the semi-circular canal back to its original place so that the vestibular system is no longer out of balance.
Incorrect answers:
- Decrease dosage of HCTZ. The patient has not had a change in dosage in this medication for the past 2 years. Any cardiogenic vertigo would have presented earlier and closer to the time of a dosage change.
- Increase dosage of Meclizine. It is possible that a higher dose may treat her vertigo however her normal dose provided no relief when it has in the past. There is a better treatment option that provides immediate relief without an increased risk of side effects.
- Immediate ENT surgical consult. The surgical route is reserved for patients whose vertigo is unresolved with CRP. Surgery is not recommended until all other options are exhausted because it is invasive and poses a risk of nerve damage.
Epidemiology: BPPV is prevalent among every 64 per 100,000 people. Gender distribution is slightly higher towards females (64%). BBPV occurs in an older population with an average age of 51-57 years of age. It is a rare incident in patients younger than 35 years without a history of prior head trauma. There is insignificant data correlating racial affinity. Some predisposing factors for BPPV include inactivity, acute alcoholism, major surgery, and central nervous system disease. Many patients may have concurrent ear pathology including: idiopathic, trauma, ear diseases, otitis media, vestibular neuritis, Meniere disease, otosclerosis, sudden sensorineural hearing loss, vestibular basilar insuffiency, acoustic neuroma, and cervical vertigo.
Etiology: BPPV is an abnormal sensation that is evoked by provocative positions that usually trigger certain eye movements like nystagmus. The cause of BPPV is categorized into 2 groups: canalithiasis and cupulolithiasis. Canalithiasis or canal rocks occur when particles are present in the canal of the semi-circular canals. These rock particles are free floating and mobile leading to vertigo by exertion of force. Cupulolithiasis, or cupula rocks, are affixed to the cupula of the crista ampullaris. In contrast, these densities are non-mobile present in the ampulla of the semi-circular canals.
Pathophysiology: In the cupulolithiasis explanation, basophilic particles are attached to the cupula rendering the posterior semicircular canal (PSC) overly sensitive to gravity. The abnormally attached structures can impinge on the cupula. The additional weight of the cupula rocks makes the pole unstable and more difficult to maintain a neutral position. The unstable cupula is prone to stay in one direction depending on the tilt of the head. Once a position is made, the weight of the cupula rocks keeps the cupula from recoiling back to a neutral position. This results in persistent nystagmus and dizziness, especially when a patient is in a specific position, ie tilted backwards.
In the canalithiasis explanation, symptoms of BPPV are related to the mobile canaliths in the posterior semi-circular canal. When the head is upright, the densities are at the most gravity-dependent position. When the head is tilted back supine, the particles circumduct approximately 90 degrees up along the arc of the PSC. After a transient lag, the particles move back down the arc by gravitational force. The downward movement of the particles causes the endolymph to flow away from the ampulla and the cupula to be deflected. The bending of the cupula results in nystagmus. When the head returns to an upright position, the rotation and cupular deflection is reversed, thus dizziness with nystagmus flickering in the opposite direction occurs. The canalithiasis explanation is analogous to pebbles inside a tire. As the tire rotates, the pebbles are picked up temporarily then fall back down with gravity. The tumbling results in dizziness. Reversal of the tire rotation makes the flow and dizziness go in opposite direction. Canalithiasis explains the delay (latency), transient nystagmus and reversal on return to upright.
Pertinent historical and physical findings: The onset of BPPV is sudden. Most patients will awake with these symptoms, often noticing vertigo while trying to sit up suddenly. Positional vertigo is variable from days and weeks to even months or years. Symptoms may resolve then recur. Severity is also variable. In extreme instances, the faintest head movement will elicit nausea and vomiting. There can be strong nystagmus, while others will be unfazed. Sensation of dizziness is not constant. Head movement triggers severe dizziness. In between episodes, patients will have few or no symptoms. Some patients may complain of a foggy or cloudy sensation.
Classic BPPV symptoms are triggered by suddenly moving from an erect position to supine position while the head is angled 45 degrees towards the side of the affected ear. After reaching the provocative position, a delay of a few seconds occurs before the dizzy spell strikes. The patient feels like they are suddenly thrown into a rolling spin toward the side of the affected ear. Symptoms begin very violently and usually resolve within 20 to 30 seconds. The sensation is triggered once again when returning to sitting erect, but the nystagmus is in the opposite direction. Generally physical findings are unremarkable. Neurological exam findings are normal except those from the Dix-Hallpike maneuver. However, abnormal neurological exam findings do not rule out a BPPV diagnosis.
Diagnostic correlations (labs/imaging): The standard clinical test for BPPV is the Dix-Hallpike maneuver. A patient with classic rotatory nystagmus with latency and limited duration is also considered diagnostic for BPPV. In the Dix-Hallpike maneuver, the patient is moved rapidly from a sitting position to a supine position with the head 45 degrees to one side. In approximately 20 to 30 seconds, the patient is then returned to the sitting position. If there is an absence of nystagmus, the maneuver is performed with the head turned to the opposite side.
Management/treatment: Medical care includes watchful waiting, vestibulosuppressant medication, vestibular rehabilitation, canalith repositioning, and surgery. BPPV can resolve with watchful watching, however it may take weeks to months for symptoms-leaving the patient in discomfort and vertigo until resolution. In addition, patients may be at risk for a fall or other events from a vertigo episode. Vestibulosuppressant medication does not control vertigo but it may provide minimal relief of symptoms. Patients may feel groggy or sleepy with this medication. Vestibular rehabilitation is noninvasive therapy that can be successful only after a considerable period of time. While the therapeutic maneuvers are performed, vertigo is repeatedly stimulated.
Canalith repositioning procedure (CRP) is a first choice treatment option due to its high benefit-to-risk ratio and its simple noninvasive treatment to cure BPPV in 1-2 sessions. There are 2 methods in canalith repositioning therapy: Epley maneuver and Semont maneuver. Both options involve head repositioning to rearrange the displaced particles, however the Epley is gentler.
In the Epley maneuver the patient begins in a sitting position with the head turned 45 degrees towards the affected side. In position 1, the patient slowly reclines to the supine position. The rate of recline is so that no nystagmus is elicited- usually about 30 seconds. In position 2, the patient continues to recline supine until 15 degrees Trendelenburg is reached and head turned 45 degrees toward affected side. It takes about 10 seconds to recline with another 20 seconds in Trendelenburg position. In position 3, the body is still in supine 15 degrees Trendelenburg with the head turned 45 degrees towards the contralateral side. In position 4, the body is turned from position 3 so that the shoulders are perpendicular to the floor with the affected ear facing upwards. The head is turned farther so that the nose extends 45 degrees below the horizon. This takes 40 seconds. In position 5, the patient is raised back to sitting with the head still turned away from affected side (at least 90-135 degrees toward contralateral side). In position 6, the head is returned to midline.
After the completion of the Epley maneuver, a Dix-Hallpike test is performed immediately. If nystagmus is observed, the Epley maneuver is repeated. After CRP treatment, patients should avoid lying down completely flat for 24-48 hours. They should sleep with the head elevated. They are also advised to avoid aggravating activities such as jumping or flipping. Patients may also want to adjust their sleeping position to prevent recurrence.
If CRP fails, surgical options are considered. They are invasive and hold risks of complications (hearing loss and facial nerve damage). Surgical options for vertigo control include: labyrinthectomy, posterior canal occlusion, singular neurectomy, vestibular nerve resection, and transtympanic aminoglycoside application. Labyrinthectomy and vestibular nerve resection are only considered in extreme cases. Singular neurectomy is difficult and has been mastered by only a select number of surgeons. The most practical surgical option is posterior canal occlusion. The posterior canal is collapsed and therefore the particles are immobilized in the canal. This procedure takes the mastoidectomy approach- the hard bone is drilled through with diamond burrs to expose the labyrinth without spilled excessive perilymphatic fluid. The labyrinth is compressed so that the flow of endolymphatic fluid and movement of particles are disrupted.
References
Anagnostou E, Kouzi I, Spengos K. Diagnosis and Treatment of Anterior-Canal Benign Paroxysmal Positional Vertigo: A Systematic Review. J Clin Neurol. 2015;11(3):262-7. http://reference.medscape.com/medline/abstract/26022461. Accessed May 26th, 2016.
Huppert D, Strupp M, Mückter H, Brandt T. Which medication do I need to manage dizzy patients?. Acta Oto-laryngologica. 2011;131(3):228-41. https://www.ncbi.nlm.nih.gov/pubmed/21142898. Accessed May 26th, 2016.
Incorrect
Correct answer: D. 1-2 sessions of in-office Canalith repositioning. Canalith repositioning (CRP) is the treatment of choice for confirmed BPPV by positive Dix-Hallpike. The most common CRP is the Epley maneuver which general principle is to return the particles in the semi-circular canal back to its original place so that the vestibular system is no longer out of balance.
Incorrect answers:
- Decrease dosage of HCTZ. The patient has not had a change in dosage in this medication for the past 2 years. Any cardiogenic vertigo would have presented earlier and closer to the time of a dosage change.
- Increase dosage of Meclizine. It is possible that a higher dose may treat her vertigo however her normal dose provided no relief when it has in the past. There is a better treatment option that provides immediate relief without an increased risk of side effects.
- Immediate ENT surgical consult. The surgical route is reserved for patients whose vertigo is unresolved with CRP. Surgery is not recommended until all other options are exhausted because it is invasive and poses a risk of nerve damage.
Epidemiology: BPPV is prevalent among every 64 per 100,000 people. Gender distribution is slightly higher towards females (64%). BBPV occurs in an older population with an average age of 51-57 years of age. It is a rare incident in patients younger than 35 years without a history of prior head trauma. There is insignificant data correlating racial affinity. Some predisposing factors for BPPV include inactivity, acute alcoholism, major surgery, and central nervous system disease. Many patients may have concurrent ear pathology including: idiopathic, trauma, ear diseases, otitis media, vestibular neuritis, Meniere disease, otosclerosis, sudden sensorineural hearing loss, vestibular basilar insuffiency, acoustic neuroma, and cervical vertigo.
Etiology: BPPV is an abnormal sensation that is evoked by provocative positions that usually trigger certain eye movements like nystagmus. The cause of BPPV is categorized into 2 groups: canalithiasis and cupulolithiasis. Canalithiasis or canal rocks occur when particles are present in the canal of the semi-circular canals. These rock particles are free floating and mobile leading to vertigo by exertion of force. Cupulolithiasis, or cupula rocks, are affixed to the cupula of the crista ampullaris. In contrast, these densities are non-mobile present in the ampulla of the semi-circular canals.
Pathophysiology: In the cupulolithiasis explanation, basophilic particles are attached to the cupula rendering the posterior semicircular canal (PSC) overly sensitive to gravity. The abnormally attached structures can impinge on the cupula. The additional weight of the cupula rocks makes the pole unstable and more difficult to maintain a neutral position. The unstable cupula is prone to stay in one direction depending on the tilt of the head. Once a position is made, the weight of the cupula rocks keeps the cupula from recoiling back to a neutral position. This results in persistent nystagmus and dizziness, especially when a patient is in a specific position, ie tilted backwards.
In the canalithiasis explanation, symptoms of BPPV are related to the mobile canaliths in the posterior semi-circular canal. When the head is upright, the densities are at the most gravity-dependent position. When the head is tilted back supine, the particles circumduct approximately 90 degrees up along the arc of the PSC. After a transient lag, the particles move back down the arc by gravitational force. The downward movement of the particles causes the endolymph to flow away from the ampulla and the cupula to be deflected. The bending of the cupula results in nystagmus. When the head returns to an upright position, the rotation and cupular deflection is reversed, thus dizziness with nystagmus flickering in the opposite direction occurs. The canalithiasis explanation is analogous to pebbles inside a tire. As the tire rotates, the pebbles are picked up temporarily then fall back down with gravity. The tumbling results in dizziness. Reversal of the tire rotation makes the flow and dizziness go in opposite direction. Canalithiasis explains the delay (latency), transient nystagmus and reversal on return to upright.
Pertinent historical and physical findings: The onset of BPPV is sudden. Most patients will awake with these symptoms, often noticing vertigo while trying to sit up suddenly. Positional vertigo is variable from days and weeks to even months or years. Symptoms may resolve then recur. Severity is also variable. In extreme instances, the faintest head movement will elicit nausea and vomiting. There can be strong nystagmus, while others will be unfazed. Sensation of dizziness is not constant. Head movement triggers severe dizziness. In between episodes, patients will have few or no symptoms. Some patients may complain of a foggy or cloudy sensation.
Classic BPPV symptoms are triggered by suddenly moving from an erect position to supine position while the head is angled 45 degrees towards the side of the affected ear. After reaching the provocative position, a delay of a few seconds occurs before the dizzy spell strikes. The patient feels like they are suddenly thrown into a rolling spin toward the side of the affected ear. Symptoms begin very violently and usually resolve within 20 to 30 seconds. The sensation is triggered once again when returning to sitting erect, but the nystagmus is in the opposite direction. Generally physical findings are unremarkable. Neurological exam findings are normal except those from the Dix-Hallpike maneuver. However, abnormal neurological exam findings do not rule out a BPPV diagnosis.
Diagnostic correlations (labs/imaging): The standard clinical test for BPPV is the Dix-Hallpike maneuver. A patient with classic rotatory nystagmus with latency and limited duration is also considered diagnostic for BPPV. In the Dix-Hallpike maneuver, the patient is moved rapidly from a sitting position to a supine position with the head 45 degrees to one side. In approximately 20 to 30 seconds, the patient is then returned to the sitting position. If there is an absence of nystagmus, the maneuver is performed with the head turned to the opposite side.
Management/treatment: Medical care includes watchful waiting, vestibulosuppressant medication, vestibular rehabilitation, canalith repositioning, and surgery. BPPV can resolve with watchful watching, however it may take weeks to months for symptoms-leaving the patient in discomfort and vertigo until resolution. In addition, patients may be at risk for a fall or other events from a vertigo episode. Vestibulosuppressant medication does not control vertigo but it may provide minimal relief of symptoms. Patients may feel groggy or sleepy with this medication. Vestibular rehabilitation is noninvasive therapy that can be successful only after a considerable period of time. While the therapeutic maneuvers are performed, vertigo is repeatedly stimulated.
Canalith repositioning procedure (CRP) is a first choice treatment option due to its high benefit-to-risk ratio and its simple noninvasive treatment to cure BPPV in 1-2 sessions. There are 2 methods in canalith repositioning therapy: Epley maneuver and Semont maneuver. Both options involve head repositioning to rearrange the displaced particles, however the Epley is gentler.
In the Epley maneuver the patient begins in a sitting position with the head turned 45 degrees towards the affected side. In position 1, the patient slowly reclines to the supine position. The rate of recline is so that no nystagmus is elicited- usually about 30 seconds. In position 2, the patient continues to recline supine until 15 degrees Trendelenburg is reached and head turned 45 degrees toward affected side. It takes about 10 seconds to recline with another 20 seconds in Trendelenburg position. In position 3, the body is still in supine 15 degrees Trendelenburg with the head turned 45 degrees towards the contralateral side. In position 4, the body is turned from position 3 so that the shoulders are perpendicular to the floor with the affected ear facing upwards. The head is turned farther so that the nose extends 45 degrees below the horizon. This takes 40 seconds. In position 5, the patient is raised back to sitting with the head still turned away from affected side (at least 90-135 degrees toward contralateral side). In position 6, the head is returned to midline.
After the completion of the Epley maneuver, a Dix-Hallpike test is performed immediately. If nystagmus is observed, the Epley maneuver is repeated. After CRP treatment, patients should avoid lying down completely flat for 24-48 hours. They should sleep with the head elevated. They are also advised to avoid aggravating activities such as jumping or flipping. Patients may also want to adjust their sleeping position to prevent recurrence.
If CRP fails, surgical options are considered. They are invasive and hold risks of complications (hearing loss and facial nerve damage). Surgical options for vertigo control include: labyrinthectomy, posterior canal occlusion, singular neurectomy, vestibular nerve resection, and transtympanic aminoglycoside application. Labyrinthectomy and vestibular nerve resection are only considered in extreme cases. Singular neurectomy is difficult and has been mastered by only a select number of surgeons. The most practical surgical option is posterior canal occlusion. The posterior canal is collapsed and therefore the particles are immobilized in the canal. This procedure takes the mastoidectomy approach- the hard bone is drilled through with diamond burrs to expose the labyrinth without spilled excessive perilymphatic fluid. The labyrinth is compressed so that the flow of endolymphatic fluid and movement of particles are disrupted.
References
Anagnostou E, Kouzi I, Spengos K. Diagnosis and Treatment of Anterior-Canal Benign Paroxysmal Positional Vertigo: A Systematic Review. J Clin Neurol. 2015;11(3):262-7. http://reference.medscape.com/medline/abstract/26022461. Accessed May 26th, 2016.
Huppert D, Strupp M, Mückter H, Brandt T. Which medication do I need to manage dizzy patients?. Acta Oto-laryngologica. 2011;131(3):228-41. https://www.ncbi.nlm.nih.gov/pubmed/21142898. Accessed May 26th, 2016.
-
Question 37 of 56
37. Question
A patient reports decreased hearing and a feeling that there is persistent water in the left ear. On exam, the patient appears to have a conductive hearing loss, there is no visible swelling or redness of the ear or drainage from the canal. Otoscope exam reveals a large amount of cerumen within the canal. Tympanic membrane is not able to be visualized. Which of the following would be appropriate for intervention?
Correct
The correct answer is C. Docusate sodium, Colace, is one of the most effective agents for breaking down cerumen. Other effective methods for removing wax are irrigation with warm water (cold may induce vertigo) and an ear curette. Alligator forceps can be useful in removing a foreign body. An ear wick is utilized when otitis externa is diagnosed and the canal is swollen to the point that topical solution may not be able to penetrate the entire length of the canal. A Katz extractor is a small catheter with an inflatable balloon which can be inflated once the catheter has been passed behind a foreign body to facilitate removal. This is primarily used for nasal foreign bodies. Cotton swabs, “Q-tips”, should be avoided as they push wax down further in the canal, ultimately leading to an impaction. Use of ear plugs or hearing aids can also lead to an impaction.
Cerumen, commonly referred to as “ear wax”, is a substance produced by apocrine sweat glands located in the subcutaneous layer of the ear canal. Cerumen production serves many purposes: moisten and lubricate the epidermal surface, protect and lubricate the skin, provide an acidic pH to minimize bacterial growth, an protect against dust, water, and potential insects. The underlying components of the wax include fatty acids, alcohols, and cholesterol precursors, among other substances. The quantity is affected by environmental conditions, climate, hypercholesterolemia, and disorders such as eczema. Genetics play a role in the consistency of the wax produced. Those of East Asian descent tend to produce a drier wax while than other races. The percentages of the underlying components as well as environmental factors contribute to the consistency and color of the wax. Quantity of cerumen production generally declines with age.
Symptoms associated with excess cerumen include diminished or absent hearing, discomfort, fullness, pruritis, dizziness, or tinnitus. The ideal duration of allowing Colace to work in breaking down the cerumen is 60 minutes but it can be effective with less time. In addition to Colace, mineral oil and hydrogen peroxide are also commonly used cerumenolytics. After the cerumen is removed using warm irrigation and/or manually with an ear curette, always reassess for any retained wax or damage to the canal or tympanic membrane. Several attempts may be necessary to completely remove the majority of the wax. Preventative measures can include using a small amount of mineral oil on a cotton ball and placing in the ear for 10-20 minutes once per week in addition to irrigation by medical personnel every 6-12 months.
References:
Incorrect
The correct answer is C. Docusate sodium, Colace, is one of the most effective agents for breaking down cerumen. Other effective methods for removing wax are irrigation with warm water (cold may induce vertigo) and an ear curette. Alligator forceps can be useful in removing a foreign body. An ear wick is utilized when otitis externa is diagnosed and the canal is swollen to the point that topical solution may not be able to penetrate the entire length of the canal. A Katz extractor is a small catheter with an inflatable balloon which can be inflated once the catheter has been passed behind a foreign body to facilitate removal. This is primarily used for nasal foreign bodies. Cotton swabs, “Q-tips”, should be avoided as they push wax down further in the canal, ultimately leading to an impaction. Use of ear plugs or hearing aids can also lead to an impaction.
Cerumen, commonly referred to as “ear wax”, is a substance produced by apocrine sweat glands located in the subcutaneous layer of the ear canal. Cerumen production serves many purposes: moisten and lubricate the epidermal surface, protect and lubricate the skin, provide an acidic pH to minimize bacterial growth, an protect against dust, water, and potential insects. The underlying components of the wax include fatty acids, alcohols, and cholesterol precursors, among other substances. The quantity is affected by environmental conditions, climate, hypercholesterolemia, and disorders such as eczema. Genetics play a role in the consistency of the wax produced. Those of East Asian descent tend to produce a drier wax while than other races. The percentages of the underlying components as well as environmental factors contribute to the consistency and color of the wax. Quantity of cerumen production generally declines with age.
Symptoms associated with excess cerumen include diminished or absent hearing, discomfort, fullness, pruritis, dizziness, or tinnitus. The ideal duration of allowing Colace to work in breaking down the cerumen is 60 minutes but it can be effective with less time. In addition to Colace, mineral oil and hydrogen peroxide are also commonly used cerumenolytics. After the cerumen is removed using warm irrigation and/or manually with an ear curette, always reassess for any retained wax or damage to the canal or tympanic membrane. Several attempts may be necessary to completely remove the majority of the wax. Preventative measures can include using a small amount of mineral oil on a cotton ball and placing in the ear for 10-20 minutes once per week in addition to irrigation by medical personnel every 6-12 months.
References:
-
Question 38 of 56
38. Question
A patient presents with left ear pain. On exam, there is pain with palpation of the tragus and the canal is erythematous, edematous and discharge is present. The tympanic membrane is unable to be visualized secondary to swelling of the canal. Which of the following topical preparations is the most appropriate treatment option?
Correct
The correct answer is B. Based on the exam, the patient is suffering from an acute otitis externa. Acetic acid-hydrocortisone is used only in mild cases. The others can be used in the treatment of otitis externa but contain an aminoglycoside which may be ototoxic if a perforation is present. Ciprofloxacin-hydrocortisone otic suspension provides excellent coverage against the two most common pathogens found in otitis externa, S. aureus and P. aeruginosa. Anerobic and fungal etiologies are also possible.
Otitis externa (OE), also referred to as external otitis or swimmer’s ear, is an inflammation of the ear canal which is usually the result of a localized infection. Less commonly, contact dermatitis and disorders such as eczema and psoriasis are other causes. OE can affect males and females of any age with ages 5-15 being the most commonly effected. Most cases occur in the summer months. Cerumen is the first line of defense against OE. Removing cerumen with cotton swabs, fingernails, or other devices removes the protective barrier, causes abrasions to the epithelium, and allows for the pH to increase, which promotes bacterial growth. Swimming is commonly implicated because excess water contact results in skin maceration and the water itself may contain bacteria, especially if swimming in untreated water.
Patients with OE can have pain, discharge, itching, or diminished hearing. Physical exam starts with examination and palpation of the outer ear. Discharge from the canal may be visible along with swelling. There may be pain with pushing in on the tragus or with pulling on the lobule (ear lobe) or other areas of the auricle, known as a positive “tug test”. Otoscope evaluation will show various degrees of erythema, edema, and discharge. The tympanic membrane may also be mildly erythematous but should not have significant bulging or fluid levels if isolated to the canal. Diagnosis is based primarily on the exam. Cultures can be performed if refractory to treatment. Malignant otitis externa is diagnosed when there is evidence that the infection has spread to either the surrounding bone, particularly the mastoid, or the skin and cartilage of the adjacent area. This usually occurs in the indigent or immunocompromised population and can be life threatening.
Uncomplicated cases are usually successfully treated with topical antimicrobial and steroid combinations mentioned above. If the swelling is severe, an ear wick may need to be placed in order to help the liquid get all the way down the canal. A wick is a small, firm piece of foam that can be inserted into the canal using forceps. A drop of saline or antibiotic will soften and expand the wick and expose the built-in central opening. Antibiotic solutions should be used for at least 1 week and may be extended an additional week if improving but not resolved. The patient should be advised to keep the ear as dry as possible for 10 days and tight-fitting ear plugs should be used if water sports are necessary. Acetic acid can be used prophylactically after swimming in lakes.
References:
Incorrect
The correct answer is B. Based on the exam, the patient is suffering from an acute otitis externa. Acetic acid-hydrocortisone is used only in mild cases. The others can be used in the treatment of otitis externa but contain an aminoglycoside which may be ototoxic if a perforation is present. Ciprofloxacin-hydrocortisone otic suspension provides excellent coverage against the two most common pathogens found in otitis externa, S. aureus and P. aeruginosa. Anerobic and fungal etiologies are also possible.
Otitis externa (OE), also referred to as external otitis or swimmer’s ear, is an inflammation of the ear canal which is usually the result of a localized infection. Less commonly, contact dermatitis and disorders such as eczema and psoriasis are other causes. OE can affect males and females of any age with ages 5-15 being the most commonly effected. Most cases occur in the summer months. Cerumen is the first line of defense against OE. Removing cerumen with cotton swabs, fingernails, or other devices removes the protective barrier, causes abrasions to the epithelium, and allows for the pH to increase, which promotes bacterial growth. Swimming is commonly implicated because excess water contact results in skin maceration and the water itself may contain bacteria, especially if swimming in untreated water.
Patients with OE can have pain, discharge, itching, or diminished hearing. Physical exam starts with examination and palpation of the outer ear. Discharge from the canal may be visible along with swelling. There may be pain with pushing in on the tragus or with pulling on the lobule (ear lobe) or other areas of the auricle, known as a positive “tug test”. Otoscope evaluation will show various degrees of erythema, edema, and discharge. The tympanic membrane may also be mildly erythematous but should not have significant bulging or fluid levels if isolated to the canal. Diagnosis is based primarily on the exam. Cultures can be performed if refractory to treatment. Malignant otitis externa is diagnosed when there is evidence that the infection has spread to either the surrounding bone, particularly the mastoid, or the skin and cartilage of the adjacent area. This usually occurs in the indigent or immunocompromised population and can be life threatening.
Uncomplicated cases are usually successfully treated with topical antimicrobial and steroid combinations mentioned above. If the swelling is severe, an ear wick may need to be placed in order to help the liquid get all the way down the canal. A wick is a small, firm piece of foam that can be inserted into the canal using forceps. A drop of saline or antibiotic will soften and expand the wick and expose the built-in central opening. Antibiotic solutions should be used for at least 1 week and may be extended an additional week if improving but not resolved. The patient should be advised to keep the ear as dry as possible for 10 days and tight-fitting ear plugs should be used if water sports are necessary. Acetic acid can be used prophylactically after swimming in lakes.
References:
-
Question 39 of 56
39. Question
Which of the following would be the best initial treatment for the findings on the attached image which occurred after a fall?Correct
The correct answer is C. The patient has a developing subchondral hematoma. If the blood is not evacuated, the patient will be left with a chronic deformity frequently referred to as a “cauliflower ear”. A compression bandage should be applied immediately after evacuating the blood to minimize reaccumulating. Antibiotics are not indicated for a closed injury until after the blood has been evacuated. Anticoagulants would allow for additional bleeding. An ear wick is used to treat otitis externa.
An auricular hematoma occurs when blunt trauma occurs to the auricle and blood collects between the perichondrium and the cartilage. The hematoma prevents perfusion coming from the perichondrial blood supply to the cartilage layer eventually leading to necrosis of the cartilage. The result is fibrosis and scar tissue which is generally irreversible and creates a visible deformity known as a “cauliflower ear”.
Small hematomas that present in less than 48 hours after injury can be treated with aspiration over the site of greatest fluctuance. An 18-22 gauge needle is used for aspiration after the skin has been prepped with an antiseptic. A larger needle is generally advised in case clotting has occurred. Larger hematomas or those that present after 48 hours may require incision and drainage followed by suturing closed the incision site. After either procedure, a pressure bandage is necessary to prevent re-accumulation and should be maintained for 72 hours. If the bandage is removed for wound checks or because it becomes dirty or falls off, it should be reapplied as soon as possible. Prophylactic antibiotics after the procedure can be considered.
References:
Incorrect
The correct answer is C. The patient has a developing subchondral hematoma. If the blood is not evacuated, the patient will be left with a chronic deformity frequently referred to as a “cauliflower ear”. A compression bandage should be applied immediately after evacuating the blood to minimize reaccumulating. Antibiotics are not indicated for a closed injury until after the blood has been evacuated. Anticoagulants would allow for additional bleeding. An ear wick is used to treat otitis externa.
An auricular hematoma occurs when blunt trauma occurs to the auricle and blood collects between the perichondrium and the cartilage. The hematoma prevents perfusion coming from the perichondrial blood supply to the cartilage layer eventually leading to necrosis of the cartilage. The result is fibrosis and scar tissue which is generally irreversible and creates a visible deformity known as a “cauliflower ear”.
Small hematomas that present in less than 48 hours after injury can be treated with aspiration over the site of greatest fluctuance. An 18-22 gauge needle is used for aspiration after the skin has been prepped with an antiseptic. A larger needle is generally advised in case clotting has occurred. Larger hematomas or those that present after 48 hours may require incision and drainage followed by suturing closed the incision site. After either procedure, a pressure bandage is necessary to prevent re-accumulation and should be maintained for 72 hours. If the bandage is removed for wound checks or because it becomes dirty or falls off, it should be reapplied as soon as possible. Prophylactic antibiotics after the procedure can be considered.
References:
-
Question 40 of 56
40. Question
A 50-year-old male reports gradual hearing loss. He states family members have noticed he has been more hard of hearing over the past year. He also reports occasional tinnitus. Past medical history includes hypertension, NIDDM, and neurofibromatosis. Physical exam reveals diminished gross hearing bilaterally with finger rub. Weber test does not lateralize, and air conduction is greater than bone conduction upon Rinne testing bilaterally. Which of the following is the most likely etiology of the patient’s hearing loss?
Correct
The correct answer is E. The exam findings are consistent with a sensorineural hearing loss although psychogenic etiology is possible. The patients’ history of neurofibromatosis makes bilateral vestibular schwannoma’s the most likely pathology. The remaining answer options are contributors to conductive hearing loss, which is not consistent with the Weber and Rinne results.
Vestibular schwannoma, also frequently referred to as acoustic neuroma, is a tumor of Schwann cell origin which occurs on the vestibular section of the eighth cranial nerve. The median age of diagnosis is 50 and the vast majority are unilateral, however, patients with neurofibromatosis can frequently have bilateral involvement. In addition to neurofibromatosis, other risk factors for developing these tumors include low dose childhood radiation exposure, as well as potential cell phone use and chronic noise exposure, although studies evaluating the last two are conflicting.
Symptoms of vestibular schwannomas vary depending on which structures the tumor impacts. Hearing loss and tinnitus are the most common complaints. Other symptoms include gait instability, facial numbness/hyperesthesia/pain, and taste disturbances. Physical exam of the canal and tympanic membrane is usually normal. In patients with a unilateral schwannoma, the Weber and Rinne tests will be abnormal. Because it is a sensorineural hearing loss, the Weber test will lateralize to the good ear (assuming there is no other pathology in the ear without the tumor) and the Rinne test will be normal because it will demonstrate air conduction greater than bone conduction. This demonstrates the importance of using the results of both tests in the interpretation. Audiometry testing will be abnormal. A MRI is used to confirm the diagnosis.
Treatment is usually via surgery or radiation. Recurrence and hearing retention vary based on the chosen intervention and underlying pathology. Post-intervention scans should be performed annually for 10 years to assess for recurrence. Many vestibular schwannomas grow at less than 1mm per year, therefore, observation with repeat imaging every 6-12 months is an option that can be discussed with the patient.
References:
https://oxfordmedicaleducation.com/clinical-examinations/tuning-fork-rinnes-webers-test/
Incorrect
The correct answer is E. The exam findings are consistent with a sensorineural hearing loss although psychogenic etiology is possible. The patients’ history of neurofibromatosis makes bilateral vestibular schwannoma’s the most likely pathology. The remaining answer options are contributors to conductive hearing loss, which is not consistent with the Weber and Rinne results.
Vestibular schwannoma, also frequently referred to as acoustic neuroma, is a tumor of Schwann cell origin which occurs on the vestibular section of the eighth cranial nerve. The median age of diagnosis is 50 and the vast majority are unilateral, however, patients with neurofibromatosis can frequently have bilateral involvement. In addition to neurofibromatosis, other risk factors for developing these tumors include low dose childhood radiation exposure, as well as potential cell phone use and chronic noise exposure, although studies evaluating the last two are conflicting.
Symptoms of vestibular schwannomas vary depending on which structures the tumor impacts. Hearing loss and tinnitus are the most common complaints. Other symptoms include gait instability, facial numbness/hyperesthesia/pain, and taste disturbances. Physical exam of the canal and tympanic membrane is usually normal. In patients with a unilateral schwannoma, the Weber and Rinne tests will be abnormal. Because it is a sensorineural hearing loss, the Weber test will lateralize to the good ear (assuming there is no other pathology in the ear without the tumor) and the Rinne test will be normal because it will demonstrate air conduction greater than bone conduction. This demonstrates the importance of using the results of both tests in the interpretation. Audiometry testing will be abnormal. A MRI is used to confirm the diagnosis.
Treatment is usually via surgery or radiation. Recurrence and hearing retention vary based on the chosen intervention and underlying pathology. Post-intervention scans should be performed annually for 10 years to assess for recurrence. Many vestibular schwannomas grow at less than 1mm per year, therefore, observation with repeat imaging every 6-12 months is an option that can be discussed with the patient.
References:
https://oxfordmedicaleducation.com/clinical-examinations/tuning-fork-rinnes-webers-test/
-
Question 41 of 56
41. Question
Which of the following is the most appropriate treatment for most cases of middle ear barotrauma?
Correct
The correct answer is D. The majority of cases of barotrauma will resolve spontaneously. If a large perforation of the tympanic membrane (TM) has occurred, a tympanoplasty may be indicated, however, these are less common. Antibiotics are only indicated if a tympanic membrane perforation has occurred in the setting of contaminated material. Glucocorticoids have no role in management. Tympanoplasty and tympanostomy tube have a place in management or prevention but are used much less frequently. Antihistamines and decongestants do not seem to speed recovery times but can be used if not contraindicated along with analgesics, which is most helpful.
Barotrauma is a term used to describe an injury to the TM resulting from pressure differences between the environment and the middle ear. Common causes of barotrauma include a rapid ascent or decent during flying, diving, decompression chamber use, hyperbaric oxygen chamber use, close proximity to a blast, sky diving, and upper respiratory tract infections that cause swelling of the nasopharynx at the site of the Eustachian tube orifice or of the Eustachian tube itself. Swallowing or yawning are methods that are used to equalize middle ear and atmospheric pressures but if the force is too large to compensate for, the TM stretches and bends beyond its capabilities causing damage. This damage, known as barotrauma, results in the history and exam findings discussed next.
Patients with barotrauma most commonly report pain. Other symptoms include diminished hearing and tinnitus. Exam may demonstrate bulging or retraction of the TM, hemotympanum, and TM perforation with or without blood in the ear canal. Gross hearing is usually diminished to varying degrees. Tympanoplasty is indicated for large or nonhealing perforations. Myringotomy or tympanostomy tubes can be used in both prevention and treatment of barotrauma but should be avoided when associated with scuba diving due to an increased chance of infection. Myringotomy openings without a tube may heal quicker than the natural course which can result in recurrent pain. Ventilation tubes may be indicated for prevention of injury in those who will be undergoing hyperbaric oxygen therapy or those who have known Eustachian tube dysfunction and must fly frequently. For all other patients, frequent swallowing or yawning during times of pressure differences may help. Other options such as premedicating with antihistamines or decongestants as well as using specialized ear plugs have mixed results.
References:
Incorrect
The correct answer is D. The majority of cases of barotrauma will resolve spontaneously. If a large perforation of the tympanic membrane (TM) has occurred, a tympanoplasty may be indicated, however, these are less common. Antibiotics are only indicated if a tympanic membrane perforation has occurred in the setting of contaminated material. Glucocorticoids have no role in management. Tympanoplasty and tympanostomy tube have a place in management or prevention but are used much less frequently. Antihistamines and decongestants do not seem to speed recovery times but can be used if not contraindicated along with analgesics, which is most helpful.
Barotrauma is a term used to describe an injury to the TM resulting from pressure differences between the environment and the middle ear. Common causes of barotrauma include a rapid ascent or decent during flying, diving, decompression chamber use, hyperbaric oxygen chamber use, close proximity to a blast, sky diving, and upper respiratory tract infections that cause swelling of the nasopharynx at the site of the Eustachian tube orifice or of the Eustachian tube itself. Swallowing or yawning are methods that are used to equalize middle ear and atmospheric pressures but if the force is too large to compensate for, the TM stretches and bends beyond its capabilities causing damage. This damage, known as barotrauma, results in the history and exam findings discussed next.
Patients with barotrauma most commonly report pain. Other symptoms include diminished hearing and tinnitus. Exam may demonstrate bulging or retraction of the TM, hemotympanum, and TM perforation with or without blood in the ear canal. Gross hearing is usually diminished to varying degrees. Tympanoplasty is indicated for large or nonhealing perforations. Myringotomy or tympanostomy tubes can be used in both prevention and treatment of barotrauma but should be avoided when associated with scuba diving due to an increased chance of infection. Myringotomy openings without a tube may heal quicker than the natural course which can result in recurrent pain. Ventilation tubes may be indicated for prevention of injury in those who will be undergoing hyperbaric oxygen therapy or those who have known Eustachian tube dysfunction and must fly frequently. For all other patients, frequent swallowing or yawning during times of pressure differences may help. Other options such as premedicating with antihistamines or decongestants as well as using specialized ear plugs have mixed results.
References:
-
Question 42 of 56
42. Question
A 40-year-old patient reports experiencing typical seasonal allergy symptoms for the past week but is now having left ear discomfort, pressure, and hearing their own voice more in the left ear than the right. Which of the following is most likely to be found on otoscope exam?
Correct
The correct answer is D. Given the patient’s age, seasonal allergy symptoms, and description of symptoms, Eustachian tube dysfunction is the most likely diagnosis. The symptoms are caused by a negative pressure within the middle ear which causes tympanic membrane retraction and a prominence of the malleus along with a shortening of the malleus handle. Absent light reflex and erythematous, sometime bulging tympanic membrane are seen with otitis media, which is an uncommon diagnosis in adults. Tympanic membrane scaring is usually seen in areas that have healed after trauma or tympanostomy tube placement.
Eustachian tube dysfunction occurs when the Eustachian tube orifice, located on the mucosal surface of the nasopharynx, fails to open (obstructive dysfunction) or close (patulous dysfunction). The Eustachian tube connects the tympanic cavity (middle ear) to the nasopharynx with the narrowest point occurring where it passes through the temporal bone and is considered part of the inner ear. Yawning or swallowing will normally open the orifice and regulate pressure. Children have a shorter, more horizontal tube until approximately 6 years of age when it has elongated and angled inferiorly, making middle ear infections much less common. Normally, the tube has a slightly negative pressure when compared to atmospheric. Obstructive dysfunction results in an increased negative pressure while patulous dysfunction allows for the pressure to be closer to atmospheric, which still results in an abnormal sensation for the patient. Obstructive etiologies include infection, allergies, and environmental irritants amongst others. Patulous causes include allergies, weight loss, laryngopharyngeal reflux, and stress amongst others.
In addition to the symptoms mentioned above, patients may also complain of popping, ringing, or a clogged sensation. History and risk factors are important determinants in making the diagnosis since the tympanic membrane can appear normal on otoscope exam, especially in cases of patulous dysfunction where there will be less or no appreciable retraction. Confirmation can be made by an otolaryngologist using nasal endoscopy to evaluate the orifice directly. A CT scan may be necessary for patients with unilateral effusions for greater than 3 months to exclude neoplasm or lymphoma.
Treatment options are targeted at the underlying disorder. Options include antibiotics, decongestants, antihistamines, behavioral modifications, systemic or nasal steroids, tympanostomy, balloon dilation of the Eustachian tube, or shrinking/removing a mass. Cases associated with viruses, environmental allergies, or bacterial infections generally resolve without long-term complications when treated. Potential complications include hearing loss, otitis media, tympanic membrane perforations and cholesteatomas.
References:
- UpToDate
- https://www.ncbi.nlm.nih.gov/books/NBK555908/
- https://www.ncbi.nlm.nih.gov/pmc/articles/PMC4600223/
Incorrect
The correct answer is D. Given the patient’s age, seasonal allergy symptoms, and description of symptoms, Eustachian tube dysfunction is the most likely diagnosis. The symptoms are caused by a negative pressure within the middle ear which causes tympanic membrane retraction and a prominence of the malleus along with a shortening of the malleus handle. Absent light reflex and erythematous, sometime bulging tympanic membrane are seen with otitis media, which is an uncommon diagnosis in adults. Tympanic membrane scaring is usually seen in areas that have healed after trauma or tympanostomy tube placement.
Eustachian tube dysfunction occurs when the Eustachian tube orifice, located on the mucosal surface of the nasopharynx, fails to open (obstructive dysfunction) or close (patulous dysfunction). The Eustachian tube connects the tympanic cavity (middle ear) to the nasopharynx with the narrowest point occurring where it passes through the temporal bone and is considered part of the inner ear. Yawning or swallowing will normally open the orifice and regulate pressure. Children have a shorter, more horizontal tube until approximately 6 years of age when it has elongated and angled inferiorly, making middle ear infections much less common. Normally, the tube has a slightly negative pressure when compared to atmospheric. Obstructive dysfunction results in an increased negative pressure while patulous dysfunction allows for the pressure to be closer to atmospheric, which still results in an abnormal sensation for the patient. Obstructive etiologies include infection, allergies, and environmental irritants amongst others. Patulous causes include allergies, weight loss, laryngopharyngeal reflux, and stress amongst others.
In addition to the symptoms mentioned above, patients may also complain of popping, ringing, or a clogged sensation. History and risk factors are important determinants in making the diagnosis since the tympanic membrane can appear normal on otoscope exam, especially in cases of patulous dysfunction where there will be less or no appreciable retraction. Confirmation can be made by an otolaryngologist using nasal endoscopy to evaluate the orifice directly. A CT scan may be necessary for patients with unilateral effusions for greater than 3 months to exclude neoplasm or lymphoma.
Treatment options are targeted at the underlying disorder. Options include antibiotics, decongestants, antihistamines, behavioral modifications, systemic or nasal steroids, tympanostomy, balloon dilation of the Eustachian tube, or shrinking/removing a mass. Cases associated with viruses, environmental allergies, or bacterial infections generally resolve without long-term complications when treated. Potential complications include hearing loss, otitis media, tympanic membrane perforations and cholesteatomas.
References:
- UpToDate
- https://www.ncbi.nlm.nih.gov/books/NBK555908/
- https://www.ncbi.nlm.nih.gov/pmc/articles/PMC4600223/
-
Question 43 of 56
43. Question
A patient presents complaining of dizziness, specifically, a room-spinning sensation. Exam reveals no obvious facial droop or slurred speech. There is unidirectional nystagmus to the left but no strabismus. A gross hearing test reveals decreased hearing in the right ear. Which of the following is the most appropriate diagnosis?
Correct
The correct answer is B. The history and exam findings are most consistent with a peripheral cause of vertigo. Of the disorders listed, only labyrinthitis and vestibular neuritis are peripheral etiologies. Labyrinthitis is the term given to symptoms consistent with vestibular neuritis with the addition of decreased hearing.
Vestibular neuritis is an inflammatory condition affecting the vestibular portion of the 8th cranial nerve. It is typically considered to be a viral or postviral disorder which can be very helpful in making the diagnosis if considered and asked in the history or review of symptoms. It may also be referred to as vestibular neuronitis, labyrinthitis, neurolabyrinthitis, or acute peripheral vestibulopathy. The term labyrinthitis is used when there is an associated hearing deficit. The primary symptoms a patient reports are nausea, vomiting, a feeling of the room or surrounding environment spinning, and a gait disturbance. The symptoms are typically constant, unlike those associated with benign positional vertigo or Meniere’s disease, which are more episodic. Also, Meniere’s symptoms usually resolve within 12 hours, as opposed to several days for vestibular neuritis.
Physical exam typically reveals a horizontal spontaneous nystagmus (typically with a rotational component) in the direction of the unaffected ear, a corrective saccade when a head-impulse test is performed toward the affected ear, absence of vertical strabismus when performing the cover-uncover test, and a standing imbalance with leaning or falling toward the affected ear. There are no visible abnormalities on otoscope exam.
Work-up is usually unnecessary if the history and symptoms are consistent with the diagnosis. A MRI is indicated if there are any inconsistencies or other concerns. Laboratory tests will not be helpful. Treatment is supportive, using antiemetics, antihistamines, or benzodiazepines but should be limited to no more than 3 days to avoid delaying or preventing the brain from developing compensatory mechanisms. A 10 day taper of prednisone can be considered but studies have been inconclusive as to the benefits. Physical therapy should be employed for persistent or more severe cases. Most patients suffer with severe symptoms for only 1-2 days and then improve over the course of a few days to weeks but some may report problems with balance and dizziness for months. The majority of patients will not have another acute event.
References:
- https://pubmed.ncbi.nlm.nih.gov/19834862/
- UpToDate
- https://www.ncbi.nlm.nih.gov/books/NBK549866/
- https://www.emra.org/emresident/article/hints-exam/
Incorrect
The correct answer is B. The history and exam findings are most consistent with a peripheral cause of vertigo. Of the disorders listed, only labyrinthitis and vestibular neuritis are peripheral etiologies. Labyrinthitis is the term given to symptoms consistent with vestibular neuritis with the addition of decreased hearing.
Vestibular neuritis is an inflammatory condition affecting the vestibular portion of the 8th cranial nerve. It is typically considered to be a viral or postviral disorder which can be very helpful in making the diagnosis if considered and asked in the history or review of symptoms. It may also be referred to as vestibular neuronitis, labyrinthitis, neurolabyrinthitis, or acute peripheral vestibulopathy. The term labyrinthitis is used when there is an associated hearing deficit. The primary symptoms a patient reports are nausea, vomiting, a feeling of the room or surrounding environment spinning, and a gait disturbance. The symptoms are typically constant, unlike those associated with benign positional vertigo or Meniere’s disease, which are more episodic. Also, Meniere’s symptoms usually resolve within 12 hours, as opposed to several days for vestibular neuritis.
Physical exam typically reveals a horizontal spontaneous nystagmus (typically with a rotational component) in the direction of the unaffected ear, a corrective saccade when a head-impulse test is performed toward the affected ear, absence of vertical strabismus when performing the cover-uncover test, and a standing imbalance with leaning or falling toward the affected ear. There are no visible abnormalities on otoscope exam.
Work-up is usually unnecessary if the history and symptoms are consistent with the diagnosis. A MRI is indicated if there are any inconsistencies or other concerns. Laboratory tests will not be helpful. Treatment is supportive, using antiemetics, antihistamines, or benzodiazepines but should be limited to no more than 3 days to avoid delaying or preventing the brain from developing compensatory mechanisms. A 10 day taper of prednisone can be considered but studies have been inconclusive as to the benefits. Physical therapy should be employed for persistent or more severe cases. Most patients suffer with severe symptoms for only 1-2 days and then improve over the course of a few days to weeks but some may report problems with balance and dizziness for months. The majority of patients will not have another acute event.
References:
- https://pubmed.ncbi.nlm.nih.gov/19834862/
- UpToDate
- https://www.ncbi.nlm.nih.gov/books/NBK549866/
- https://www.emra.org/emresident/article/hints-exam/
-
Question 44 of 56
44. Question
A patient presents with complaints of decreased hearing and drainage from the left year. Otoscope exam reveals the findings in the attached image. What is the most likely diagnosis?Correct
The correct answer is B. Barotrauma may result in a perforated tympanic membrane (TM) but not an abnormal growth (both seen in this image). Otitis externa is associated with otorrhea, but the canal should be erythematous and edematous, and neither is present in this image. Otitis media is associated with an erythematous and sometimes bulging TM, again, neither is demonstrated in this image. A cholesteatoma could have the appearance of a foreign body, but it is usually much more irregular in shape than most foreign bodies found in the ear and the provider can usually see an attachment to underlying structures with cholesteatomas.
A cholesteatoma is a benign middle ear mass comprised of keratinized squamous epithelium. It can be congenital or acquired secondary to infection, trauma, or surgery. Congenital etiologies typically occur at around 4-5 years of age. Once the abnormality starts, it will continue to grow and expand. Despite this growth in a relatively small space, these are usually painless but cause hearing loss, drainage, and sometimes dizziness. Cranial nerve palsies can also occur. Otoscope exam can demonstrate an irregularly shaped, white mass with a smooth surface behind the TM, bulging TM (congenital) or deep retraction pockets (acquired), TM perforation, and otorrhea.
The diagnosis is usually confirmed by an otolaryngologist using a microscope or by imaging with CT or MRI including the temporal bone. Treatment usually requires surgical excision and may involve mastoidectomy if the growth has invaded the mastoid. Tympanoplasty is required to repair the TM. If retraction pockets are present, tympanostomy tubes may be utilized. Recurrence is possible. Hearing may be improved after surgery depending on the involvement of surrounding structures.
References:
Incorrect
The correct answer is B. Barotrauma may result in a perforated tympanic membrane (TM) but not an abnormal growth (both seen in this image). Otitis externa is associated with otorrhea, but the canal should be erythematous and edematous, and neither is present in this image. Otitis media is associated with an erythematous and sometimes bulging TM, again, neither is demonstrated in this image. A cholesteatoma could have the appearance of a foreign body, but it is usually much more irregular in shape than most foreign bodies found in the ear and the provider can usually see an attachment to underlying structures with cholesteatomas.
A cholesteatoma is a benign middle ear mass comprised of keratinized squamous epithelium. It can be congenital or acquired secondary to infection, trauma, or surgery. Congenital etiologies typically occur at around 4-5 years of age. Once the abnormality starts, it will continue to grow and expand. Despite this growth in a relatively small space, these are usually painless but cause hearing loss, drainage, and sometimes dizziness. Cranial nerve palsies can also occur. Otoscope exam can demonstrate an irregularly shaped, white mass with a smooth surface behind the TM, bulging TM (congenital) or deep retraction pockets (acquired), TM perforation, and otorrhea.
The diagnosis is usually confirmed by an otolaryngologist using a microscope or by imaging with CT or MRI including the temporal bone. Treatment usually requires surgical excision and may involve mastoidectomy if the growth has invaded the mastoid. Tympanoplasty is required to repair the TM. If retraction pockets are present, tympanostomy tubes may be utilized. Recurrence is possible. Hearing may be improved after surgery depending on the involvement of surrounding structures.
References:
-
Question 45 of 56
45. Question
A child is brought by parents for evaluation of frequent crying, fever, and pulling at the right ear. The child has no PMH, is up-to-date with vaccinations, has no allergies, and weighs 10 kg. Exam of the ear is shown in the attached image. What is the most appropriate treatment option?Correct
The correct answer is B. The history along with the appearance of a bulging, erythematous tympanic membrane with no well-defined light reflex is consistent with an otitis media. The recommended initial antibiotic and starting dose is amoxicillin at 90 mg/kg/day divided BID for 10 days. Cases refractory to amoxicillin or in areas where beta-lactamase-producing Haemophilus influenzae are prevalent, amoxicillin plus clavulanate (Augmentin) at 90 mg/kg/day divided BID for 10 days is recommended. Azithromycin (Zithromax) is reasonable if a significant penicillin allergy is present but are not as effective against H. flu and S. pneumoniae and should be dosed at 10 mg/kg once on day one and then 5 mg/kg daily for days 2-5. While there is a small amount of cerumen present in this patient’s ear, it is not the most abnormal finding and would not be consistent with the history provided.
Acute otitis media (AOM) is the result of bacteria (most commonly S. pneumoniae, H. influenzae, and Moraxella catarrhalis) trapped within the middle ear, usually secondary to inflammation and edema from a recent viral upper respiratory tract infection. AOM most frequently occurs in children age 6-12 months, with boys slightly more affected than girls, and declines significantly after approximately 6 years of age when the Eustachian tube develops a more inferior slant. Risk factors include family history, day care attendance, second-hand smoke exposure, pacifier use, and Native American race. The primary protective factor is breastfeeding, especially if done for at least 6 months.
Pain or fussiness are the most common complaints. Physical exam usually demonstrates some degree of tympanic membrane bulging. There may be air-fluid levels visible behind the membrane. The membrane itself may appear cloudy, opaque, white/yellow (pus in middle ear), erythematous (inflammation or blood), or be perforated. Performing pneumatic otoscopy using an insufflator bulb can be performed to confirm decreased or absent mobility but this can be challenging in uncooperative patients.
Low risk patients (> 6 months old, immunocompetent, non-toxic appearing, absence of cleft palate) can be treated with a “wait and see” approach for up to 72 hours. If symptoms spontaneously improve during that time, antibiotics can be avoided. In addition to the antibiotic discussion above, other acceptable antibiotics include cephalosporins (cefdinir, cefpodoxime, cefuroxime, ceftriaxone), clarithromycin, clindamycin, and trimethoprim-sulfamethoxazole. Ibuprofen and/or acetaminophen should be provided for comfort and fever. Topical lidocaine can also be used in children over 2. Antihistamines and decongestants do not show any benefit in treating AOM. Complications include hearing loss, imbalance, tympanic membrane perforation, myringosclerosis/tympanosclerosis, cholesteatoma, mastoiditis, and infections of the brain or meninges. Indications for placement of tympanostomy tubes include: symptoms > 3 months, recurrence (3 episodes in 6 months or 4 episodes in 12 months), or at-risk children.
References:
Incorrect
The correct answer is B. The history along with the appearance of a bulging, erythematous tympanic membrane with no well-defined light reflex is consistent with an otitis media. The recommended initial antibiotic and starting dose is amoxicillin at 90 mg/kg/day divided BID for 10 days. Cases refractory to amoxicillin or in areas where beta-lactamase-producing Haemophilus influenzae are prevalent, amoxicillin plus clavulanate (Augmentin) at 90 mg/kg/day divided BID for 10 days is recommended. Azithromycin (Zithromax) is reasonable if a significant penicillin allergy is present but are not as effective against H. flu and S. pneumoniae and should be dosed at 10 mg/kg once on day one and then 5 mg/kg daily for days 2-5. While there is a small amount of cerumen present in this patient’s ear, it is not the most abnormal finding and would not be consistent with the history provided.
Acute otitis media (AOM) is the result of bacteria (most commonly S. pneumoniae, H. influenzae, and Moraxella catarrhalis) trapped within the middle ear, usually secondary to inflammation and edema from a recent viral upper respiratory tract infection. AOM most frequently occurs in children age 6-12 months, with boys slightly more affected than girls, and declines significantly after approximately 6 years of age when the Eustachian tube develops a more inferior slant. Risk factors include family history, day care attendance, second-hand smoke exposure, pacifier use, and Native American race. The primary protective factor is breastfeeding, especially if done for at least 6 months.
Pain or fussiness are the most common complaints. Physical exam usually demonstrates some degree of tympanic membrane bulging. There may be air-fluid levels visible behind the membrane. The membrane itself may appear cloudy, opaque, white/yellow (pus in middle ear), erythematous (inflammation or blood), or be perforated. Performing pneumatic otoscopy using an insufflator bulb can be performed to confirm decreased or absent mobility but this can be challenging in uncooperative patients.
Low risk patients (> 6 months old, immunocompetent, non-toxic appearing, absence of cleft palate) can be treated with a “wait and see” approach for up to 72 hours. If symptoms spontaneously improve during that time, antibiotics can be avoided. In addition to the antibiotic discussion above, other acceptable antibiotics include cephalosporins (cefdinir, cefpodoxime, cefuroxime, ceftriaxone), clarithromycin, clindamycin, and trimethoprim-sulfamethoxazole. Ibuprofen and/or acetaminophen should be provided for comfort and fever. Topical lidocaine can also be used in children over 2. Antihistamines and decongestants do not show any benefit in treating AOM. Complications include hearing loss, imbalance, tympanic membrane perforation, myringosclerosis/tympanosclerosis, cholesteatoma, mastoiditis, and infections of the brain or meninges. Indications for placement of tympanostomy tubes include: symptoms > 3 months, recurrence (3 episodes in 6 months or 4 episodes in 12 months), or at-risk children.
References:
-
Question 46 of 56
46. Question
. Most cases of traumatic tympanic membrane perforation are best managed with which of the following options?
Correct
The correct answer is C. The majority of tympanic membrane (TM) perforations will heal spontaneously. Antibiotic solutions can speed up the rate of healing but do not improve hearing outcomes. Perforations located in the posteriorsuperior aspect of the TM are an indication for possible tympanoplasty due to poor healing. Patient’s should be instructed to keep the ear as dry as possible in order to avoid a possible secondary acute otitis media infection.
The TM is a thin, three-layered structure which separates the middle from the external ear. In its normal state, it is semi-translucent, white/reflective, and flexible. Perforation can occur due to infection, mechanical trauma, or barotrauma. Common causes of mechanical trauma include a slap to the side of the head covering the ear, nearby blast, cotton swab or other faulty method of attempting to clean the ear. Symptoms include pain, diminished hearing, bleeding or discharge, ringing (tinnitus), and/or dizziness. Physical exam can show a wide spectrum of perforation sizes depending on the underlying etiology. Bleeding, discharge, or signs of otitis media or cholesteatoma may be present.
Approximately 90% of perforations will spontaneously heal within 6 weeks, with smaller ones healing within 3-4 weeks. If antibiotic solutions are prescribed, aminoglycosides (the “mycins”) should be avoided due to possible ototoxicity. Ophthalmic antibiotic drops can be substituted for otic preparations and may be better tolerated since they are less acidic. Keeping the ear dry is important to emphasize to the patient, as well as avoiding inserting any foreign bodies such as cotton swabs, hearing aids, in-ear speakers, etc. Repair of the TM, tympanoplasty, may be indicated if perforation is secondary to a cholesteatoma, to restore significant hearing loss, or to repair a chronic non-healing perforation in order to minimize the risk of developing otitis media. Complications include developing chronic otitis media and permanent hearing loss. Other complications such as mastoiditis and facial nerve involvement are usually related to secondary infections.
References:
Incorrect
The correct answer is C. The majority of tympanic membrane (TM) perforations will heal spontaneously. Antibiotic solutions can speed up the rate of healing but do not improve hearing outcomes. Perforations located in the posteriorsuperior aspect of the TM are an indication for possible tympanoplasty due to poor healing. Patient’s should be instructed to keep the ear as dry as possible in order to avoid a possible secondary acute otitis media infection.
The TM is a thin, three-layered structure which separates the middle from the external ear. In its normal state, it is semi-translucent, white/reflective, and flexible. Perforation can occur due to infection, mechanical trauma, or barotrauma. Common causes of mechanical trauma include a slap to the side of the head covering the ear, nearby blast, cotton swab or other faulty method of attempting to clean the ear. Symptoms include pain, diminished hearing, bleeding or discharge, ringing (tinnitus), and/or dizziness. Physical exam can show a wide spectrum of perforation sizes depending on the underlying etiology. Bleeding, discharge, or signs of otitis media or cholesteatoma may be present.
Approximately 90% of perforations will spontaneously heal within 6 weeks, with smaller ones healing within 3-4 weeks. If antibiotic solutions are prescribed, aminoglycosides (the “mycins”) should be avoided due to possible ototoxicity. Ophthalmic antibiotic drops can be substituted for otic preparations and may be better tolerated since they are less acidic. Keeping the ear dry is important to emphasize to the patient, as well as avoiding inserting any foreign bodies such as cotton swabs, hearing aids, in-ear speakers, etc. Repair of the TM, tympanoplasty, may be indicated if perforation is secondary to a cholesteatoma, to restore significant hearing loss, or to repair a chronic non-healing perforation in order to minimize the risk of developing otitis media. Complications include developing chronic otitis media and permanent hearing loss. Other complications such as mastoiditis and facial nerve involvement are usually related to secondary infections.
References:
-
Question 47 of 56
47. Question
What is the recommended age to begin screening for hearing impairment in a low-risk child who had a normal routine newborn screening?
Correct
The correct answer is C. All newborns should be screened within the first month of life. If the initial screening was unremarkable and the child is considered low-risk, the next screening should occur around age 4 and then annually through age 8-10. If screenings have been unremarkable, adolescents should be screened once between ages 11-14, 15-17, and 18-21. If the neonatal assessment was abnormal, a more detailed audiologic assessment should occur by 3 months and intervention should occur by 6 months of age if significant impairment has been detected. Risk factors for developing hearing loss in the neonatal period include delivery complications, the need for NICU admission of at least 5 days, maternal infections which can be transmitted to the fetus/newborn, abnormal development of the skull or ear, hyperbilirubinemia > 35 mg/dL, family history, various genetic syndromes known to result in hearing loss, and use of ototoxic medications.
In addition to congenital and genetic risk factors, other etiologies of hearing loss include otitis media/externa, trauma, masses (carcinoma, osteoma, polyps, exostosis, cholesteatoma), diabetes, psoriasis, cerumen impaction, otosclerosis, tympanic membrane perforation, barotrauma, chronic noise exposure, Meniere disease, stroke, multiple sclerosis, use of ototoxic drugs (antibiotics, chemotherapy, aspirin), and presbycusis (age-related). A history from the patient or family members/friends may report concerns about a decline in hearing, although it may be subtle to the patient or spouse/partner if it has occurred over an extended period of time.
Basic screening can include a whisper test or finger rub, where the provider creates a soft sound in the close proximity to one ear and then the other to see if the patient can detect it. Using a 256 Hz tuning fork to perform both a Weber and Rinne test can also be useful in differentiating between sensorineural and conductive etiologies of hearing loss. The Weber test is performed by placing a vibrating fork on the top, middle of the scalp and asking if the sound is heard equally in both ears. The Rinne is then performed by vibrating the fork again and placing the handle on the mastoid, then moving the body of the fork directly over the canal and asking which seemed louder to the patient. The second step of the Rinne, “air conduction”, should be lower than the first, “bone conduction”. Both tests should be performed in order to interpret the results. If the patient reports louder hearing in one ear on Weber and the Rinne is normal, the provider should perform the Rinne in the opposite ear. If bone conduction is greater than air in the opposite ear, sensorineural hearing loss in that ear is likely. If the Weber test is louder in one ear and the Rinne produces bone conduction greater than air in that same ear, a conductive hearing loss is present in that ear.
When a hearing deficit is detected, an otoscopic exam should be performed to evaluate for any potential causes and when possible, pneumoscopy should be performed to evaluate for TM mobility. Other tools used in the evaluation of hearing loss include pure tone testing, speech audiometry, and impedance audiometry. A thorough history and exam as well as any laboratory testing or imaging based on the differential diagnosis should also be performed so that the etiology of the deficit is addressed.
References:
- UpToDate
- https://www.who.int/news-room/fact-sheets/detail/deafness-and-hearing-loss
- https://www.aafp.org/afp/2019/0715/p98.html
Incorrect
The correct answer is C. All newborns should be screened within the first month of life. If the initial screening was unremarkable and the child is considered low-risk, the next screening should occur around age 4 and then annually through age 8-10. If screenings have been unremarkable, adolescents should be screened once between ages 11-14, 15-17, and 18-21. If the neonatal assessment was abnormal, a more detailed audiologic assessment should occur by 3 months and intervention should occur by 6 months of age if significant impairment has been detected. Risk factors for developing hearing loss in the neonatal period include delivery complications, the need for NICU admission of at least 5 days, maternal infections which can be transmitted to the fetus/newborn, abnormal development of the skull or ear, hyperbilirubinemia > 35 mg/dL, family history, various genetic syndromes known to result in hearing loss, and use of ototoxic medications.
In addition to congenital and genetic risk factors, other etiologies of hearing loss include otitis media/externa, trauma, masses (carcinoma, osteoma, polyps, exostosis, cholesteatoma), diabetes, psoriasis, cerumen impaction, otosclerosis, tympanic membrane perforation, barotrauma, chronic noise exposure, Meniere disease, stroke, multiple sclerosis, use of ototoxic drugs (antibiotics, chemotherapy, aspirin), and presbycusis (age-related). A history from the patient or family members/friends may report concerns about a decline in hearing, although it may be subtle to the patient or spouse/partner if it has occurred over an extended period of time.
Basic screening can include a whisper test or finger rub, where the provider creates a soft sound in the close proximity to one ear and then the other to see if the patient can detect it. Using a 256 Hz tuning fork to perform both a Weber and Rinne test can also be useful in differentiating between sensorineural and conductive etiologies of hearing loss. The Weber test is performed by placing a vibrating fork on the top, middle of the scalp and asking if the sound is heard equally in both ears. The Rinne is then performed by vibrating the fork again and placing the handle on the mastoid, then moving the body of the fork directly over the canal and asking which seemed louder to the patient. The second step of the Rinne, “air conduction”, should be lower than the first, “bone conduction”. Both tests should be performed in order to interpret the results. If the patient reports louder hearing in one ear on Weber and the Rinne is normal, the provider should perform the Rinne in the opposite ear. If bone conduction is greater than air in the opposite ear, sensorineural hearing loss in that ear is likely. If the Weber test is louder in one ear and the Rinne produces bone conduction greater than air in that same ear, a conductive hearing loss is present in that ear.
When a hearing deficit is detected, an otoscopic exam should be performed to evaluate for any potential causes and when possible, pneumoscopy should be performed to evaluate for TM mobility. Other tools used in the evaluation of hearing loss include pure tone testing, speech audiometry, and impedance audiometry. A thorough history and exam as well as any laboratory testing or imaging based on the differential diagnosis should also be performed so that the etiology of the deficit is addressed.
References:
- UpToDate
- https://www.who.int/news-room/fact-sheets/detail/deafness-and-hearing-loss
- https://www.aafp.org/afp/2019/0715/p98.html
-
Question 48 of 56
48. Question
An 18-month-old male is brought for evaluation of fever and fussiness. The child is fully vaccinated and has no past medical history. On exam, there is swelling and noticeable pain to palpation behind the left ear over the mastoid prominence. What is the most likely underlying etiology that led to this finding?
Correct
The correct answer is C. The patient has a history and exam concerning for acute mastoiditis. The most common etiology which leads to mastoiditis is otitis media. Cholesteatoma and otitis externa can also cause mastoiditis but are much less common. An isolated cellulitis located directly over the mastoid would be especially unusual, especially in an otherwise healthy patient and should still be considered mastoiditis until proven otherwise. Ramsey Hunt syndrome is a zoster infection of cranial nerve VII (facial nerve) which results in vesicular eruptions to the auricle, auditory canal, scalp, cheek, tongue, or palate area. It can also be accompanied by a fever, ear pain, and facial paralysis.
Mastoiditis is an infection involving the air cells within mastoid portion of the temporal bone. This area is represented by a prominence located directly behind the inferior aspect of the ear. This area of the temporal bone arises from the middle ear at birth, and both aerates and enlarges as a human grows. This connection explains why a middle ear infection (otitis media) is the primary cause of mastoiditis. Most cases occur in children less than 2-years-old. Infection results in pain, focal erythema and edema, and possibly anterior displacement of the auricle. There are also findings consistent with otitis media. Complications can include abscess formation, facial nerve paralysis, decreased hearing, dizziness, and even more invasive infections such as meningitis, epidural or subdural abscess, and venous sinus thrombosis.
The diagnosis is usually made clinically. A CT scan can be used to evaluate the extent of the infection or any complications. Treatment starts with IV antibiotics, usually accompanied by myringotomy, with or without tympanostomy tubes. Vancomycin or linezolid are recommended choices for uncomplicated cases or those with no recent antibiotic use. Chronic recurrence should receive a cephalosporin plus vancomycin or linezolid. Mastoidectomy is indicated for more serious cases. Most cases will resolve without complications when given antibiotics. Hearing loss or permanent neurologic defects can occur in more advanced cases.
References:
Incorrect
The correct answer is C. The patient has a history and exam concerning for acute mastoiditis. The most common etiology which leads to mastoiditis is otitis media. Cholesteatoma and otitis externa can also cause mastoiditis but are much less common. An isolated cellulitis located directly over the mastoid would be especially unusual, especially in an otherwise healthy patient and should still be considered mastoiditis until proven otherwise. Ramsey Hunt syndrome is a zoster infection of cranial nerve VII (facial nerve) which results in vesicular eruptions to the auricle, auditory canal, scalp, cheek, tongue, or palate area. It can also be accompanied by a fever, ear pain, and facial paralysis.
Mastoiditis is an infection involving the air cells within mastoid portion of the temporal bone. This area is represented by a prominence located directly behind the inferior aspect of the ear. This area of the temporal bone arises from the middle ear at birth, and both aerates and enlarges as a human grows. This connection explains why a middle ear infection (otitis media) is the primary cause of mastoiditis. Most cases occur in children less than 2-years-old. Infection results in pain, focal erythema and edema, and possibly anterior displacement of the auricle. There are also findings consistent with otitis media. Complications can include abscess formation, facial nerve paralysis, decreased hearing, dizziness, and even more invasive infections such as meningitis, epidural or subdural abscess, and venous sinus thrombosis.
The diagnosis is usually made clinically. A CT scan can be used to evaluate the extent of the infection or any complications. Treatment starts with IV antibiotics, usually accompanied by myringotomy, with or without tympanostomy tubes. Vancomycin or linezolid are recommended choices for uncomplicated cases or those with no recent antibiotic use. Chronic recurrence should receive a cephalosporin plus vancomycin or linezolid. Mastoidectomy is indicated for more serious cases. Most cases will resolve without complications when given antibiotics. Hearing loss or permanent neurologic defects can occur in more advanced cases.
References:
-
Question 49 of 56
49. Question
A patient reports a persistent, non-pulsatile “buzzing” sound in the right ear for several months. Examination of the ear is unremarkable and the patient seems to have equal hearing when evaluating both ears with a finger rub test. Which test would be most useful in determining the underlying cause?
Correct
The correct answer is A. Had the patient reported a pulsatile quality to the buzzing, angiography via CT or MRI would be the appropriate initial choice since a pulsatile quality is more frequently associated with a vascular etiology. Audiometric testing can include an audiogram, tympanometry, auditory reflex, and otoacoustic emissions tests. The combination of these tests can help determine if abnormalities exist within the middle ear, cochlea, and brainstem. The Weber and Rinne tests are useful in helping to differentiate between sensorineural and conductive causes of hearing loss. Since there is not significant complaint or finding of hearing deficit, the test would be less useful. A tilt table test is mostly used in the work-up of syncope.
Tinnitus is a term used for an abnormal perception of sound when no environmental noise is present. Some terms a patient may use to describe their symptoms include ringing, buzzing, whooshing, humming, hissing, throbbing, or even a musical quality to the noise. Tinnitus noise can be constant or intermittent as well as having a pulsatile or non-pulsatile quality. Tinnitus effects a significant amount of the population and can be very debilitating to a person’s quality of life. Most cases of tinnitus are related to sensorineural hearing loss resulting from use of ototoxic medications, age (presbycusis), otosclerosis, vestibular schwannoma, or Chiari malformation. Other potential etiologies are vast and includes a number of different vascular disorders, spasm of inner ear muscles, Eustachian tube dysfunction, and TMJ dysfunction. The list of potential etiologies can be narrowed down using a detailed history. Tinnitus described as “rushing/whooshing”, “humming”, or that is pulsatile or positional is frequently associated with a vascular cause. A “clicking” quality is usually related to middle ear muscle spasms. While a high-pitched noise is likely sensorineural and a lower-pitched sound is commonly associated with Meniere disease. Physical exam should include a detailed ear and neurologic exam as well as assessing for improved or worsening symptoms with jaw and head range of motion as well as changes in body position. Auscultation for bruits over the neck, in front of the ear as well as the temple and mastoid areas should be performed to detect a potential vascular cause.
Management options are targeted at addressing the underlying cause, if possible as well as preventing progressing by limiting loud noise exposure, ototoxic medications, trauma, etc. Some studies show that melatonin may be useful. Patients with underlying mental health disorders may benefit from tricyclics or SSRI’s but these are not routinely recommended for patients without this history. Tinnitus retraining and/or cognitive-behavioral therapies may also be beneficial.
References:
Incorrect
The correct answer is A. Had the patient reported a pulsatile quality to the buzzing, angiography via CT or MRI would be the appropriate initial choice since a pulsatile quality is more frequently associated with a vascular etiology. Audiometric testing can include an audiogram, tympanometry, auditory reflex, and otoacoustic emissions tests. The combination of these tests can help determine if abnormalities exist within the middle ear, cochlea, and brainstem. The Weber and Rinne tests are useful in helping to differentiate between sensorineural and conductive causes of hearing loss. Since there is not significant complaint or finding of hearing deficit, the test would be less useful. A tilt table test is mostly used in the work-up of syncope.
Tinnitus is a term used for an abnormal perception of sound when no environmental noise is present. Some terms a patient may use to describe their symptoms include ringing, buzzing, whooshing, humming, hissing, throbbing, or even a musical quality to the noise. Tinnitus noise can be constant or intermittent as well as having a pulsatile or non-pulsatile quality. Tinnitus effects a significant amount of the population and can be very debilitating to a person’s quality of life. Most cases of tinnitus are related to sensorineural hearing loss resulting from use of ototoxic medications, age (presbycusis), otosclerosis, vestibular schwannoma, or Chiari malformation. Other potential etiologies are vast and includes a number of different vascular disorders, spasm of inner ear muscles, Eustachian tube dysfunction, and TMJ dysfunction. The list of potential etiologies can be narrowed down using a detailed history. Tinnitus described as “rushing/whooshing”, “humming”, or that is pulsatile or positional is frequently associated with a vascular cause. A “clicking” quality is usually related to middle ear muscle spasms. While a high-pitched noise is likely sensorineural and a lower-pitched sound is commonly associated with Meniere disease. Physical exam should include a detailed ear and neurologic exam as well as assessing for improved or worsening symptoms with jaw and head range of motion as well as changes in body position. Auscultation for bruits over the neck, in front of the ear as well as the temple and mastoid areas should be performed to detect a potential vascular cause.
Management options are targeted at addressing the underlying cause, if possible as well as preventing progressing by limiting loud noise exposure, ototoxic medications, trauma, etc. Some studies show that melatonin may be useful. Patients with underlying mental health disorders may benefit from tricyclics or SSRI’s but these are not routinely recommended for patients without this history. Tinnitus retraining and/or cognitive-behavioral therapies may also be beneficial.
References:
-
Question 50 of 56
50. Question
The attached image is an example of which device used for foreign body removal?Correct
The correct answer is B. Ear curette’s come in a variety of shapes, sizes and materials. They can have angles at the end of the shaft that may be beneficial when removing a foreign body as well as have a scoop or an open loop at the end. Some are made of flexible plastic. Some are more rigid plastic and others are metal. Each has it’s own advantage or disadvantage depending on the size and shape of the foreign body as well as patient anatomy. The remaining options are all examples of different types of devices used to remove foreign bodies in the nose or ear canal with the Katz extractor being used less for ear foreign bodies.
The vast majority of ear foreign bodies involve the ear canal, which will be discussed here. Other foreign body issues involving the ear are related to impacted/embedded piercings of the pinna. Most foreign bodies of the ear canal occur in children less than 6-7 years of age or in special needs adults or children. Outside of this patient population, ear canal foreign bodies are usually the result of insects or accidents/trauma. In children, a retained foreign body may be asymptomatic and the parent may or may not be aware of anything being intentionally placed into the canal if they did not visualize it or were not informed by the child. Once symptomatic, the parent may notice the child seeming to be in pain, have decreased hearing, or visible discharge from the ear which prompts them to see attention.
Physical exam should include an evaluation of both canals and both nares to ensure that nothing is missed. An otoscope is generally used for the initial inspection of both. If an abnormality is identified, the provider should make sure they are visualizing a true foreign body or wax and not a mass, polyp or cholesteatoma before attempting to remove it. The choice of equipment and method of removal varies depending on the equipment available, type of foreign material, depth into the canal, patient anatomy, patient cooperation, and provider comfort level. Referral to otolaryngology should be made on an individual basis. An ear curette is designed to scoop the foreign body out or to be slid behind it and then pushing the object out of the canal, being careful not to push it further down the canal or scratch the canal itself. Irrigation is one method that can be attempted with little concern about subsequent injury to the patient. Commercial devices are available or a basic syringe equipped with butterfly tubing or a large bore angiocath without the needle can easily be made. Irrigate with warm water to avoid injury from hot water or vertigo induced from cold. Alligator forceps are small and thin and can be utilized to grasp part of the foreign body but do not work well on larger, harder, or more rounded objects. A day hook can be used if there is a central opening, like a bead. The Schuknecht foreign body extractor is attached to suction and may be able to create enough of a seal to retrieve the object. Super glue can be applied to the tip of a cotton swab and held to the foreign body for approximately 60 seconds and then retracted but may not work on all materials and care must be taken to avoid touching it to the canal itself. Insects that are still alive can be killed by instilling either mineral oil or lidocaine and then removed by one of the methods mentioned above. Most patients do not require additional treatment after successful removal. If signs of canal infection are present, topical antibiotic preparations are usually adequate and the infection will quickly resolve once the foreign body has been removed.
References:
Incorrect
The correct answer is B. Ear curette’s come in a variety of shapes, sizes and materials. They can have angles at the end of the shaft that may be beneficial when removing a foreign body as well as have a scoop or an open loop at the end. Some are made of flexible plastic. Some are more rigid plastic and others are metal. Each has it’s own advantage or disadvantage depending on the size and shape of the foreign body as well as patient anatomy. The remaining options are all examples of different types of devices used to remove foreign bodies in the nose or ear canal with the Katz extractor being used less for ear foreign bodies.
The vast majority of ear foreign bodies involve the ear canal, which will be discussed here. Other foreign body issues involving the ear are related to impacted/embedded piercings of the pinna. Most foreign bodies of the ear canal occur in children less than 6-7 years of age or in special needs adults or children. Outside of this patient population, ear canal foreign bodies are usually the result of insects or accidents/trauma. In children, a retained foreign body may be asymptomatic and the parent may or may not be aware of anything being intentionally placed into the canal if they did not visualize it or were not informed by the child. Once symptomatic, the parent may notice the child seeming to be in pain, have decreased hearing, or visible discharge from the ear which prompts them to see attention.
Physical exam should include an evaluation of both canals and both nares to ensure that nothing is missed. An otoscope is generally used for the initial inspection of both. If an abnormality is identified, the provider should make sure they are visualizing a true foreign body or wax and not a mass, polyp or cholesteatoma before attempting to remove it. The choice of equipment and method of removal varies depending on the equipment available, type of foreign material, depth into the canal, patient anatomy, patient cooperation, and provider comfort level. Referral to otolaryngology should be made on an individual basis. An ear curette is designed to scoop the foreign body out or to be slid behind it and then pushing the object out of the canal, being careful not to push it further down the canal or scratch the canal itself. Irrigation is one method that can be attempted with little concern about subsequent injury to the patient. Commercial devices are available or a basic syringe equipped with butterfly tubing or a large bore angiocath without the needle can easily be made. Irrigate with warm water to avoid injury from hot water or vertigo induced from cold. Alligator forceps are small and thin and can be utilized to grasp part of the foreign body but do not work well on larger, harder, or more rounded objects. A day hook can be used if there is a central opening, like a bead. The Schuknecht foreign body extractor is attached to suction and may be able to create enough of a seal to retrieve the object. Super glue can be applied to the tip of a cotton swab and held to the foreign body for approximately 60 seconds and then retracted but may not work on all materials and care must be taken to avoid touching it to the canal itself. Insects that are still alive can be killed by instilling either mineral oil or lidocaine and then removed by one of the methods mentioned above. Most patients do not require additional treatment after successful removal. If signs of canal infection are present, topical antibiotic preparations are usually adequate and the infection will quickly resolve once the foreign body has been removed.
References:
-
Question 51 of 56
51. Question
What is the most common etiology of malignant tumors of the external auditory canal?
Correct
The correct answer is E. Squamous cell carcinomas are the most common ear canal cancers followed by adenoid cystic carcinoma and then basal cell carcinoma. Melanoma and Kaposi sarcoma are less common. Ceruminous adenomas are generally not malignant, and cholesteatomas are both not malignant and also are found in the middle ear as opposed to the canal.
Risk factors for squamous cell carcinomas of the external auditory canal are primarily chronic, recurrent otitis media, or externa as well as radiation exposure due to treatment of prior head and neck cancers. The closer the lesion occurs to the opening of the canal and the pinna, the more likely that sun exposure or frostbite damage are the cause. Tobacco and alcohol usage are the most common causes of head and neck cancers in general. These lesions generally grow slowly are unlikely metastasize, but they can expand to occlude the canal, push into the tympanic membrane, or erode into the temporal bone if left untreated. If metastasis does occur, cervical lymph nodes are the most likely location. Patients may complain of pain, drainage, or hearing changes. Bleeding is possible and symptoms related to facial nerve involvement are as well. Cases may be initially misdiagnosed as otitis externa, suppurative otitis media, or cholesteatoma. Exam may reveal a friable, irregular mass of the canal as well as drainage or bleeding. If the patient has delayed evaluation or has previously been misdiagnosed, cervical adenopathy, facial paralysis, or a temporal mass may be noticed.
Work up can include a CT or MRI and biopsy. Treatment involves surgical resection to clean borders followed by radiation and possibly chemotherapy. More advanced cases may require mastoidectomy, temporal bone resection, tympanic membrane resection, or lymph node excision. If the temporal bone is involved, the 5-year survival rate ranges from 20-70% depending on how far the disease has progressed.
References:
- https://www.hindawi.com/journals/criot/2011/615210/
- https://www.ncbi.nlm.nih.gov/pmc/articles/PMC5340731/
- https://www.ncbi.nlm.nih.gov/pmc/articles/PMC6081282/
- https://www.ncbi.nlm.nih.gov/pmc/articles/PMC3936559/
- https://medicine.uiowa.edu/iowaprotocols/carcinoma-external-auditory-canal-and-middle-ear
Incorrect
The correct answer is E. Squamous cell carcinomas are the most common ear canal cancers followed by adenoid cystic carcinoma and then basal cell carcinoma. Melanoma and Kaposi sarcoma are less common. Ceruminous adenomas are generally not malignant, and cholesteatomas are both not malignant and also are found in the middle ear as opposed to the canal.
Risk factors for squamous cell carcinomas of the external auditory canal are primarily chronic, recurrent otitis media, or externa as well as radiation exposure due to treatment of prior head and neck cancers. The closer the lesion occurs to the opening of the canal and the pinna, the more likely that sun exposure or frostbite damage are the cause. Tobacco and alcohol usage are the most common causes of head and neck cancers in general. These lesions generally grow slowly are unlikely metastasize, but they can expand to occlude the canal, push into the tympanic membrane, or erode into the temporal bone if left untreated. If metastasis does occur, cervical lymph nodes are the most likely location. Patients may complain of pain, drainage, or hearing changes. Bleeding is possible and symptoms related to facial nerve involvement are as well. Cases may be initially misdiagnosed as otitis externa, suppurative otitis media, or cholesteatoma. Exam may reveal a friable, irregular mass of the canal as well as drainage or bleeding. If the patient has delayed evaluation or has previously been misdiagnosed, cervical adenopathy, facial paralysis, or a temporal mass may be noticed.
Work up can include a CT or MRI and biopsy. Treatment involves surgical resection to clean borders followed by radiation and possibly chemotherapy. More advanced cases may require mastoidectomy, temporal bone resection, tympanic membrane resection, or lymph node excision. If the temporal bone is involved, the 5-year survival rate ranges from 20-70% depending on how far the disease has progressed.
References:
- https://www.hindawi.com/journals/criot/2011/615210/
- https://www.ncbi.nlm.nih.gov/pmc/articles/PMC5340731/
- https://www.ncbi.nlm.nih.gov/pmc/articles/PMC6081282/
- https://www.ncbi.nlm.nih.gov/pmc/articles/PMC3936559/
- https://medicine.uiowa.edu/iowaprotocols/carcinoma-external-auditory-canal-and-middle-ear
-
Question 52 of 56
52. Question
An adult patient presents for epistaxis. Physical exam appears consistent with an anterior cause. What is the most appropriate number of minutes that the patient should be instructed to pinch the nose in an attempt to stop the bleeding?
Correct
The correct answer is C. For most patients without a coagulopathy, 10-15 minutes is adequate to stop epistaxis. The patient should be asked to gently blow the nose into gauze or tissue in order to clear it of clots. The provider should then immediately spray oxymetazoline (Afrin) into the side of the bleeding and then have the patient tightly pinch the nares together, up against the septum. Firm, constant pressure should be maintained for at least 10 minutes, at which point the nares can be reassessed to see if additional methods of control are necessary (see below).
The etiology of epistaxis can include trauma (accidental or finger picking), dry air, rhinitis, cocaine use, coagulopathy (drug induces or genetic), friable neoplasm or vascular lesion, hypertension, alcoholism, and intranasal steroids. The vast majority (90%) of cases are considered anterior with the rest termed posterior. The majority of anterior bleeds occur in the anatomic area known as Kiesselbach’s plexus or Little’s area which is comprised of 5 arteries which supply the septum. Posterior bleeds arise from 2 arteries located in an area termed Woodruff’s plexus and can be more difficult to control due to the anatomy. They may present with more bilateral epistaxis as well as swallowing blood.
Physical exam should include a thorough evaluation of the nares with adequate lighting if bleeding volume allows after being cleared of clots. A nasal speculum inserted to open superiorly and inferiorly (as opposed to medially and laterally) can be utilized to assist in the evaluation. Suctioning can also be used. A variety of procedures can be used if bleeding does not stop after 10-15 minutes of pressure, depending on the severity and location of bleeding as well as provider comfort level. Silver nitrate can be used to seal a vessel if bleeding is minimal or has stopped. It will not work for briskly bleeding sources. Manual packing can be performed with a variety of materials including nasal tampons, petroleum impregnated gauze ribbon, or balloon catheter. Topical thrombogenics or tranexamic acid are also available. If a posterior bleed is the cause, an appropriately sized balloon device or a Foley catheter balloon inflated once in the posterior cavity are required. Acute hypertension should be controlled, and blood thinner or NSAIDS stopped after consulting with the prescribing provider. If bleeding is not controlled, operative ligation by ENT may be necessary.
Routine antibiotics are no longer recommended to prevent potential toxic shock but can be used for patients at risk or for posterior bleeds. Anterior nosebleeds should be seen, ideally by ENT, in follow-up in 3-5 days if packing was required. Longer if the patient has a coagulopathy. If bleeding reoccurs, more advanced treatment may be necessary. Posterior bleeds should be admitted for more definitive treatment by ENT and packing should stay in for no more than 72 hours. Future prevention can include use if saline nasal sprays or humidified air.
References:
- https://www.ncbi.nlm.nih.gov/books/NBK435997/
- UpToDate
- https://www.ncbi.nlm.nih.gov/pmc/articles/PMC3970224/
Incorrect
The correct answer is C. For most patients without a coagulopathy, 10-15 minutes is adequate to stop epistaxis. The patient should be asked to gently blow the nose into gauze or tissue in order to clear it of clots. The provider should then immediately spray oxymetazoline (Afrin) into the side of the bleeding and then have the patient tightly pinch the nares together, up against the septum. Firm, constant pressure should be maintained for at least 10 minutes, at which point the nares can be reassessed to see if additional methods of control are necessary (see below).
The etiology of epistaxis can include trauma (accidental or finger picking), dry air, rhinitis, cocaine use, coagulopathy (drug induces or genetic), friable neoplasm or vascular lesion, hypertension, alcoholism, and intranasal steroids. The vast majority (90%) of cases are considered anterior with the rest termed posterior. The majority of anterior bleeds occur in the anatomic area known as Kiesselbach’s plexus or Little’s area which is comprised of 5 arteries which supply the septum. Posterior bleeds arise from 2 arteries located in an area termed Woodruff’s plexus and can be more difficult to control due to the anatomy. They may present with more bilateral epistaxis as well as swallowing blood.
Physical exam should include a thorough evaluation of the nares with adequate lighting if bleeding volume allows after being cleared of clots. A nasal speculum inserted to open superiorly and inferiorly (as opposed to medially and laterally) can be utilized to assist in the evaluation. Suctioning can also be used. A variety of procedures can be used if bleeding does not stop after 10-15 minutes of pressure, depending on the severity and location of bleeding as well as provider comfort level. Silver nitrate can be used to seal a vessel if bleeding is minimal or has stopped. It will not work for briskly bleeding sources. Manual packing can be performed with a variety of materials including nasal tampons, petroleum impregnated gauze ribbon, or balloon catheter. Topical thrombogenics or tranexamic acid are also available. If a posterior bleed is the cause, an appropriately sized balloon device or a Foley catheter balloon inflated once in the posterior cavity are required. Acute hypertension should be controlled, and blood thinner or NSAIDS stopped after consulting with the prescribing provider. If bleeding is not controlled, operative ligation by ENT may be necessary.
Routine antibiotics are no longer recommended to prevent potential toxic shock but can be used for patients at risk or for posterior bleeds. Anterior nosebleeds should be seen, ideally by ENT, in follow-up in 3-5 days if packing was required. Longer if the patient has a coagulopathy. If bleeding reoccurs, more advanced treatment may be necessary. Posterior bleeds should be admitted for more definitive treatment by ENT and packing should stay in for no more than 72 hours. Future prevention can include use if saline nasal sprays or humidified air.
References:
- https://www.ncbi.nlm.nih.gov/books/NBK435997/
- UpToDate
- https://www.ncbi.nlm.nih.gov/pmc/articles/PMC3970224/
-
Question 53 of 56
53. Question
Which of the following is associated with the attached nasal image?Correct
The correct answer is D. The image depicts a nasal polyp. When combined with aspirin intolerance and asthma, this is referred to as Samter’s triad. Beck’s triad (muffled heart sounds, distended neck veins, hypotension) is associated with cardiac tamponade. Fallot’s pentad (pulmonary stenosis, ventricular septal defect, right ventricular hypertrophy, overriding aorta, plus patent foramen ovale or atrial septal defect) is associated with congenital heart disease. Reynold’s pentad (right upper quadrant abdominal pain, fever, jaundice, hypotension, altered mental status) is associated with ascending cholangitis. Virchow’s triad (venous stasis, hypercoagulability, vessel injury) is associated with DVT or PE risk factors.
Patients with Samter’s triad are at risk for developing aspirin-exacerbated respiratory disease (AERD). When these patients are exposed to aspirin or other NSAIDS, an enzyme called cyclooxygenase-1 is inhibited in approximately 30% of patients with both asthma and nasal polyps. In general, AERD develops rapidly, usually between the ages of 20 and 50. The disorder can involve both the upper and lower respiratory tracts as well as cause a rash, flushing, abdominal pain or vomiting. Lower respiratory symptoms may be more resistant to treatments than a typical asthma exacerbation. The diagnosis is usually made on history and symptoms accompanying aspirin or NSAID use shortly before onset. These medications should be avoided until the patient has undergone controlled desensitization.
Nasal polyps are benign growths of the nasal mucosa. They are frequently associated with chronic rhinosinusitis, cystic fibrosis, and vasculitis, amongst others. Polyps can be related to genetics or local irritants which result in excess permeability of the nasal epithelial basement membrane with subsequent edema and continued inflammation. The vast majority of inflammatory polyps are bilateral. Unilateral polyps should raise suspicion for malignancy. Complications can include sinusitis, epistaxis, sleep disruption or frank sleep apnea. Many uncomplicated polyps can be managed with intranasal steroids and saline irrigation for approximately 2-3 months. Oral steroids may be beneficial if intranasal are inadequate. Biologics or surgery are also options. Biopsy or work-up for systemic disorders such as cystic fibrosis should be considered especially if unilateral, abnormal appearance, or refractory to treatments.
References:
Incorrect
The correct answer is D. The image depicts a nasal polyp. When combined with aspirin intolerance and asthma, this is referred to as Samter’s triad. Beck’s triad (muffled heart sounds, distended neck veins, hypotension) is associated with cardiac tamponade. Fallot’s pentad (pulmonary stenosis, ventricular septal defect, right ventricular hypertrophy, overriding aorta, plus patent foramen ovale or atrial septal defect) is associated with congenital heart disease. Reynold’s pentad (right upper quadrant abdominal pain, fever, jaundice, hypotension, altered mental status) is associated with ascending cholangitis. Virchow’s triad (venous stasis, hypercoagulability, vessel injury) is associated with DVT or PE risk factors.
Patients with Samter’s triad are at risk for developing aspirin-exacerbated respiratory disease (AERD). When these patients are exposed to aspirin or other NSAIDS, an enzyme called cyclooxygenase-1 is inhibited in approximately 30% of patients with both asthma and nasal polyps. In general, AERD develops rapidly, usually between the ages of 20 and 50. The disorder can involve both the upper and lower respiratory tracts as well as cause a rash, flushing, abdominal pain or vomiting. Lower respiratory symptoms may be more resistant to treatments than a typical asthma exacerbation. The diagnosis is usually made on history and symptoms accompanying aspirin or NSAID use shortly before onset. These medications should be avoided until the patient has undergone controlled desensitization.
Nasal polyps are benign growths of the nasal mucosa. They are frequently associated with chronic rhinosinusitis, cystic fibrosis, and vasculitis, amongst others. Polyps can be related to genetics or local irritants which result in excess permeability of the nasal epithelial basement membrane with subsequent edema and continued inflammation. The vast majority of inflammatory polyps are bilateral. Unilateral polyps should raise suspicion for malignancy. Complications can include sinusitis, epistaxis, sleep disruption or frank sleep apnea. Many uncomplicated polyps can be managed with intranasal steroids and saline irrigation for approximately 2-3 months. Oral steroids may be beneficial if intranasal are inadequate. Biologics or surgery are also options. Biopsy or work-up for systemic disorders such as cystic fibrosis should be considered especially if unilateral, abnormal appearance, or refractory to treatments.
References:
-
Question 54 of 56
54. Question
What physical exam finding would most commonly be found in a child presenting with the most common etiology of rhinitis?
Correct
The correct answer is A. The most common form of rhinitis is allergic. Seasonal allergies trigger nasal itching which prompts the child to frequently rub or push the tip of the nose up, which eventually leads to a blueish-gray to purple discoloration under the eyes and around the nasal crease. Nonallergic rhinitis typically results in a normal color of the nasal mucosa while viral rhinosinusitis is associated with a beefy red appearance. Epistaxis is not common with rhinitis, rather, it is usually the result of dry membranes, forceful blowing, or trauma. Purulent discharge is associated with bacterial rhinosinusitis or a retained foreign body. Septal erosions are associated with cocaine use.
Rhinitis can occur as the result of infections, allergens, environmental irritants, various medications, changes in hormone balance, and neural system dysfunction. It is generally classified as “allergic”, “infectious”, “non-allergic, non-infectious”, and “mixed”. Symptoms usually consist of sneezing, clear drainage from the nares or down the posterior aspect which may result in cough, and/or pruritis. Allergic rhinitis is usually intermittent, more common in individuals with eczema or asthma, and frequently associated with conjunctivitis. A thorough history of potential allergen exposures is helpful in making the diagnosis. Infectious rhinitis is usually the result of a viral infection but can also be associated with a bacterial infection. Thicker discharge or other systemic symptoms such as fever, malaise, myalgias, or cough may be present. The patient may report a recent exposure to someone with similar symptoms. Non-allergic, non-infectious cases generally occur later in life and are usually perennial, associated with more congestion and less itching and sneezing. The history may reveal that strong, irritating odors trigger the symptoms. Mixed rhinitis combines feature of both allergic and non-allergic types. Patients should also be asked about using nasal decongestant sprays, CPAP for sleep apnea, cocaine, and alcohol which can cause rhinitis as well as a detailed review of systemic medications and a consideration of the patients’ medical history. Disorders that may cause rhinitis can include granulomatous disorders, cystic fibrosis, and hypothyroidism.
Treatment is targeted at the most likely underlying etiology. Avoiding known allergens and irritants is key. Oral or topical antihistamines can be helpful in minimizing symptoms. Intranasal steroids are likely the most effective medications. Oral decongestants may benefit some patients but should be avoided in patients with hypertension, cardiac disorders, and children < 1. Topical decongestants can also be effective but can result in a rebound effect within as little as 3 days, called rhinitis medicamentosa, so they should be used with caution and sparingly. Other effective treatments include nasal ipratropium bromide and oral leukotriene receptor antagonists. A referral to an allergist or otolaryngologist may be warranted if they symptoms are not being adequately managed.
References:
Incorrect
The correct answer is A. The most common form of rhinitis is allergic. Seasonal allergies trigger nasal itching which prompts the child to frequently rub or push the tip of the nose up, which eventually leads to a blueish-gray to purple discoloration under the eyes and around the nasal crease. Nonallergic rhinitis typically results in a normal color of the nasal mucosa while viral rhinosinusitis is associated with a beefy red appearance. Epistaxis is not common with rhinitis, rather, it is usually the result of dry membranes, forceful blowing, or trauma. Purulent discharge is associated with bacterial rhinosinusitis or a retained foreign body. Septal erosions are associated with cocaine use.
Rhinitis can occur as the result of infections, allergens, environmental irritants, various medications, changes in hormone balance, and neural system dysfunction. It is generally classified as “allergic”, “infectious”, “non-allergic, non-infectious”, and “mixed”. Symptoms usually consist of sneezing, clear drainage from the nares or down the posterior aspect which may result in cough, and/or pruritis. Allergic rhinitis is usually intermittent, more common in individuals with eczema or asthma, and frequently associated with conjunctivitis. A thorough history of potential allergen exposures is helpful in making the diagnosis. Infectious rhinitis is usually the result of a viral infection but can also be associated with a bacterial infection. Thicker discharge or other systemic symptoms such as fever, malaise, myalgias, or cough may be present. The patient may report a recent exposure to someone with similar symptoms. Non-allergic, non-infectious cases generally occur later in life and are usually perennial, associated with more congestion and less itching and sneezing. The history may reveal that strong, irritating odors trigger the symptoms. Mixed rhinitis combines feature of both allergic and non-allergic types. Patients should also be asked about using nasal decongestant sprays, CPAP for sleep apnea, cocaine, and alcohol which can cause rhinitis as well as a detailed review of systemic medications and a consideration of the patients’ medical history. Disorders that may cause rhinitis can include granulomatous disorders, cystic fibrosis, and hypothyroidism.
Treatment is targeted at the most likely underlying etiology. Avoiding known allergens and irritants is key. Oral or topical antihistamines can be helpful in minimizing symptoms. Intranasal steroids are likely the most effective medications. Oral decongestants may benefit some patients but should be avoided in patients with hypertension, cardiac disorders, and children < 1. Topical decongestants can also be effective but can result in a rebound effect within as little as 3 days, called rhinitis medicamentosa, so they should be used with caution and sparingly. Other effective treatments include nasal ipratropium bromide and oral leukotriene receptor antagonists. A referral to an allergist or otolaryngologist may be warranted if they symptoms are not being adequately managed.
References:
-
Question 55 of 56
55. Question
Which of the following is the most acute complication sustained from nasal trauma which requires urgent management
Correct
The correct answer is C. Patients who have sustained nasal trauma should be evaluated for the formation of a septal wall hematoma. If present, it requires urgent drainage to avoid the more intermediate and long-term complications which include the other listed answer options. A septal abscess is a common sub-acute complication of a septal hematoma, occurring within the first 72 hours. The infection can rapidly spread to nearby facial structures and even into the intracranial area. If the hematoma is not decompressed, avascular necrosis can occur which will lead to inadequate perfusion of the perichondrium which supplies nutrients to the septal cartilage. Ultimately a collapse of the nose, referred to as a saddle nose, or a septal deviation or perforation can result.
A nasal septal hematoma is an accumulation of blood under the mucoperichondrium or mucoperiosteum and usually develops quickly but can take up to 72 hours or infrequently, even longer. It can occur on one side of the septum or both. The occurrence is relatively rare but if it is not considered in the differential of the patient with nasal trauma and specifically looked for on physical exam, the resulting complications can be significant. The vast majority are secondary to trauma but occasionally may be due to surgical complications or bleeding disorders. Physical exam should include a thorough inspection as well as palpation of the septum. Inspection may reveal a blue, purple, or red area of mucosal swelling. Early hematomas may be more appreciated with palpation than inspection. A normal septum should be relatively firm and have a concave contour. A hematoma will feel softer and should have some degree of fluctuance. If oxymetazoline nasal spray is applied, a hematoma will not shrink. The diagnosis is usually clinical but may be detected on CT scan if ordered to assess for other injuries.
Treatment should be provided in consultation with an ENT provider. Local anesthesia is generally adequate but general anesthesia may be necessary in some cases. Small collections may be drained using an 18 or 20 gauge needle aspiration. Larger collections require scalpel incision, suction, excision of a small area of mucosa to prevent premature closure, followed by drain placement and bilateral packing to prevent re-accumulation. The packing and drain are usually removed when there has been no drainage for 24 hours. Prophylactic antibiotics are prescribed.
The prognosis is good if early recognition and intervention occur. In addition to the complications discussed above, other potential problems include cavernous sinus thrombosis, nasal vestibulitis, and secondary bacteremia. Close follow-up after drainage is important to ensure re-accumulation or infection are not occurring. If a hematoma is not appreciated on the initial evaluation, the patient or responsible caregiver should be advised to seek immediate re-evaluation if increasing pain or the feeling of increased septal swelling is occurring.
References:
Incorrect
The correct answer is C. Patients who have sustained nasal trauma should be evaluated for the formation of a septal wall hematoma. If present, it requires urgent drainage to avoid the more intermediate and long-term complications which include the other listed answer options. A septal abscess is a common sub-acute complication of a septal hematoma, occurring within the first 72 hours. The infection can rapidly spread to nearby facial structures and even into the intracranial area. If the hematoma is not decompressed, avascular necrosis can occur which will lead to inadequate perfusion of the perichondrium which supplies nutrients to the septal cartilage. Ultimately a collapse of the nose, referred to as a saddle nose, or a septal deviation or perforation can result.
A nasal septal hematoma is an accumulation of blood under the mucoperichondrium or mucoperiosteum and usually develops quickly but can take up to 72 hours or infrequently, even longer. It can occur on one side of the septum or both. The occurrence is relatively rare but if it is not considered in the differential of the patient with nasal trauma and specifically looked for on physical exam, the resulting complications can be significant. The vast majority are secondary to trauma but occasionally may be due to surgical complications or bleeding disorders. Physical exam should include a thorough inspection as well as palpation of the septum. Inspection may reveal a blue, purple, or red area of mucosal swelling. Early hematomas may be more appreciated with palpation than inspection. A normal septum should be relatively firm and have a concave contour. A hematoma will feel softer and should have some degree of fluctuance. If oxymetazoline nasal spray is applied, a hematoma will not shrink. The diagnosis is usually clinical but may be detected on CT scan if ordered to assess for other injuries.
Treatment should be provided in consultation with an ENT provider. Local anesthesia is generally adequate but general anesthesia may be necessary in some cases. Small collections may be drained using an 18 or 20 gauge needle aspiration. Larger collections require scalpel incision, suction, excision of a small area of mucosa to prevent premature closure, followed by drain placement and bilateral packing to prevent re-accumulation. The packing and drain are usually removed when there has been no drainage for 24 hours. Prophylactic antibiotics are prescribed.
The prognosis is good if early recognition and intervention occur. In addition to the complications discussed above, other potential problems include cavernous sinus thrombosis, nasal vestibulitis, and secondary bacteremia. Close follow-up after drainage is important to ensure re-accumulation or infection are not occurring. If a hematoma is not appreciated on the initial evaluation, the patient or responsible caregiver should be advised to seek immediate re-evaluation if increasing pain or the feeling of increased septal swelling is occurring.
References:
-
Question 56 of 56
56. Question
A patient sustains a direct blow to the nose from a thrown baseball. There is moderate swelling and possible deformity visible. Mild epistaxis is present but no visible septal hematoma. What is the most appropriate plan?
Correct
The correct answer is C. While answer A is not completely inappropriate, it is generally preferred to wait for the swelling to subside to be able to better visualize cosmetic alignment. In addition, if a stabilizing splint is required after manipulation, it may not provide adequate support once swelling has subsided if applied too early. Waiting too long before manipulation could result in partial healing, making the fracture more difficult to realign and possible nonunion. Nasal packing is not indicated unless uncontrolled epistaxis or septal hematoma is present.
Nasal bone fractures are the most common facial fractures. An appropriate history and exam can help make the diagnosis and minimize unnecessary imaging. When trauma comes from the side of the nose, lateral displacement is highly likely. The more forceful the trauma, the more likely it will result in bilateral nasal bone fractures and displacement requiring realignment as well as possible septal hematoma. Most patients will report the feeling of occlusion to the nares or the sensation of turbulent air when inhaling. Trauma that contacts the nose directly from the front more commonly damages the tip of the nasal bone(s) and can result in some degree of a depressed fracture in addition to pushing the bone fragments laterally causing a saddle deformity. The septum may also be involved. After any nasal trauma, inspect and palpate for the presence of a septal hematoma. Document if present or absent. Children are more likely to have a septal injury with significant trauma rather than fractures due to the fact that the majority of the nose is cartilage.
Imaging can usually be avoided or delayed until swelling has subsided unless there is a concern for other facial fractures. Facial numbness, double or blurred vision, changes noticed when biting down, or rhinorrhea indicate that the injury may not be isolated and a CT scan is warranted. If a septal hematoma is appreciated on exam, it should be drained in order to avoid necrosis and permanent deformity.
When required for cosmetic or functional reasons, reduction should ideally occur within 1-2 weeks following the injury for adults and 7-10 days for children due to their tendency to heal more rapidly. The delay in treatment is to allow enough time for the swelling to subside so that realignment can be better visualized by the provider but before the bones have started to heal and stabilize. If a stabilizing splint is needed, the delay in care also allows for a better fitting splint and less chance that it will become loose prior to adequate healing. After 2 weeks from injury, an open technique may be necessary to adequately realign the bones. The reduction will be made more pleasant for the patient if mild general sedation is provided. Next, an equal mixture of 4% topical lidocaine and oxymetazoline should be applied to the septum. This can provide some additional anesthesia for the procedure and reduce bleeding. Antibiotics are generally unnecessary unless a septal hematoma or abscess has developed. Packing is usually unnecessary unless required to control bleeding from disrupting a clot from the initial injury.
References:
Incorrect
The correct answer is C. While answer A is not completely inappropriate, it is generally preferred to wait for the swelling to subside to be able to better visualize cosmetic alignment. In addition, if a stabilizing splint is required after manipulation, it may not provide adequate support once swelling has subsided if applied too early. Waiting too long before manipulation could result in partial healing, making the fracture more difficult to realign and possible nonunion. Nasal packing is not indicated unless uncontrolled epistaxis or septal hematoma is present.
Nasal bone fractures are the most common facial fractures. An appropriate history and exam can help make the diagnosis and minimize unnecessary imaging. When trauma comes from the side of the nose, lateral displacement is highly likely. The more forceful the trauma, the more likely it will result in bilateral nasal bone fractures and displacement requiring realignment as well as possible septal hematoma. Most patients will report the feeling of occlusion to the nares or the sensation of turbulent air when inhaling. Trauma that contacts the nose directly from the front more commonly damages the tip of the nasal bone(s) and can result in some degree of a depressed fracture in addition to pushing the bone fragments laterally causing a saddle deformity. The septum may also be involved. After any nasal trauma, inspect and palpate for the presence of a septal hematoma. Document if present or absent. Children are more likely to have a septal injury with significant trauma rather than fractures due to the fact that the majority of the nose is cartilage.
Imaging can usually be avoided or delayed until swelling has subsided unless there is a concern for other facial fractures. Facial numbness, double or blurred vision, changes noticed when biting down, or rhinorrhea indicate that the injury may not be isolated and a CT scan is warranted. If a septal hematoma is appreciated on exam, it should be drained in order to avoid necrosis and permanent deformity.
When required for cosmetic or functional reasons, reduction should ideally occur within 1-2 weeks following the injury for adults and 7-10 days for children due to their tendency to heal more rapidly. The delay in treatment is to allow enough time for the swelling to subside so that realignment can be better visualized by the provider but before the bones have started to heal and stabilize. If a stabilizing splint is needed, the delay in care also allows for a better fitting splint and less chance that it will become loose prior to adequate healing. After 2 weeks from injury, an open technique may be necessary to adequately realign the bones. The reduction will be made more pleasant for the patient if mild general sedation is provided. Next, an equal mixture of 4% topical lidocaine and oxymetazoline should be applied to the septum. This can provide some additional anesthesia for the procedure and reduce bleeding. Antibiotics are generally unnecessary unless a septal hematoma or abscess has developed. Packing is usually unnecessary unless required to control bleeding from disrupting a clot from the initial injury.
References: